ORTHOPEDIC MCQS ONLINE BANK OITE 22
ORTHOPEDIC MCQS ONLINE BANK OITE 22
FOR OITE 22 FIGURES CLICK OITE22FIG
02.1
- A 30-year-old patient sustains an isolated type II open femoral shaft fracture. In addition to irrigation and debridement, management should include
- 1- skeletal traction.
- 2- unlocked unreamed intramedullary nailing.
- 3- locked reamed intramedullary nailing.
- 4- external fixation.
- 5- open reduction and internal fixation with a plate and screws.
- Question 02.01
- Answer = 3
- Reference(s)
- Brumback RJ, Ellison PS Jr, Poka A, Lakatos R, Bathon GH, Burgess AR eds Intramedullary nailing of open fractures of the femoral shaft. J Bone Joint Surg Am 1989;71:1324-1331.
- O'Brien PJ, Meek RN, Powell JN Blachut PA eds Primary intramedullary nailing of open femoral shaft fractures. J Trauma 1991;31:113-
- 02.2
- answer
- back
- What abnormality at the cellular level causes fibrous dysplasia?
- 1- An activating mutation of the G protein
- 2- A germline loss of function mutation of the G protein
- 3- A single X chromosome
- 4- Abnormal fibrillin
- 5- Abnormal neurofibrillin
- Question 02.02
- Answer = 1
- back to this question
- next question
- Reference(s)
- Lefkowitz RJ: G proteins in medicine. N Engl J Med 1995;332:186-187.
- Zaleske D: Metabolic and endocrine abnormalities, in Morrissy RT., Weinstein SL (ed.): Lovell and Winter's Pediatric Orthopaedics, ed 5. Philadelphia, PA, Lippincott Williams & Wilkins, 2001, pp 177-241.
- 02.3
- answer
- back
- Which of the following is considered a typical characteristic of deep lesions in articular cartilage that cross the tidemark in adults?
- 1- Heals with intramembranous ossification
- 2- Heals with hyaline cartilage
- 3- Contains abundant type I collagen at 1 year
- 4- Predominately repopulated from surrounding chondrocytes
- 5- Contains abundant aggrecan at 1 year
- Question 02.03
- Answer = 3
- back to this question
- next question
- Reference(s)
- Chen FS, Frenkel SR, Di Cesare PE: Repair of articular cartilage defects: Part I. Basic science of cartilage healing. Am J Orthop 1999;28:31-33.
- Chen FS, Frenkel SR, Di Cesare PE: Repair of articular cartilage defects: Part II. Treatment options. Am J Orthop 1999;28:88-96.
- 02.4
- answer
- back
- When treating a distal radius fracture by closed reduction and external fixation, palmar tilt can be restored by combining longitudinal traction with
- 1- wrist flexion.
- 2- wrist extension.
- 3- volar shift of the carpus
- 4- forearm supination
- 5- forearm pronation
- Question 02.04
- Answer = 3
- Reference(s)
- Agee JM External fixation: Technical advances based upon multiplanar ligamentotaxis. Orthop Clin North Am 1993;24:265-274.
- Dee W, Klein W, Rieger H: Reduction techniques in distal radius fractures. Injury 2000;31:48-55.
- 02.5
- What is the most common site of regional osteoporosis and pathologic fracture in women after spinal cord injury?
- 1- Spine
- 2- Proximal lanterns
- 3- Proximal femur
- 4- Distal radius radius
- 5- Distal femur
- Question 02.05
- Answer = 5
- Reference(s)
- Garland DE, Adkins RH Stewart CA, Ashford R, Vigil D: Regional osteoporosis in women who have a complete spinal cord injury. J Bone Joint Surg Am 2001;83:1195-2000.
- Eichenholtz SN: Management of long-bone fractures in paraplegic patients. J Bone Joint Surg Am 1963;45:299-310.
- 02.06
- answer
- back
- Following total hip arthroplasty, patients with Parkinson's disease would be expected to attain equivalent results to patients without neuromuscular disease, in which of the following clinical areas,
- 1 Postoperative complications
- 2 Reoperation rate
- 3 Perioperative mortality rate
- 4 Dislocation rate
- 5 Rate of aseptic loosening
- Question 02.06
- Answer = 5
- back to this question
- next question
- Reference(s)
- Cabancla ME, Weber M: Total hip arthroplasty in patients with neuromuscular disease. Instr Course Lect 2000;49:163-168
- 02.7
- answer
- back
- What is the typical location for elastofibroma injury ?
- 1- Between the subscapularis and scapula
- 2- Paraspinal musculature of the spine
- 3- Anterior chest wall beneath the pectoralis major
- 4- Posterior chest wall under the scapula
- 5- Within the latissimus dorsi
- Question 02.07
- Answer = 4
- back to this question
- next question
- Reference(s)
- Yamamoto T, Akisue T, Kurosaka M, Mizuno K, Mukai H: Elastofibroma in shoulder osteoarthritis: A theoretical concept of the etiology. Clin Orthop 2001;387:127-131.
- Enzinger FM, Weiss SW: Elastofibroma, in Enzinger FM, Weiss SW ed Soft Tissue Tumors. St Louis, MO, Mosby, 1983, pp 122-127.
- 02.8
- answer
- back
- When performing open reduction and internal Fixation of u displaced fracture of the scapular neck, which of the following peripheral nerves is at the greatest risk of injury
- 1. Axillarv
- 2. Musculocutaneous
- 3. Upper subscapular
- 4. Radial
- 5 Dorsal scapular
- Question 02.08
- Answer = 1
- back to this question
- next question
- Reference(s)
- Goss TP Fractures of the scapula, in lannotti JP, Williams GR. Jr eds Disorders of the Shoulder: Diagnosis and Management. Philadelphia, PA, Lippincott Williams & Wilkins, 1999, pp 619-620.
- Job, C: Gross anatomy of the shoulder, in Matsen FA III, Rockwood CA Jr eds The Shoulder. Philadelphia, PA, WB Saunders, 1990, pp 34-97.
- 02.9
- A 29 year old woman undergoes an inside-out medial meniscus repair. Two weeks after surgery, the patient reports constant burning pain in the knee and proximal leg What is the most likely diagnosis?
- 1- Deep vein thrombosis
- 2- Failed meniscal repair
- 3- AVN malformation
- 4- Saphenous nerve injury
- 5- Popliteal nerve injury
- Question 02.09
- Answer = 4
- Reference(s)
- Small NC: Complications in arthroscopy: The knee and other joints. Arthroscopy 1986;2:253-258.
- Austin KS, Sherman OH: Complications of arthroscopic meniscal repair. Am J Sports Med 1993;21:864-869.
- 02.10
- answer
- back
- Which of the following is considered a serious and possibly irreversible side effect of anabolic steroid use?
- 1- Hypertension
- Depression
- 3- Growth Retardation
- 4- Decreased glucose tolerance
- 5- Increased total cholesterol
- Question 02.10
- Answer = 3
- back to this question
- next question
- Reference(s)
- Silver MD: Use of ergogenic aids by athletes. J Am Acad Orthop Surg 2001;9:61-70. Adolescents and anabolic steroids: A subjective review. American Academy of Pediatrics Committee on Sports Medicine and Fitness. Pediatrics 1997;99:904-908.
- 02.11
- answer
- back
- A 6 year old boy sustained a mildly displaced closed fracture of the left proximal tibia and fibula. His parents report that prior to the fracture he had no symptoms or medical problems. The patient has no pain or fever, and examination reveals no tenderness, swelling, or palpable mass a the fracture site. Radiographs obtained at the time of injury show no lytic lesions. Figure 1 shows a follow-up radiograph obtained at the time of cast removal. The next most appropriate step in management should be
- 1 observation with repeat radiographs in a week
- 2 needle aspiration for culture of the fracture site
- 3 open biopsy and curettage of the lesion.
- 4 skeletal survey
- 5 MRI of the tibia.
- Figures 1
- Question 02.11
- Answer = 1
- back to this question
- next question
- Reference(s):
- Ball CM, Dawe CJ: Transient posttraurnatic cystlike lesions of bone. J Pediatr Orthop 2001;21:9-13.
- Malghem J, Maldague B, Claus D, Clapuyt P: Transient cyst-like cortical defects following fractures in children: Medullary fat within the subperiosteal haomatoma. J Bone Joint Surg Br 1990;72:862-865.
- 02.12
- answer
- back
- A 78 year old man is referred for evaluation of his shoulder, as shown in Figure 2. The patient has very minimal pain and denies any history of injury. His medical history is unremarkable. Examination shows poor active range of motion. What is the next most appropriate step in evaluation?
- Serum protein electrophoresis
- Incisional biopsy
- Skeletal survey
- Ct of the chest, abdomen, and pelvis
- MRI of the C-spine
- Figures 2
- Question 02.12
- Answer = 5
- back to this question
- next question
- Reference(s)
- Sequeira W: The neuropathic joint. Clin Exp. Rheumatol 1994;12:325-337.
- 02.13
- Which of the following is considered the most important factor when predicting outcome in patients with fractures of the sacrum?
- 1- Amount of fracture displacement
- Presence of vertical instability
- 3- Presence of neurologic compromise
- 4- Presence of rami fractures
- 5- Use of surgical stabilization
- Question 02.13
- Answer = 3
- Reference(s)
- Kellam JF, Fischer TJ, Torrentta P 111, Bosse MJ Harris MB (eds): Orthopaedic Knowledge Update: Trauma 2. Rosemont, IL, American Academy of Orthopaedic Surgeons, 2000, pp 277-281.
- Miranda MA, Riemer BL, Butterfield SL, Burke CJ III: Pelvic ring injuries: A long term functional outcome study. Clin Orthop 1996;329:152-159
- 02.14
- answer
- back
- Which of the following test results suggests that a plantar foot ulcer will heal in a patient with type I diabetes mellitus?
- 1- Arterial brachial index of greater than 1
- 2- Normal hemoglobin A3
- 3- Arterial toe pressures of greater than 45 mm Hg
- 4- Toe capillary refill time of greater than 3 seconds
- 5- Biphasic waveform on arterial Doppler studies
- Question 02.14
- Answer = 3
- back to this question
- next question
- Reference(s)
- Brodsky JW The diabetic foot, in Coughlin MJ Mann RA eds Surgery of the Foot and Ankle, ed 7.St Louis, MO, Mosby Publishers, 1999, pp 895-969.
- Apelqvist J, Castenfors J, Larsson J, Stenstrom A, Agardh CD: Prognostic value of systolic ankle and toe blood pressure levels in outcome of diabetic foot ulcer. Diabetes Care 1989;12:373-378.
- 02.15
- answer
- back
- An otherwise healthy 66-year-old woman has right thigh pain and is unable to bear weight after falling. The patient underwent a primary total hip arthroplasty 12 years ago and has had gradually worsening start-up pain for the past 2 years. Radiographs are shown in Figures 3a and 3b. Management should consist of
- 1-traction followed by functional bracing.
- 2-open reduction, dual strut bone grafting, and cable fixation with component retention.
- 3-open reduction and plating (cerclage wires proximal, screws distal) with component retention.
- 4-revision to a long-stem femoral component with strut bone grafting and cables.
- 5-impaction grafting after internal fixation of the fracture.
- Figures 3b
- Figures 3a
- Question 02.15
- Answer = 4
- back to this question
- next question
- Reference(s)
- Crockarell JR Jr, Berry DJ, Lewallen DG: Nonunion after periprosthetic femoral fracture associated with total hip arthroplasty. J Bone Joint Surg Am 1999;81:1073-1079.
- Siegmeth A, Menth-Chiari WA, Wozasek GE, Vecsei V: Femur fractures in patients with hip arthroplasty: Indications for revision arthroplasty. J South Orthop Assoc 1998;7:251-258.
- Incavo SJ Beard DM, Puppmo F, Ries M, Wiedel J: One-stage revision of periprosthetic fractures around loose cemented total hip arthroplasty. Am J Orthop 1998;27:35-41.
- 02.16
- answer
- back
- At what time after fracture is there a maximal vascular response (blood flow rate) at the fracture site maximized?
- 1- Immediately after injury
- 2- 2 weeks
- 3- 4 weeks
- 4- 6 weeks
- 5- 12 weeks
- Question 02.16
- Answer = 2
- back to this question
- next question
- Reference(s)
- Buckwalter JA, Einhorn TA, Simon SR eds Orthopaedic Basic Science: Biology and Biomechanics of the Musculoskeletal System, ed 2. Rosemont, IL, American Academy of Orthopaedic Surgeons, 2000, pp 372-399.
- Browner BD, Jupiter JB Levine AM, Trafton PG eds Skeletal Trauma, ed 2. Philadelphia, PA, WB Saunders, 1998, pp 59-68.
- 02.17
- A 51-year-old woman has neck and left arm pain. Sagittal and axial MR1 scans are shown in Figures 4a and 4b. What is the most likely neurologic abnormality caused by this condition?
- 1- Left-sided Babinski sign
- 2- Decreased left brachioradialis reflex
- 3- Decreased left triceps reflex
- 4- Weak left wrist extensors
- 5- Diminished sensation over the left thumb
- Figures 4a
- Figures 4b
- Question 02.17
- Answer = 3
- Reference(s)
- Hoppenfeld S: Evaluation of nerve root lesions involving the upper extremity, in Hoppenfeld S ed Orthopaedic Neurology: A Diagnostic Guide to Neurologic Levels. Philadelphia, PA, JB Lippincott, 1977, pp 7-44.
- Klimkiewicz JJ, Scherping SC: Physical diagnosis, in Wiesel SW, Delahay JN eds Principles of Orthopaedic Medicine and Surgery. Philadelphia, PA, WB Saunders, 2001, pp 51-70.
- 02.18
- answer
- back
- In the fully flexed finger, initiation of metacarpophalangeal joint extension occurs through the
- action of the extensor digitorum communis through the sagittal band.
- action of the extensor mechanism through the central slip.
- contraction of the lumbrical and relaxation of the flexor digitorum profundis.
- contraction of the interossei.
- tenodesis effect of the extensor mechanism as the wrist is actively flexed.
- Question 02.18
- Answer = 1
- back to this question
- next question
- Reference(s)
- Buckwalter JA, Einhorn TA, Simon SR eds Orthopaedic Basic Science: Biology and Biomechanics of the Musculoskeletal System, ed 2. Rosemont, IL, American Academy of Orthopaedic Surgeons, 2000, pp 732-827.
- 02.19
- answer
- back
- A 14-year-old girl who is 1 year postmenarche has a progressive, apex right thoracolumbar scoliosis that measures 55° by the Cobb method. She also has anonprogressive grade I isthmic spondylolisthesis at the lumbosacral junction. She reports no pain, and her neurologic examination is normal. Management should consist of arthrodesis of
- 1- the spondylolisthesis and observation of the scoliosis.
- 2- the spondylolisthesis and bracing of the scoliosis.
- 3- the scoliosis and observation of the spondylolisthesis.
- 4- both conditions as one long fusion.
- 5- both conditions as two separate areas of fusion.
- Question 02.19
- Answer = 3
- back to this question
- next question
- Reference(s)
- Fisk JR, Moe JH Winter RB Scoliosis spondylolysis, and spondylolisthesis: Their relationship as reviewed in 539 patients. Spine 1978;3:234-245.
- Bradford DS: Spondylolysis and spondylolistheses, in Lonstein JE Bradford DS, Winter RB
- Ogilvie JW eds Moe's Textbook of Scoliosis and Other Spinal Deformities, ed 3. Philadelphia, PA, WB Saunders, 1995,pp 399-430.
- 02.20
- answer
back
- Removal of symptomatic hardware from the fibula following open reduction and internal fixation of ankle fractures has been associated with
- 1- a higher rate of refracture.
- 2- a 10% incidence of infection.
- 3- a 10% incidence of wound breakdown.
- 4- inconsistent improvement in symptoms.
- 5- no change in the SF-36 score.
- Question 02.20
- Answer = 4
- Reference(s)
- Brown OL Dirschl DR, Obremskey WT: Incidence of hardware-related pain and its effect on functional outcomes after open reduction and internal fixation of ankle fractures. J Orthop Trauma2001;15:271-274.
- Jacobsen S, Honnens de Lichtenberg M, Jensen CM, Torholm C: Removal of internal fixation - the effect on patients' complaints: A study of 66 cases of removal of internal fixation after malleolar fractures. Foot Ankle Int 1994;15:170-171.
- 02.21
- A 40-year-old patient has a palpable anterior tibial mass that is painful with activity. Figures 5a and 5b show T1- and T2-weighted MRI scans. To help determine the diagnosis, the best course of action is
- 1- en bloc excisional biopsy.
- 2- needle aspiration.
- 3- CT.
- 4- electromyography and nerve conduction velocity studies.
- 5- PET.
- Figures 5a
- Figures 5b
- Question 02.21
- Answer = 2
- back to this question
- next question
- Reference(s)
- Tread-11 EL: Synovial cysts and ganglia: The value of magnetic resonance imaging. Semin Arthritis Rheum 1994;24:61-70 .
- 02.22
- answer
- back
- What structure is at greatest risk when the anteriomedial portal is used during elbow arthroscopy?
- 1- Brachial artery
- 2- Ulnar nerve
- 3- Median nerve
- 4- Posterior interosseous nerve
- 5- Biceps tendon insertion
- Question 02.22
- Answer = 3
- back to this question
- next question
- Reference(s)
- Morrey BF: Complications of elbow arthroscopy. Instr Course Lect 2000;49:255-258.
- Lyons TR Field LD Savoie FH III: Basics of elbow arthroscopy. Instr Course Lect 2000;49:239-246.
- 02.23
- answer
- back
- A 35-year-old woman has chronic pain in the interspace between the third and fourth toes that occasionally radiates distally into the toes. The foot examination is otherwise normal. Management consisting of lidocaine and steroid injections provides only temporary relief. The interspace is surgically explored, and the digital nerve appears normal. The next most appropriate step in management should consist of
- 1- excision of the interdigital nerve as far proximal in the interspace as possible.
- 2- decompression of the nerve with release of the transverse intermetatarsal ligament.
- 3- exploration of the adjacent second and fourth interspaces.
- 4- intrafascicular release of the interdigital nerve.
- 5- dorsiflexion osteotomy of the third and fourth metatarsals.
- Question 02.23
- Answer = 2
- back to this question
- next question
- Reference(s)
- Mizel MS, Miller RA, Scioli MW (eds): Orthopaedic Knowledge Update: Foot and Ankle 2. Rosemont, IL, American Academy of Orthopaedic Surgeons, 1998, pp 101-111.
- Okafor B, Shergill G, Angel J: Treatment of Morton's neuroma by neurolysis. Foot Ankle Int 1997;18:284-287.
- Gauthier G: Thomas Morton's disease: A nerve entrapment syndrome. A new surgical technique. Clin Orthop 1979;142:90-92.
- 02.24
- Healing after complete midsubstance laceration of skeletal muscle in an adult is characterized by
- l- hypertrophy of the distal muscle.
- 2- dense connective scar tissue.
- 3- complete functional recovery.
- 4- denervation of the proximal muscle portion.
- 5- proximal muscle atrophy.
- Question 02.24
- Answer = 2
- Reference(s)
- Buckwalter JA, Einhorn TA, Simon SR (eds): Orthopaedic Basic Science: Biology and Biomechanicsof the Musculoskeletal System, ed 2. Rosemont, IL, American Academy of Orthopaedic Surgeons, 2000, pp 683-716.
- Menetrey J, Kasemkijwattana C, Fu FH, Moreland MS, Huard J: Suturing versus immobilization of a muscle laceration: A morphological and functional study in a mouse model. Am J Sports Med 1999;27:222-229.
- 02.25
- answer
- back
- Radiographs of an adolescent who had metatarsus adductus as an infant that was refractory to treatment may show which of the following findings?
- 1- Obliquity of the medial cuneiform
- 2- Superior placement of the tarsal navicular on the head of the talus
- 3- Medial subluxation of the cuboid
- 4- Lateral subluxation of the middle and lateral cuneiforms
- 5- Coalition of the cuboid and calcareus
- Question 02.25
- Answer = 1
- back to this question
- next question
- Reference(s)
- Farsetti P, Weinstein SL, Ponseti IV: The long-term functional and radiographic outcomes ofuntreated and non-operatively treated metatarsus adductus. J Bone Joint Surg Am 1994;76:257-265.
- Marcinko DE, Hetico HR: Structural metatarsus adductus deformity: Surgical case report. J FootSurg 1992;31:607-610.
- Morcuende JA, Ponseti IV: Congenital metatarsus adductus in early human fetal development: A histologic study. Chin Orthop 1996;333:261-266.
- 02.26
- answer
- back
- What is the most appropriate treatment for a patient with osteonecrosis of the femoral head with degenerative acetabular changes?
- 1- Core decompression
- 2- Bipolar hemiarthroplasty
- 3- Resurfacing hemiarthroplasty
- 4- Total hip arthroplasty
- 5- Vascularized fibular autografting
- Question 02.26
- Answer = 4
- back to this question
- next question
- Reference(s)
- Judet H, Gilbert A: Long-term results of free vascularized fibular grafting for femoral head necrosis.Clin Orthop 2001;386:114-119.
- Steinberg ME, Larcom PG, Stafford B, Hosick WB, Corces A, Bands RE, Hartman KE Coredecompression with bone grafting for osteonecrosis of the femoral head. Clin Orthop 2001;386:71-78.
- Siguier T, Siguier M, Judet T, Charnley G, Brumpt B: Partial resurfacing arthroplasty of the femoral head in avascular necrosis: Methods, indications, and results. Clin Orthop 2001;386:85-92.
- 02.27
- answer
- back
- In the surgical management of comminuted talar neck fractures, what is the most common position of malreduction`?
- 1- Varus
- 2- Valgus
- 3- Flexion
- 4- Internal rotation
- 5- External rotation
- Question 02.27
- Answer = 1
- back to this question
- next question
- Reference(s)
- Coughlin MJ Mann RA eds Surgery of the Foot and Ankle, ed 6. St Louis, MO, Mosby Publishers,1993, p 1554.
- Beaty JH ed Orthopaedic Knowledge Update 6. Rosemont, IL, American Academy of Orthopaedic Surgeons, 1999, pp 597-612.
- 02.28
- A 17-year-old pitcher reports pain over the medial aspect of the elbow that occurs during the acceleration phase of throwing, and it prevents him from throwing at the velocity needed to be competitive. What structure likely has been injured?
- 1- Pronator teres
- 2- Flexor carpi uInaris
- 3- Radial collateral ligament
- 4- Anterior bundle of the u1nar collateral ligament
- 5- Posterior bundle of the uInar collateral ligament
- Question 02.28
- Answer = 4
- Reference(s)
- Rettig AC, Sherrill C, Snead DS, Mendler JC Mieling P: Nonoperative treatment of u1nar collateral ligament injuries in throwing athletes. Am J Sports Med 2001;29:15-17.
- Conway JE Jobe FW Glousman RE, Pink M: Medial instability of the elbow in throwing athletes:Treatment by repair or reconstruction of the u1nar collateral ligament. J Bone Joint Surg Am1992;74:67-83.
- 02.29
- answer
- back
- An 8-year-old boy sustained a distal radius shaft fracture with disruption of the distal radioulnar joint. The distal ulna is dislocated dorsally but is clinically reducible. Initial management should consist of closed reduction and application of along arm cast with the
- 1- thumb in abduction.
- 2- forearm in supination
- 3- forearm in pronation
- 4- wrist in extension.
- 5- wrist in radial deviation.
- Question 02.29
- Answer = 2
- back to this question
- next question
- Reference(s)
- Letts M, Rowhani N: Galeazzi-equivalent injuries of the wrist in children. J Pediatr Orthop 1993;13:561-566.
- Campbell RM Jr: Operative treatment of fractures and dislocations of the hand and wrist region in children. Orthop Clin North Am 1990;21:217-243.
- 02.30
- answer
- back
- What is the most common problem following a Darrach procedure (distal ulna resection)?
- 1- Proximal ulna stump instability
- 2- Extensor carpi ulnaris subluxation
- 3- Injury of the dorsal branch of the uInar nerve
- 4- Decreased forearm rotation arc
- 5- Ulnar translation of the carpus
- Question 02.30
- Answer = 1
- back to this question
- next question
- Reference(s)
- Nanchahal J, Sykes PJ, Williams RL Excision of the distal ulna in rheumatoid arthritis: Is the price too high? J Hand Surg Br 1996;21:189-196.
- Tulipan DJ, Eaton RG Eberhart RE: The Darrach procedure defended: Technique redefined and long-term follow-up. J Hand Surg Am 1991;16:438-444.
- 02.31
- answer
- back
- What type of neurologic monitoring is most accurate during reduction of a grade III spondylolisthesis?
- 1- Electromyography
- 2- Electroencephalographs
- 3- Somatosensory-evoked potential
- 4- Motor-evoked potential
- 5- Clonus test
- Question 02.31
- Answer = 1
- back to this question
- next question
- Reference(s)
- Herdmann J, Deletis V. Edmonds HL Jr, Morota N: Spinal cord and nerve root monitoring in spine surgery and related procedures. Spine 1996;21:879-885.
- Moed BR, Hartman MJ Ahmed BK Cody DD, Craig JG Evaluation of intraoperative nerve-monitoring during insertion of an iliosacral implant in an animal model. J Bone Joint Surg Am 1999;81:1529-1537.
- 02.32
- A previously active 65-year-old woman with unilateral knee osteoarthritis underwent an uncomplicated, cemented total knee arthroplasty 6 months ago. She now participates in a regular, low-impact aerobics program. Her participation in this type of activity is most likely to lead to
- 1- implant loosening.
- 2- polyethylene wear.
- 3- periprosthetic fracture.
- 4- hip pain.
- 5- a good functional outcome.
- Question 02.32
- Answer = 5
- Reference(s)
- Healy WL Iorio R, Lemos MJ Athletic activity after joint replacement. Am J Sports Med2001;29:377-388.
- Bradbury N, Borton D, Spoo G, Cross MJ Participation in sports after total knee replacement. Am J Sports Med 1998;26:530-535.
- 02.33
- answer
- back
- A study of patients undergoing total hip arthroplasty has been completed that compares the length of time it takes the patients to walk without aids to their age. What statistical technique should be used to analyze the data?
- 1- Student's t test
- 2- Analysis of variance
- 3- Linear regression
- 4- Chi-square analysis
- 5- Frequency analysis
- Question 02.33
- Answer = 3
- back to this question
- next question
- Reference(s)
- Buckwalter JA, Einhorn TA, Simon SR eds Orthopaedic Basic Science: Biology and Biornechanicsof the Musculoskeletal System, ed 2. Rosemont, IL, American Academy of Orthopaedic Surgeons,2000, pp 2-17.
- Kaawach W, Ecklund K, Di Canzio J, Zurakowski D, Waters PM: Normal ranges of scapholunate distance in children 6 to 14 years old. J Pediatr Orthop 2001;21:464-467.
- 02.34
- answer
- back
- A 35-year-man sustained an injury to his Achilles tendon I year ago. He was treated in a cast for 6 weeks. Examination reveals weakness in ankle plantar flexion and a 4-cm defect by palpation. Use of an ankle-foot orthosis improves his function, but he desires amore permanent correction. Treatment should now consist of
- 1- ankle arthrodesis.
- 2- debridement and end-to-end repair of the Achilles tendon with the foot in full plantar flexion.
- 3- transfer of the flexor hallucis longus tendon to the calcaneus.
- 4- transfer of the anterior tibialis tendon through the interosseous membrane to the calcaneus.
- 5- recession of the gastrocnemius muscle with tendon advancement and repair.
- Question 02.34
- Answer = 3
- back to this question
- next question
- Reference(s)
- Coughlin MJ Disorders of tendons, in Coughlin MJ Mann RA eds Surgery of the Foot and Ankle, ed 7. St Louis, MO, Mosby Publishers, 1999, pp 786-861.
- Wapner KL Pavlock GS, Hecht PJ, Naelli F, Walther R: Repair of chronic Achilles tendon rupture with flexor hallucis longus tendon transfer. Foot Ankle 1993;14:443-449.
- Gabel S, Manoli A II: Neglected rupture of the Achilles tendon. Foot Ankle Int 1994;15:512-517.
- 02.35
- Reconstruction or prosthetic replacement of the radial head is most important following
- an acute radial head fracture with associated distal radioulnar joint instability because the radial head is an important
- 1- restraint to varus stress.
- 2- restraint to valgus stress.
- 3- restraint to proximal migration of the radius.
- 4- enabler of elbow flexion and extension.
- 5- enabler of forearm rotation.
- answer
- back
- Question 02.35
- Answer = 3
- back to this question
- next question
- Reference(s)
- Kellam JF, Fischer TJ, Tornetta P III, Bosse MJ Harris MB eds Orthopaedic Knowledge Update: Trauma 2. Rosemont, IL, American Academy of Orthopaedic Surgeons, 2000, pp 39-51
- 02.36
- The normal capsular anatomy of the older child provides increased opportunity for an
- acute hematogenous osteomyelitis of the metaphysic to extend to an associated septic
- arthritis at the
- 1- distal lateral tibia/fibula and distal femur.
- 2- distal first metacarpal and proximal tibia.
- 3- proximal radius and proximal tibia.
- 4- proximal femur and proximal radius.
- 5- proximal humerus and distal first metacarpal.
- Question 02.36
- Answer = 4
- Reference(s)
- Morrissy RT.: Bone and joint sepsis, in Morrissy RT., Weinstein SL eds Lovell and Winter's Pediatric Orthopaedics, ed 5. Philadelphia, PA, Lippincott Williams & Wilkins, 2001, pp 459-507.
- Herring JA: Bone and joint infections, in Herring JA ed Tachdjian's Pediatric Orthopaedics, ed 3. Philadelphia, PA, WB Saunders, 2002, pp 1841-1879.
- 02.37
- Which of the following is an effect of insulin in muscle?
- 1- Increased glycogen synthesis
- 2- Increased protein catabolism
- 3- Decreased amino acid uptake
- 4- Decreased ribosomal protein synthesis
- 5- Decreased glucose entry into the cell
- answer
- back
- Question 02.37
- Answer = 1
- back to this question
- next question
- Reference(s)
- Buckwalter JA, Einhorn TA, Simon SR eds Orthopaedic Basic Science: Biology and Biornechanicsof the Musculoskeletal System, ed 2. Rosemont, IL, American Academy of Orthopaedic Surgeons,2000, pp 683-716.
- Simon SR ed Orthopaedic Basic Science. Rosemont, IL, American Academy of Orthopaedic Surgeons, 1994, pp 89-125.
- 02.38
- A 51-year-old woman who underwent humcral head replacement for a four-part fracture-dislocation of the proximal humerus 2 weeks ago now reports increased pain and swelling. She has been afebrile, and there has been slight drainage. Examination reveals swelling and erythema of the anterior shoulder. What is the next most appropriate step in management?
- 1- IV antibiotics
- 2- Oral antibiotics
- 3- Wound aspiration
- 4- Arthroscopic irrigation and debridement
- 5- Irrigation and debridement
- answer
- back
- Question 02.38
- Answer = 5
- back to this question
- next question
- Reference(s)
- Codd TP Yamaguchi K, Pollock RG et al: Infected shoulder arthroplasties: Treatment with staged reimplantation versus resection arthroplasty. J Shoulder Elbow Surg 1996;5S:5.
- Cofield RH Chang W, Sperling JW Complications of shoulder arthroplasty, in lannotti JP, Williams
- GR. Jr eds Disorders of the Shoulder: Diagnosis and Management. Philadelphia, PA, LippincottWilliams & Wilkins, 1999, pp 571-593.
- Sperling JW Kozak TK Hanssen AD, Cofield RH Infection after shoulder arthroplasty. Clin Orthop 2001;382:206-216.
- 02. 39.
- A 5O-year-old patient undergoes a core biopsy for a proximal tibial lesion. Biopsy results are consistent with a giant cell tumor of bone. The next most appropriate step in management should consist of
- 1- radiation therapy.
- 2- a repeat radiograph in 3 months.
- 3- administration of bisphosphonate.
- 4- curettage with local adjuvant.
- 5- extra-articular knee resection.
- answer
- back
- Question 02.39
- Answer = 4
- Reference(s)
- Gitelis S, McDonald DS: Surgery for Bone and Soft Tissue Tumors, in Simon MA, Springfield D eds Surgery for Bone and Soft Tissue Tumors. Philadelphia, PA, JB Lippincott, 1998, pp 181-207.
- 02.40
- A 22-year-old football player who falls on his flexed knee has posterior knee pain and
- swelling. Radiographs reveal no fractures or avulsions. Examination reveals a negative
- Lachman test result, a negative anterior drawer test, 2+ posterior drawer test, and no
- varus or valgus laxity at 0° or 30°. Appropriate management should consist of
- 1- rehabilitation with emphasis on quadriceps strengthening
- 2- rehabilitation with emphasis on hamstrings strengthening.
- 3- rehabilitation with emphasis on anterior cruciate ligament strengthening.
- 4- direct repair of the posterior cruciate ligament.
- 5- reconstruction of the posterior cruciate ligament.
- Question 02.40
- Answer = 1
- back to this question
- next question
- Reference(s)
- Cosgarea AJ, Jay PR: Posterior cruciate ligament injuries: Evaluation and management. J Am Acad Orthop Surg 2001;9:297-307.
- Covey DC: Injuries of the posterolateral corner of the knee. J Bone Joint Surg Am 2001;83:106-118.
- 02.41
- Figures 6a through 6c show the plain radiograph, MRI scan, and bone scan of an 80-year-old woman. These findings are most consistent with a diagnosis of
- 1- metastatic carcinoma.
- 2- compression fracture.
- 3- osteomyelitis.
- 4- tuberculosis.
- 5- Paget's disease.
- answer
- back
- Figures 6a Figures 6b Figures 6c
- Question 02.41
- Answer = 2
- back to this question
- next question
- Reference(s)
- Vaccaro AR, Shah SH, Schweitzer ME, Rosenfeld JF, Coder JM: MRI description of vertebral osteomyelitis, neoplasm, and compression fracture. Orthopedics 1999;22:67-73.
- 02.42
- A 13-year-old boy has peripatellar knee pain with tenderness reproduced by palpation at
- the proximal lateral corner of the patella. He denies any history of trauma. Activity
- restrictions, quadriceps strengthening exercises, and oral nonsteroidal anti-inflammatory
- drugs have failed to provide relief. A radiograph is shown in Figure 7. Treatment should
- now consist of
- 1- lateral retinacular release.
- 2- distal patellar tendon realignment.
- 3- chondroplasty.
- 4- advancement of the vastus medialis.
- 5- patellectomy.
- answer
- back
- Question 02.42
- Answer = 1
- Reference(s)
- Busch MT: Sports medicine in children and adolescents, in Morrissy RT, Weinstein SL (eds): Lovelland Winter's Pediatric Orthopaedics, ed 5. Philadelphia, PA, Lippincott Williams & WiWns, 2001, pp 1273-1318.
- Herring JA: Disorders of the knee, in Herring JA (ed): Tachdjian's Pediatric Orthopaedics, ed 3. Philadelphia, PA, WB Saunders, 2002, pp 789-838.
- Mori Y, Okumo H, Iketani H, Kuroki Y: Efficacy of lateral retinacular release for painful bipartite patella. Am J Sports Med 1995;23:13-18.
- Ogata K: Painful bipartite patella: A new approach to operative treatment. J Bone Joint Surg Am 1994;76:573-578.
- 02.43
- A 35-year-old man sustains a minimally displaced fracture of the distal radius that is
- treated in a long arm cast. Eight weeks after injury, he is unable to extend the
- interphalangeal joint of his thumb. Treatment should now consist of
- 1- decompression of the radial nerve in the forearm.
- 2- decompression of the radial nerve in the spiral groove.
- 3- tenolysis of the flexor pollicis longus tendon.
- 4- A1 pulley release in the thumb.
- 5- extensor indicis proprius to extensor pollicis longus tendon transfer.
- Question 02.43
- Answer = 5
- Reference(s)
- Hove LM: Delayed rupture of the thumb extensor tendon: A 5-year study of 18 consecutive cases. Acta Orthop Scand 1994;65:199-203.
- Magnussen PA, Harvey FJ, Tonkin MA: Extensor indicis proprius transfer for rupture of the extensor pollicis longus tendon. J Bone Joint Surg Br 1990;72:881-883.
- 02.44
- Warfarin exerts its effect on vitamin K in which of the following structures?
- 1- Small intestine
- 2- Liver
- 3- Vessel endothelium
- 4- Proximal tubules of the kidney
- 5- Distal tubules of the kidney
- answer
- back
- Question 02.44
- Answer = 2
- back to this question
- next question
- Reference(s)
- Buckwalter JA, Einhorn TA, Simon SR (eds): Orthopaedic Basic Science: Biology and Biomechanics of the Musculoskeletal System, ed 2. Rosemont, IL, American Academy of Orthopaedic Surgeons, 2000, pp 217-237 .
- 02.45
- A patient with an isolated femur fracture has an injury severity score (ISS) of 9. A
- patient with bilateral femur fractures and no other injuries has an ISS of
- 1- 9.
- 2- 16.
- 3- 18.
- 4- 25.
- 5- 81.
- answer
- back
- Question 02.45
- Answer = 1
- back to this question
- next question
- Reference(s)
- Koval KJ (ed): Orthopaedic Knowledge Update 7. Rosemont, IL, American Academy of Orthopaedic Surgeons, 2002, pp 111-120.
- Copeland CE, Mitchell KA, Brumback RJ, Gens DR, Burgess AR: Mortality in patients with bilateral femoral fractures. J Orthop Trauma 1998;12:315-319.
- 02.46
- A 23-year-old distance runner has had posterior plantar heel pain for the past 4 weeks.
- Radiographs reveal an area of sclerosis perpendicular to the trabecular stress lines in the
- posterior superior aspect of the calcareus Based on these findings, what is the bet
- course of action?
- I- Biopsy of the affected lesion
- 2- MRI of the foot and ankle
- 3- Restrict the patient to limited running
- 4- Ultrasound treatment
- 5- Use of nonsteroidal anti-inflammatory drugs
- answer
- back
- Question 02.46
- Answer = 3
- back to this question
- next question
- Reference(s)
- Boden BP, Osbahr DC, Jimenez C: Low-risk stress fractures. Am J Sports Med 2001;29:100-111. Leabhart JW: Stress fracture of the calcaneus. J Bone Joint Surg Am 1959;41:1285-1290.
- 02.47
- Which of the following statements best characterizes patients with back pain secondary to ankylosing spondylitis?
- 1- The pain is associated with morning stiffness.
- 2- The pain worsens with exercise.
- 3- The pain develops suddenly.
- 4- The patients typically are elderly men.
- 5- Sleep disturbances are rare.
- Question 02.47
- Answer = 1
- Reference(s)
- Clauw DJ, Petzke F: Rheumatic disorders, in Wiesel SW, Delahay JN (eds): Principals of Orthopaedic Medicine and Surgery. Philadelphia, PA, WB Saunders, 2001, pp 231-263.
- Booth RE, Simpson JM, Herkowitz HN: Arthritis of the spine, in Herkowitz HN, Garfm SR, Balderston
- RA, Eismont FJ, Bell GR, Wiesel SW (eds): The Spine, ed 4. Philadelphia, PA, WB Saunders, 1999, pp 429-453.
- 02.48
- An African-American 45-year-old woman with chronic back pain underwent an L4-5
- laminectomy and fusion but still reports significant pain despite taking low-dose codeine daily. Her prescription is changed to 25 mg of amitriptyline to be taken at bedtime and increased to 50 mg after 5 days. The patient reports for follow-up in 1 month and states that she has not been taking it every night because she could not tolerate the side effects. What is the next most appropriate step in management?
- 1- Explain the importance of adhering to the prescribed treatment for it to be beneficial.
- 2- Recruit family members to ensure that the patient follows the prescribed treatment.
- 3- Explore side effects and consider lowering the dose.
- 4- Check blood levels before making a decision about the dose.
- 5- Refer the patient to a spine surgeon.
- answer
- back
- Question 02.48
- Answer = 3
- back to this question
- next question
- Reference(s)
- Balant LP, Balant-Gorgia EA: Cultural differences: Implications on drug therapy and global drugdevelopment. Int J Clin Pharmacol Ther 2000;38:47-52.
- Jacobsen F: Psychopharmacology in Comas-Diaz L, Green B eds Women of Color. New York, NY, Guilford Press, 1994, pp 319-338.
- Silver B, Poland RE, Lin K-M: Ethnicity and the pharmacology of tricyclic antidepressants, in Lin KM,
- Poland RE, Nakasaki G eds Psychopharrnacology and psychobiology of ethnicity. Washington, DC, American Press, 1993, pp 61-89.
- 02. 49.
- A 14-year-old boy undergoes tibial lengthening with a thin pin circular fixator. Six
- weeks after the corticotomy and frame application, the patient notes pain while
- distracting the frame. Examination reveals that the incisions are healed and the pins have no drainage. Radiographs show 2.5 cm of regenerate bone; the wires appear bowed.
- What is the most likely source of the pain?
- 1- Premature consolidation
- 2- Pin tract infection
- 3- Poor regenerate bone formation
- 4- Wire loosening
- 5- Psychogenic
- answer
- back
- Question 02.49
- Answer = 1
- back to this question
- next question
- Reference(s)
- Murray JH, Fitch RD: Distraction histiogenesis: Principles and indications. J Am Acad Orthop Surg 1996;4:317-327.
- Lane CE, Herzenberg JE DiPietro MA: Radiographic imaging for Ilizarov limb lengthening in children. Pediatr Radiol 1991;21:117-120.
- 02. 50.
- What is the effect of cigarette smoking on the risk of pseudarthrosis following lumbar fusion?
- 1- Markedly increased (up to 500%)
- 2- Markedly increased but reversible if the patient stops smoking at the time of surgery
- 3- Slightly increased
- 4- Decreased
- 5- No effect
- answer
- back
- Question 02.50
- Answer = 1
- back to this question
- next question
- Reference(s)
- Porter SE, Hanley EN Jr: The musculoskeletal effects of smoking. J Am Acad Orthop Surg 2001;9:9-17.
- 02.51
- A 26-year-old man who was injured in a motorcycle accident reports pelvic pain.
- Examination reveals that he is hemodynamically stable and neurologically intact and has no associated injuries. What is the most appropriate definitive treatment for the injury shown in Figures 8a through 8d?
- 1- Anterior external fixation
- 2- Pelvic antishock c-clamp
- 3- Open reduction and internal fixation
- of the symphysis
- 4- Open reduction and internal fixation of the
- symphysis and right iliosacral screw
- fixation
- 5- Open reduction and internal fixation of the
- symphysis and bilateral iliosacral screw
- fixation
- Figures 8
- A
- C
- B
- D
- Question 02.51
- Answer = 3
- Reference(s)
- Matta JM Tornetta P III: Internal fixation of unstable pelvic ring injuries. Clin Orthop1996;329:129-140.
- Tornetta P 111, Dickson K, Matta JM Outcome of rotationally unstable pelvic ring injuries treated operatively. Clin Orthop 1996;329:147-151.
- 02. 52.
- What term is applied to the process by which a cell synthesizes and releases a molecule
- (such as growth factor) that binds to a receptor on a nearby cell within the same tissue?
- 1- Paracrine
- 2- Apoptosis
- 3- Autocrine
- 4- Autocinesis
- 5- Phagocytosis
- answer
- back
- Question 02.52
- Answer = 1
- back to this question
- next question
- Reference(s)
- Buckwalter JA, Einhom TA, Simon SR eds Orthopaedic Basic Science: Biology and Biomechanics of the Musculoskeletal System, ed 2. Rosemont, IL, American Academy of Orthopaedic Surgeons,2000, pp 829-844.
- Einhorn TA, Lee CA: Bone regeneration: New findings and potential clinical applications. J Am Acad Orthop Surg 2001;9:157-165.
- 02. 53.
- A 60-year-old man 1110 ruptured the proximal tendon of the long head of the biceps
- while lifting a heavy box 2 weeks ago now reports shoulder pain. Examination reveals
- full active shoulder motion and normal rotator cuff strength. Management should consist of
- 1- physical therapy.
- 2- a corticosteroid injection.
- 3- arthroscopic debridement.
- 1- biceps tendon repair.
- 5- biceps tenodesis.
- answer
- back
- Question 02.53
- Answer = 1
- back to this question
- next question
- Reference(s)
- Burkhead WZ Jr: The Biceps Tendon, in Rockwood CA Jr, Matsen FA III eds The Shoulder.Philadelphia, PA, WB Saunders, 1990, pp 791-836.
- Carroll RE, Hamilton LR Rupture of biceps brachii: A conservative method of treatment. J Bone Joint Surg Am 1967;49:1016.
- 02. 54.
- A 15-year-old boy has evidence of a solid fusion mass following posterior spinal instrumentation for idiopathic scoliosis. The patient was asymptomatic until 13 months postoperatively, at which time he noted mild back pain with a 1-week history of an enlarging fluctuant mass in the caudal midline scar. He has no fever or headache and is neurologically intact. Aspiration of the mass reveals purulent fluid with a negative Gram stain. Management should consist of administration of antibiotics and
- 1- serial aspirations of the fluctuant mass.
- 2- injection of corticosteroids into the fluctuant mass.
- 3- removal of the caudal portion of the spinal instrumentation.
- 4- incision and debridement with retention of spinal hardware.
- 5- incision and debridement with removal of all hardware.
- answer
- back
- Question 02.54
- Answer = 5
- back to this question
- next question
- Reference(s)
- Richards BS: Delayed infections following posterior spinal instrumentation for the treatment of idiopathic scoliosis. J Bone Joint Surg Am 1995;77:524-529.
- Dubousset J, Shufflebarger H, Wenger D: Late "infection" with CD instrumentation. Orthop Trans 1994;18:121.
- 02. 55.
- A 16-year-old wrestler has had knee pain after abrading his knee in practice 2 weeks ago. Examination reveals a healing 2-mm eschar, marked anterior knee tenderness and mild swelling, a trace joint effusion, and active knee motion from 5° to 120° of flexion. The ligament examination is normal. The next step in management should consist of
- I- physical therapy.
- 2- aspiration of the knee joint.
- 3- aspiration of the prepatellar bursa.
- 4- stress radiographs.
- 5- MRI
- Question 02.55
- Answer = 3
- Reference(s)
- Mysnyk MC, Snook GA: Wrestling, in Fu F, Stone K eds Sports Injuries: Mechanisms, Injury, Prevention. Baltimore, MD, Williams & Wilkins, 1994, pp 715-729.
- Wroble RR, Mysnyk MC, Foster DT, Albright JP: Patterns of knee injuries in wrestling: A six year study. Am J Sports Med 1986;14:55-66.
- 02. 56.
- A 30-year-old man sustains the injury shown in Figure 9 following a snowmobile accident. The injury is associated with a 10-cm open wound that was managed with local irrigation and functional bracing at a rural hospital. Examination at 1-week follow-up reveals a benign, well-healed wound but no active wrist or finger extension. What is the next most appropriate step in management?
- 1- Electrodiagnostic testing of radial nerve function
- 2- Continued fracture bracing and observation
- 3- Exploration of the radial nerve and plate fixation
- 4- Intramedullary nailing of the fracture
- 5- External fixation of the fracture
- answer
- back
- Figures 9
- Question 02.56
- Answer = 3
- back to this question
- next question
- Reference(s)
- Bostman O, Bakalim G, Vainionpaa S, Wilppula E, Patiala H, Rokkanen P: Immediate radial
- nerve palsy complicating fracture of the shaft of the humerus: When is early exploration justified? Injury 1985;16:499-502.
- Foster RJ Swiontkowski MF Bach AW Sack JT Radial nerve palsy caused by open humeral shaft fractures. J Hand Surg Am 1993;18:121-124.
- 02. 57.
- A 7-year-old boy who fell on his outstretched hand sustained the closed fracture shown in Figures 10a and 10b Management consists of application of along arm cast. There is no change in position of the bone following cast application. Based on these findings, the parents should be advised that the most likely outcome will be
- 1- healing with resumption of normal growth.
- 2- partial premature growth arrest.
- 3- complete growth arrest.
- 4- delayed union.
- 5- osteonecrosis of the distal radial epiphysis.
- answer
- back
- A
- A
- B
- Question 02.57
- Answer = 1
- back to this question
- next question
- Reference(s)
- Peterson HA: Physeal fractures: Part 2. Two previously unclassified types. J Pediatr Orthop1994;14:431-438.
- Peterson HA: Physeal fractures: Part 3. Classification. J Pediatr Orthop 1994;14:439-448.
- 02.58.
- In distinguishing patients with vascular claudication from those with neurogenic
- claudication (spinal stenosis), patients with the latter condition are most likely able to
- 1- walk downhill better than uphill.
- 2- stand for extended periods.
- 3- shop without a grocery cart.
- 4- predict their walking distance.
- 5- ride a stationary bicycle.
- answer
- back
- Question 02.58
- Answer = 5
- back to this question
- next question
- Reference(s)
- Rothman RH Simeone FA eds The Spine, ed 3. Philadelphia, PA, WB Saunders, 1992, pp 791-857.
- 02. 59.
- Which of the following factors has the greatest influence on achieving a good outcome
- following surgical treatment of tibial plateau fractures?
- I- Obtaining anatomic reduction of the articular surface
- 2- Restoring mechanical alignment and stability
- 3- Using minimally invasive stabilization techniques
- 4- Initiating early knee range of motion
- 5- Initiating early weight bearing
- Question 02.59
- Answer = 2
- Reference(s)
- Lansinger O, Bergman B, Korner L, Andersson GB: Tibial condylar fractures: A twenty-year follow-up. J Bone Joint Surg Am 1986;68:13-19.
- Honkonen SE: Indications for surgical treatment of tibial condyle fractures. Clin Orthop 1994;302: 199-205.
- 02. 60.
- When measuring for scoliosis, the correlation between the angle of trunk rotation and the radiographic Cobb angle is considered
- 1- zero.
- 2- logarithmic.
- 3- linear.
- 4- biphasic.
- 5- dependent on age.
- Question 02.60
- Answer = 3
- Reference(s)
- Bunnell WP: An objective criterion for scoliosis screening. J Bone Joint Surg Am 1984;66:1381-1387.
- Newton P, Wenger D: Idiopathic and congenital scoliosis, in Morrissy RT, Weinstein SL eds Lovell and Winter's Pediatric Orthopaedics, ed 5. Philadelphia, PA, Lippincott Williams & Wilkins, 2001, pp 677-740.
- 02. 61.
- An otherwise healthy 65-year-old woman has incapacitating hip pain for which
- nonsurgical management has failed to provide relief. A radiograph is shown in Figure
- 11. The next most appropriate step in treatment should consist of
- I- bipolar hip arthroplasty.
- 2- total hip arthroplasty.
- 3- core decompression.
- 4- fibula strut grafting.
- 5- intertrochanteric flexion osteotomy.
- Figures 11
- Question 02.61
- Answer = 2
- Reference(s)
- Taylor AH Shannon M, Whitehouse SL, Lee MB, Learmonth ID: Harris Galante cementless acetabular replacement in avascular necrosis. J Bone Joint Surg Br 2001;83:177-182.
- Hartley WT, McAuley JP, Culpepper WJ Engh CA Jr, Engh CA Sr: Osteonecrosis of the femoral head treated with cementless total hip arthroplasty. J Bone Joint Surg Am 2000;82:1408-1413.
- 02. 62.
- What lesion typically found in an infant is believed to be a precursor to adult
- adamantinoma?
- l- Osteofibrous dysplasia
- 2- Desmoplastic fibroma
- 3- Nonossifying fibroma
- 4- Congenital pseudarthrosis of the tibia
- 5- Multiple enchondromatosis (Ollier's disease)
- Question 02.62
- Answer = 1
- Reference(s)
- Springfield DS, Rosenberg AE, Mankin HJ Mindell ER: Relationship between osteofibrous dysplasia and adamantinoma. Clin Orthop 1994;309:234-244.
- 02. 63.
- A 14-year-old football playcr sustains a midsubstance tear of the anterior cruciate
- ligament. An MRI scan shows no evidence of meniscal injury. Nonsurgical management of this injury will most likely result in
- 1- recurrent instability with a probable meniscal tear.
- 2- a limb-length discrepancy.
- 3- a stable knee at skeletal maturity.
- 4- early degenerative joint changes.
- 5- varus deformity.
- Question 02.63
- Answer = 1
- Reference(s)
- Angel KR, Hall DJ: Anterior cruciate ligament injury in children and adolescents. Arthroscopy1989;5:197-200.
- Janarv PM, Nystrom A, Werner S, Hirsch G: Anterior cruciate ligament injuries in skeletally immature patients. J Pediatr Orthop 1996;16:673-677.
- Pressman AE, Letts RM, Jarvis JG Anterior cruciate ligament tears in children: An analysis of operative versus nonoperative treatment. J Pediatr Orthop 1997;17:505-511.
- 02. 64.
- Which of the following findings best indicates that a slipped capital femoral epiphysis
- can be considered stable?
- 1- Ultrasound of the hip reveals an effusion but no metaphyseal remodeling.
- 2- A cross-table lateral radiograph of the hip reveals a 30% slippage.
- 3- The patient is able to walk with or without crutches.
- 4- The patient is unable to walk with crutches but is comfortable in traction.
- 5- An AP radiograph of the hip reveals widening of the proximal physis.
- Question 02.64
- Answer = 3
- Reference(s)
- Kallio P, Paterson DC, Foster BK Lequesne GW Classification in slipped capital femoral epiphysis: Sonographic assessment of stability and remodeling. Clin Orthop 1993;294:196-203.
- Kallio PE, Mah ET, Foster BK Paterson DC, LeQuesne GW Slipped capital femoral epiphysis: Incidence and clinical assessment of physeal instability. J Bone Joint Surg Br 1995;77:752-755.
- Loder R T, Aronsson DD, Dobbs MB: Slipped capital femoral epiphysis. J Bone Joint Surg Am 2000;82:1170-1188.
- 02. 65.
- After compression plating of a diaphyseal fracture, gapping is noted on the surface of the bone opposite the plate. The lamellar bone formed within this gap would be expected to be oriented in what relationship to the axis of the bone?
- 1- 15
- 2- 45°
- 3- 60°
- 4- Parallel
- 5- Perpendicular
- Question 02.65
- Answer = 5
- Reference(s)
- Buckwalter JA, Einhorn TA, Simon SR eds Orthopaedic Basic Science: Biology and Biomechanics of the Musculoskeletal System, ed 2. Rosemont, IL, American Academy of Orthopaedic Surgeons, 2000, pp 372-399.
- Browner BD, Jupiter JB Levine AM, Trafton PG eds Skeletal Trauma, ed 2. Philadelphia, PA, WB Saunders, 1998, pp 59-68.
- 02. 66.
- Loss of periacetabular Cilium) bone mineral density in the first 2 years following total hip arthroplasty in which a press-fit acetabular component was used is most likely
- attributable to
- 1- altered acetabular stress patterns.
- 2- polyethylene particulate osteolysis.
- 3- bone necrosis following acetabular reaming.
- 4- metal ion-induced osteolytic cell activity.
- 5- age-related (type I) osteoporosis.
- Question 02.66
- Answer = 1
- Reference(s)
- Wright JM Pellicci PM, Salvati EA, Ghelman B, Roberts MM, Koh JL Bone density adjacent topress-fit acetabular components: A prospective analysis with quantitative computed tomography. J Bone Joint Surg Am 2001;83:529 536.
- 02. 67.
- A healthy, active, cognitively intact 70-year-old woman sustains an unstable
- intertrochanteric fracture that is stabilized with a well-placed sliding hip screw. Allowing immediate unrestricted weight bearing will result in
- 1- a decreased time to functional return.
- 2- an increased risk of postoperative hip dislocation.
- 3- an increased risk of sideplate breakage.
- 4- an increased risk of implant cutout.
- 5- an increased risk of wound healing problems.
- Question 02.67
- Answer = 1
- Reference(s)
- Kellam JF, Fischer TJ, Tornetta P 111, Bosse MJ Harris MB eds Orthopaedic Knowledge Update:Trauma 2. Rosemont, IL, American Academy of Orthopaedic Surgeons, 2000, pp 125-131.
- Koval KJ, Aharonoff GB, Su ET, Zuckerman JD: Effect of acute inpatient rehabilitation on outcome after fracture of the femoral neck or intertrochanteric structure. J Bone Joint Surg Am 1998;80:357-364.
- 02. 68.
- The most appropriate starting point when placing a pedicle screw at the level of L4 is at
- the junction of the
- 1- pars interarticularis of L4 and the tip of the inferior articular facet of L3.
- 2- pars interarticularis of L4 and the inferior articular facet of L4.
- 3- pars interarticularis of L4 and the lamina of L4.
- 4- inferior articular facet of L3 and the superior articular facet of L4.
- 5- transverse process of L4, the lateral aspect of the superior articular facet of L4, and
- the pars interarticularis of L4.
- Question 02.68
- Answer = 5
- Reference(s)
- Lonstein JE Denis F, Perra JH, Pinto MR, Smith MD, Winter RB Complications associated withpedicle screws. J Bone Joint Surg Am 1999;81:1519-1528.
- Levine AM: Surgical techniques for the treatment of thoracic, thoracolumbar, lumbar, and sacral trauma, in Herkowit7 HN, Garfin SR, Balderston RA, Eismont FJ, Bell GR Wiesel SW eds The Spine, ed 4. Philadelphia, PA, WB Saunders, 1999, pp 1003-1070.
- 02. 69.
- With regard to a patient's self-assessment of shoulder function, those patients covered by workers' compensation insurance, when compared with other patients, tend to have
- 1- worse self-assessed function and health status.
- 2- better self-assessed function but worse self-assessed health status.
- 3- better self-assessed health status but worse self-assessed function.
- 4- better self-assessed function and health status.
- 5- no difference in self-assessed function and health status.
- Question 02.69
- Answer = 1
- Reference(s)
- Viola RW Boatright KC Smith KL, Sidles JA, Matsen FA III: Do shoulder patients insured by workers' compensation present with worse self-assessed function and health status? J Shoulder Elbow Surg2000:9:368-372
- Matsen FA III, Ziegler DW DeBartolo SE: Patient self-assessment of health status and function in glenohumeral degenerative joint disease. J Shoulder Elbow Surg 1995;4:345-351.
- 02. 70.
- A 4-year-old girl undergoes repair of both flexor tendons in zone II. Initial postoperative physical therapy should consist of
- 1- passive mobilization only.
- 2- active extension and passive flexion mobilization.
- 3- active place and hold mobilization.
- 4- immobilization.
- 5- unrestricted active mobilization.
- Question 02.70
- Answer = 4
- Reference(s)
- Stewart KM: Review and comparison of current trends in the postoperative management of tendon repair. Hand Clin 1991;7:447-460.
- Wakefield AR: Hand injuries in children. J Bone Joint Surg Am 1964;46:1226.
- 02. 71.
- A 60-year-old woman has a painful bunion. Her foot examination is normal except for
- the hallux valgus deformity. A standing AP radiograph is shown in Figure 12. Because
- the patient would like definitive correction of the deformity, the best course of action
- should be
- 1- resection arthroplasty (Keller arthroplasty).
- 2- arthrodesis of the metatarsophalangeal joint.
- 3- distal soft-tissue realignment procedure and a
- proximal metatarsal osteotomy.
- 4- distal metatarsal osteotomy (chevron procedure).
- 5- distal metatarsal osteotomy (chevron procedure)
- and a proximal phalangeal osteotomy (Akin).
- Question 02.71
- Answer = 3
- Reference(s)
- Mann RA, Coughlin MJ Adult hallux valgus in Coughlin MJ Mann RA eds Surgery of the Foot and Ankle, ed 7. St Louis, MO, Mosby Publishers, 1999, pp 150-269.
- Coughlin MJ Hallux valgus J Bone Joint Surg Am 1996;78:932-966
- 02. 72.
- A 75-year-old man undergoes humeral head replacement for rotator cuff tear arthropathy. A postoperative radiograph is shown in Figure 13. Which of the following factors most likely accounts for the position of the humeral head?
- 1- The head is too large.
- 2- The humeral prosthesis is seated too proud in the humeral canal.
- 3- The component is loose within the humeral canal.
- 4- There is incompetence of the coracoacromial arch.
- 5- There is incompetence of the subscapularis.
- Figure 13
- Question 02.72
- Answer = 4
- Reference(s)
- Flatow EL, Weinstein DM, Duralele XA Coracoacromial ligament preservation in rotator cuff surgery. J Shoulder Elbow Surg 1994;3:S73.
- Field LD Dines DM, Zabinski SJ Warren RF Herniarthroplasty of the shoulder for rotator cuff arthropathy. J Shoulder Elbow Surg 1997;6:18-23.
- Zeman CA, Arcand MA, Cantrell JS, Skedros JG Burkhead WZ Jr: The rotator cuff-deficient arthritic shoulder: Diagnosis and surgical management. J Am Acad Orthop Surg 1998;6:337-348.
- 02.73.
- Figures 14a and 14b show the radiographs of a 7-year-old boy who has a displaced sleeve fracture of the patella. Examination reveals a palpable step-off transversely in the
- midpatellar region. The patient cannot extend the knee actively. Based on these
- findings, management should consist of
- 1- a compression dressing.
- 2- a long leg cast.
- 3- closed reduction and a long leg cast.
- 4- open reduction and tension band wiring.
- 5- physical therapy.
- Figures 14a
- Figures 14b
- Question 02.73
- Answer = 4
- Reference(s)
- Maguire JK Canale ST: Fractures of the patella in children and adolescents. J Pediatr Orthop 1993;13:567-571.
- Price C, Phillips J, Devito D: Management of fractures, in Morrissy RT., Weinstein SL eds Lovell and Winter's Pediatric Orthopaedics, ed 5. Philadelphia, PA, Lippincott Williams & Wilkins, 2001, pp 1319-1422.
- 02. 74.
- Elderly patients undergoing transtibial amputation regain their preoperative functional
- status in what percentage of cases?
- 1- Up to 10%
- 2- 11% to 30%
- 3- 31 % to 50%
- 4- 51 % to 70%
- 5- 71% to 90%a
- Question 02.74
- Answer = 3
- Reference(s)
- Pandian G, Kowalske K: Daily functioning of patients with an amputated lower extremity. Clin Ortho f1999;361:91-97.
- Frykberg RG Arora S, Pomposelli FB Jr, LoGerfo F: Functional outcome in the elderly following lower extremity amputation. J Foot Ankle Surg 1998;37:181-185
- 02. 75.
- In an obtunded patient with a tibial fracture and suspected compartment syndrome,
- fasciotomy should be performed when intracompartment pressures are
- I- greater than 30 mm Hg.
- 2- greater than 40 mm Hg.
- 3- within 40 mm Hg of the systolic blood pressure.
- 4- within 30 mm Hg of the diastolic blood pressure.
- 5- within 30 mm Hg of the mean arterial blood pressure.
- Question 02.75
- Answer = 4
- Reference(s)
- McQueen MM, Court-Brown CM: Compartment monitoring in tibial fractures: The pressure threshold for decompression. J Bone Joint Surg Br 1996;78:99-104.
- Nassif JM Gorczyca JT Cole JK Pugh KP Pienkowski D: Effect of acute reamed versus unreamed intramedullary nailing on compartment pressure when treating closed tibial shaft fractures: A randomized prospective study. J Orthop Trauma 2000;14:554-558
- 02. 76.
- The medial cord of the brachial plexus receives contributions from
- 1- all roots of the plexus.
- 2- the fifth and sixth cervical roots.
- 3- the fifth, sixth, and seventh cervical roots.
- 4- the sixth, seventh, and eighth cervical roots.
- 5- the eighth cervical root and the first thoracic root.
- Question 02.76
- Answer = 5
- Reference(s)
- Tubiana R, McCullough CJ, Masquelet AC: An Atlas of Surgical Exposures of the Upper Extremity.Philadelphia, PA, JB Lippincott, 1990, pp 148-151.
- Tobaino MM: Nonobstetric brachial plexus injuries. J Am Soc Surg Hand 2001;1:135-153
- 02.77.
- A premenarchal 10-year-old girl who is Risser 0 has a progressive right thoracic
- idiopathic scoliosis. Continued anterior spinal growth following posterior spinal fusion
- may result in
- I- thoracic hyperkyphosis.
- 2- pelvic obliquity.
- 3- pectus carinatum.
- 4- pectus excavatum.
- 5- crankshaft phenomenon.
- Question 02.77
- Answer = 5
- Reference(s)
- Burton DC, Asher MA, Lai SM.: Scoliosis correction maintenance in skeletally immature patients witidiopathic scoliosis: Is anterior fusion really necessary? Spine 2000;25:61-68.
- Roberto RF Lonstein JE Winter RB Denis F: Curve progression in Risser stage 0 or 1 patients after posterior spinal fusion for idiopathic scollosis. J Pediatr Orthop 1997;17:718-725.
- 02. 78.
- What structure resists inferior translation of the shoulder in the adducted position?
- I - Superior labral complex
- 2- Superior glenohumeral ligament
- 3- Middle glenohumeral ligament
- 4- Inferior glenohumeral ligament
- 5- Coracoacromial ligament
- Question 02.78
- Answer = Item Deleted
- Reference(s)
- 02. 79.
- The common pathologic process that occurs in patients with rickets, regardless of the
- cause, is a failure to
- I- mineralize the matrix in the zone of provisional calcification.
- 2- adequately synthesize chondroitin sulfate.
- 3- adequately synthesize type 11 collagen.
- 4- metabolize and degrade glycoproteins.
- 5- resorb the primary spongiosum.
- Question 02.79
- Answer = 1
- Reference(s)
- Buckwalter JA, Einhorn TA, Simon SR eds Orthopaedic Basic Science: Biology and Biomechan of the Musculoskeletal System, ed 2. Rosemont, IL, American Academy of Orthopaedic Surgeons,2000, pp 78-109.
- Miller WL Portale AA: Genetic causes of rickets. Curr Opin Pediatr 1999;11:333-339.
- 02. 80.
- A 17-year-old boy has progressive numbness and tingling in his right small finger and the ulnar aspect of the ring finger. Examination reveals flattening of the hypothenar
- eminence and cubitus valgus. History reveals that he had a right elbow fracture at age 4
- years. The previous injury was most likely what type of fracture'?
- 1- Medial condyle
- 2- Radial neck
- 3- Olecranon
- 4- Lateral condyle
- 5- Lateral epicondyle
- Question 02.80
- Answer = 4
- Reference(s)
- Wilkins KE Beaty JH, Chambers HG, Toniolo RM.: Fractures and dislocations of the elbow regioin Rockwood CA Jr, Wilkins Ke Beaty JH eds Fractures in Children, ed 4. Philadelphia, PA,Lippincott-Raven, 1996, pp 653-904.
- Herring JA: Upper extremity injuries, in Herring JA ed Tachdjian's Pediatric Orthopaedics, ed 3. Philadelphia, PA, WB Saunders, 2002, pp 2115-2250.
- Ogden JA: Humerus, in Ogden JA ed Skeletal Injury in the Child, ed 2. New York, NY, Springer-Verlag, 2000, pp 456-541.
- Miller EM: Late ulna nerve palsy. Surg Gynecol Obstet 1924;38:37-46.
- 02. 81.
- The antithrombotic effect of aspirin is felt to be related to
- 1- factor X inhibition.
- 2- prothrombin inhibition.
- 3- blocking platelet aggregation.
- 4- vitamin K competition.
- 5- the binding of antithrombin 111.
- Question 02.81
- Answer = 3
- Reference(s)
- Buckwalter JA, Einhorn TA, Simon SR eds Orthopaedic Basic Science: Biology and Biomech of the Musculoskeletal System, ed 2. Rosemont, IL, American Academy of Orthopaedic Surge, 2000, pp 218-237.
- 02. 82.
- A 64-year-old patient with rheumatoid arthritis is scheduled for total hip arthroplasty.
- Which of the following medications should be temporarily discontinued in the immediate preoperative and early postoperative period?
- 1- Gold
- 2- Antimalarial agents
- 3- Methotrexate
- 4- Penicillamine
- 5- Sulfasalazine
- Question 02.82
- Answer = 3
- Reference(s)
- Weinblatt ME: Antirheumatic drug therapy and the surgical patient, in Sledge CB, Ruddy S, HarrisED Jr, Kelly WN (eds): Arthritis Surgery. Philadelphia, PA, WB Saunders, 1994, pp 669-673.
- Pellici PM, Tria AJ Jr, Garvin KL (eds): Orthopaedic Knowledge Update: Hip and Reconstruction 2. Rosemont, IL, American Academy of Orthopaedic Surgeons, 2000, pp 3-6
- 02. 83.
- Which of the following modalities is a sensory function served by tracts in the posterior
- column of the spinal cord?
- 1- Pain
- 2- Position
- 3- Lighttouch
- 4- Temperature
- 5- Pinprick
- Question 02.83
- Answer = 2
- Reference(s)
- Bohlman\ HH Eucker TB: Spine and spinal cord injuries, in Herkowitz HN Garfin SR, Balderston RA,
- Eismont FJ, Bell GR., Wiesel SW (eds): The Spine, ed 4. Philadelphia, PA, WB Saunders, 1999, pp 889-1002.
- McGuire RA: Physical examination in spinal trauma, in Levine AM, Eisemont FJ, Garfin SR, Ziegler JE (eds): Spinal Trauma. Philadelphia, PA, WB Saunders, 1998, pp 16-27.
- 02. 84.
- When performing a percutaneous trigger finger release of the thumb, what structure is most vulnerable to injury?
- I - Median motor nerve to the thenars
- 2- Thumb common digital nerve
- 3- Thumb radial digital nerve
- 4- Thumb ulnar digital nerve
- 5- Motor branch to the adductor pollicis mu5clc
- Question 02.84
- Answer = 3
- Reference(s)
- Pope DF, Wolfe SW: Safety and efficacy of percutaneous trigger finger release. J Hand surg Am 1995;20:280-283.
- Bain GI, Turnbull J, Charles MN, Roth JH, Richards RS: Percutaneous A1 pulley release: A cadaveric study. J Hand Surg Am 1995;20:781-784
- 02. 85.
- During an anterior approach to the sacroiliac joint, the surgeon inadvertently injures the
- 1.5 nerve root. Weakness will most likely be observed in the muscles innervated by
- which of the following nerves?
- 1- Femoral
- 2- Obturator
- 3- Pudendal
- 4- Superior gluteal
- 5- Genitofemoral
- Question 02.85
- Answer = 4
- Reference(s)
- Atlihan D, Tekdemir I, Ates Y, Elhan A: Anatomy of the anterior sacroiliac joint with reference to lumbosacral nerves. Clin Orthop 2000;376:236-241.
- Jenkins DB: Hollinshead's Functional Anatomy of the Limbs and Back, ed 6. Philadelphia, PA, WB Saunders, 1991, pp 256-259.
- 02. 86.
- A 32-year-old man returns for a 6-week follow-up evaluation after undergoing an open
- Bankart repair for recurrent shoulder dislocations. Examination reveals obvious
- weakness of the deltoid and infraspinatus muscles. Injury of what neurologic structure
- accounts for this finding?
- 1- Axillary nerve
- 2- Suprascapular nerve
- 3- Upper trunk of the brachial plexus
- 4- Lateral cord of the brachial plexus
- 5- Posterior cord of the brachial plexus
- Question 02.86
- Answer = 3
- Reference(s)
- Jobe C: Gross anatomy of the shoulder, in Matsen FA III, Rockwood CA Jr (eds): The Shoulder.Philadelphia, PA, WB Sunders 1990, pp 34-97.
- Lazarus MD, Guttmann D: Complications of instability surgery, in lannotti JP, Williams GR. Jr (eds): Disorders of the Shoulder: Diagnosis and Management. Philadelphia, PA, Lippincott Williams & Wilkins, 1999, pp 361-393.
- 02. 87.
- What region of the posterior tibial tendon has the poorest intrinsic blood supply
- (watershed area)?
- 1- At the tendon insertion sites on the plantar foot
- 2- At the tendon insertion site on the medial navicular
- 3- At the musculotendinousjunction
- 4- Between the navicular insertion site and the distal medial malleolus
- 5- Posterior to the medial malleolus within the retinacular sheath
- Question 02.87
- Answer = 4
- Reference(s)
- Frey C, Shereff M, Greenidge N: Vascularity of the posterior tibial tendon. J Bone Joint Surg Am1990;72:884-888.
- Lin SS, Lee TH., Chao W, Wapner KL: Nonoperative treatment of patients with posterior tibial tendontis in Wapner KL ed Foot and Ankle Clinics: Tendon Injury and Reconstruction. Philadelphia, PA, WB Saunders, 1996, pp 261-277.
- 02. 88.
- The largest percentage of children diagnosed as abused are in what age group?
- 1- Birth to 4 years
- 2- 4 years to 8 years
- 3- 8 years to 12 years
- 4- 12 years to 16 years
- 5- Older than 16 years
- Question 02.88
- Answer = 1
- Reference(s)
- Akbarnia BA, Campbell RM The role of the orthopaedic surgeon in child abuse, in Morrissy RT.,
- Weinstein SL (eds): Lovell and Winter's Pediatric Orthopaedics, ed 5. Philadelphia, PA, Lippincott Williams & Wilkins, 2001, pp 1423-1445.
- Herring JA: General principles of managing orthopaedic injuries, in Herring JA ed Tachdjian's Pediatric Orthopaedics, ed 3. Philadelphia, PA, WB Saunders, 2002, pp 2059-2086.
- Akbarnia B, Torg JS, Kirkpatrick J, Sussman S: Manifestations of the battered-child syndrome. J Bone Joint Surg Am 1974;56:1159-1166
- 02. 89.
- The stability of a circular external fixator can be increased by
- 1- increasing the ring size.
- 2- increasing the number of wires.
- 3- increasing the distance between the fracture site and the rings.
- 4- decreasing the wire diameter.
- 5- decreasing the wire tension.
- Question 02.89
- Answer = 2
- Reference(s)
- Buckwalter JA, Einhorn TA, Simon SR (eds): Orthopaedic Basic Science: Biology and Biomechanics of the Musculoskeletal System, ed 2. Rosemont, IL, American Academy of Orthopaedic Surgeons, 2000, pp 371-399.
- Kummer FJ: Biomechanics of the Ilizarov external fixator Clin Orthop 1992;280:11-14.
- 02. 90.
- A 4-year-old boy has had a limp for approximately 1 month. Examination reveals that
- the right hip has full flexion and adduction, but there is guarding in abduction and
- internal rotation. Radiographs and MRI scans of the right hip show osteonecrosis of the
- anterior superior portion of the femoral head with no collapse. Initial management
- should consist of
- 1- physical therapy.
- 2- abduction bracing.
- 3- bed rest in traction.
- 4- adductor tenotomy and long leg abduction casting.
- 5- incomplete intertrochanteric osteotomy
- Question 02.90
- Answer = 1
- Reference(s)
- Clancy M, Steel HH The effect of an incomplete intertrochanteric osteotomy on Legg-Calve- Perthes disease. J Bone Joint Surg Am 1985;67:213-216.
- Salter RB Thompson GH Legg-Calve-Perthes disease: The prognostic significance of the subchondral fracture and a two-group classification of the femoral head involvement. J Bone Joint Surg Am 1984;66:479-489.
- deSanctis N, Rega AN, Rondinella F: Prognostic evaluation of Legg-Calve-Perthes disease by MRI: Part I. The role of physeal involvement. J Pediatr Orthop 2000;20:455-462.
- deSanctis N, Rondinella F: Prognostic evaluation of Legg-Calve-Perthes disease by MRI: Part lI. Pathomorphogenesis and new classification. J Pediatr Orthop 2000;20:463-470.
- 02. 91.
- Bipolar hip arthroplasty maintains a useful role in revision arthroplasty for which of the
- following conditions?
- 1- Pelvic discontinuity
- 2- Segmental acetabular defects
- 3- Limb-length discrepancy
- 4- Cavitary aeetabular bone loss
- 5- Chronic instability
- Question 02.91
- Answer = 5
- Reference(s)
- Parvizi J, Morrey BF: Bipolar hip arthroplasty as a salvage treatment for instability of the hip. J Bone Joint Surg Am 2000;82:1132-1139
- 02. 92.
- When using pedicle screw fixation as a component of spinal instrumentation, at what
- level is bicortical fixation most appropriate?
- 1- T4
- 2- T12
- 3- L5
- 4- S 1
- 5- S2
- Question 02.92
- Answer = 4
- Reference(s)
- Gaines RW Jr: The use of pedicle-screw internal fixation for the operative treatment of spinal disorders. J Bone Joint Surg Am 2000;82:1458-1476.
- Vaccaro AR, Garfin SR: Pedicle screw fixation in the lumbar spine. J Am Acad Orthop Surg 1995;3:263-274
- 02. 93.
- A 25-year-old man has a flail left upper extremity after being involved in a high-speed
- motorcycle accident. Examination also reveals ptosis, miosis, and enophthalmos
- involving his left eye. This finding most likely indicates avulsion of what nerve roots?
- 1- C1 and C2
- 2- C3 and C4
- 3- C4 and C5
- 4- C5 and C6
- 5- C8 and T1
- Question 02.93
- Answer = 5
- Reference(s)
- Hentz VR Narakas A: The results of microneurosurgical reconstruction in complete brachial plexus palsy: Assessing outcome and predicting results. Orthop Clin North Am 1988;19:107-114.
- Narakas AO: Brachial plexus surgery. Orthop Clin North Am 1981;12:303-323.
- 02. 94.
- A 12-year-old girl has had localized back pain for the past 6 months. There is no
- associated febrile illness or constitutional symptoms. The MRI scan and radiographs
- shown in Figures 15a through 15c reveal coarse striations. Laboratory studies show that
- the CBC and erythrocyte sedimentation rate are within normal limits. What is the most
- likely diagnosis?
- 1- Nonossifying fibroma
- 2- Unicameral bone cyst
- 3- Osteoblastoma
- 4- Hemangioma of bone
- 5- Fibrous dysplasia
- Figures 15a
- Figures 15c
- Figures 15b
- Question 02.94
- Answer = 4
- Reference(s)
- Springfield D, Gebhart M: Bone and soft tissue tumors, in Morrissy RT., Weinstein SL eds Lovell and Winter's Pediatric Orthopaedics, ed 5. Philadelphia, PA, Lippincott Williams & Wilkins, 2001, pp 507-562
- 02. 95.
- A 35-year-old recreational tennis player reports pain in his left shoulder during all
- activities. History reveals that he underwent a procedure to stabilize his shoulder after
- several recurrent dislocations 10 years ago. He states that he has never been free from
- pain, but he has had no further episodes of instability. A current radiograph is shown in
- Figure 16. Management should consist of
- I- a classic Bankart repair.
- 2- a capsular shrinkage procedure.
- 3- a strengthening program for the rotator cuff and deltoid.
- 4- an inferior capsular shift.
- 5- removal of the screw.
- Question 02.95
- Answer = 5
- Reference(s)
- Zuckerman JD, Matsen FA III: Complications about the glenohumeral joint related to the use of screws and staples. J Bone Joint Surg Am 1984;66:175-180.
- Bigliani LU Weinstein DM, Glasgow MT, Pollack RG Flatlow EL: Glenohumeral arthroplasty for arthritis after instability surgery. J Shoulder Elbow Surg 1995;4:87-94
- 02. 96.
- What muscle action is defined by muscle activation with simultaneous muscle
- lengthening?
- I- Concentric
- 2- Eccentric
- 3- Isokinetic
- 4- Isotonic
- 5- Isometric
- Question 02.96
- Answer = 2
- Reference(s)
- Skeletal Muscle: Form and Function, in Human Kinetics. Champaign, IL, Alan J McComas Publisher: Human Kinetics, 1996, p 177.
- Kreighbaum E, Barthels K: Biomechanics: A Qualitative Approval for Studying Human Movement, ed 2. Minneapolis, MN, Burgess Publishing Company, 1985, p 317.
- 02. 97.
- A patient has had aching thigh pain for the past month. An AN radiograph is shown in
- Figure 17. What is the most likely source of pain''
- I- Malignant degeneration
- 2- Microfracture
- 3- Degenerative arthritis of the hip
- 4- Infection
- 5- Slipped capital femoral epiphysis
- Figures 17
- Question 02.97
- Answer = 2
- Reference(s)
- Guille JT Kumar SJ MacEwen GD Fibrous dysplasia of the proximal part of the femur: Long-term results of curettage and bone grafting and mechanical realignment. J Bone Joint Surg Am 1998;80:648-658
- 02. 98.
- The flexion lateral cervical spine radiograph .shown in Figure 18 suggests an injury, to
- which of the following ligaments?
- I- Anterior longitudinal
- 2- Posterior longitudinal
- 3- Ligamentum flavutn
- 4- Transverse atlantal
- 5- Infraspinous
- Figures 18
- Question 02.98
- Answer = 4
- Reference(s)
- Greene KA, Dickman CA, Marciano FF Drabier J, Drayer BP, Sonntag VK Transverse atlantal ligament disruption associated with odontoid fractures. Spine 1994;19:2307-2314
- 02. 99.
- A 35-year-old patient has left knee pain. The radiographs and CT scan are shown in
- Figures 19a through 19c. A biopsy specimen is shown in Figure 19d. What is the most
- likely diagnosis?
- I- Giant cell tumor
- 2- Giant cell rich osteosarcoma
- 3- Renal cell carcinoma
- 4- Chondroblastoma
- 5- Hyperparathyroidisin (Brown tumor)
- Figures 19c
- Figures 19d
- Figures 19b
- Figures 19a
- Question 02.99
- Answer = 1
- Reference(s)
- Gitelis S, McDonald DS: Surgery for Bone and Soft Tissue Tumors, in Simon MA, Springfield D eds Surgery for Bone and Soft Tissue Tumors. Philadelphia, PA, JB Lippincott, 1998, pp 181-207.
- 02. 100.
- During a total knee arthroplasty for an 18° valgus deformity, the femoral and tibial bone cuts are made and soft-tissue tension assessed. In full extension, the lateral side is tight, but the medial side is loose. In flexion, the medial and lateral soft tissues are well balanced. Which of the following structures should be released first?
- 1- Posterior cruciate ligament
- 2- Lateral collateral ligament
- 3- Iliotibial band
- 4- Lateral gastrocnemius tendon
- 5- Posterolateral capsule
- Question 02.100
- Answer = 3
- Reference(s)
- Whiteside LA: Selective ligament release in total knee arthroplasty of the knee in valgus. Clin Orthop 1999;367:130-140
- 02. 101.
- A 6-year-old girl has fourth toes that overlap the third toes bilaterally since birth. Her parents report difficulty in fitting shoes. Examination reveals redness and tenderness over the fourth toes. Management should consist of
- 1- observation.
- 2- taping the toes.
- 3- extensor release of the fourth toes.
- 4- flexor tenotomies of the third toes.
- 5- plantar flexion osteotomy of the fourth proximal phalanges.
- Question 02.101
- Answer = 4
- Reference(s)
- Turner PL: Strapping of curly toes in children. Aust N Z J Surg 1987;57:467-470.
- Ross ER, Menelaus MB: Open flexor tenotomy for hammer toes and curly toes in childhood. J Bone Joint Surg Br 1984;66:770-771
- 02. 102.
- In the evaluation of an unconscious polytrauma patient with suspected spinal injury, which of the following is considered the most appropriate radiographic evaluation of the spine?
- 1-Cervical spine series
- 2-Cervical spine series and MRI of the cervical spine
- 3-Cervical spine series and AP and lateral radiographs of the lumbar spine
- 4-Complete series of radiographs of the cervical, thoracic, and lumbar spine
- 5- MRI of the cervical, thoracic, and lumbar spine
- Question 02.102
- Answer = 4
- Reference(s)
- Kellam JF, Fischer TJ, Torrenta P III, Bosse MJ Harris MB eds Orthopaedic Knowledge Update: Trauma 2. Rosemont, IL, American Academy of Orthopaedic Surgeons, 2000, pp 319-328.
- Stanislas MJ Latham JM Porter KM, Alpar EK Stirling AJ: A high risk group for thoracolumbar fractures. Injury 1998;29:15-18
- 02.103.
- A 45-year-old tennis player sustains an anterior dislocation of the glenohumeral joint after falling on the tennis court. His shoulder is reduced within 1 hour, and radiographs show a concentric reduction with no associated fractures. Examination 2 weeks later reveals that the patient can actively elevate his arm to 60°. His passive forward flexion is 170°. What is the most likely cause for the lack of active shoulder elevation?
- 1- Glenohumeral ligament injury
- 2- Axillary nerve palsy
- 3- Adhesive capsulitis
- 4- Brachial plexus injury
- 5- Rotator cuff tear
- Question 02.103
- Answer = 5
- Reference(s)
- Hawkins RJ, Bell RH, Hawkins RH, Koppert GJ Anterior dislocation of the shoulder in the older patient. Clin Orthop 1986;206:192-195.
- Neviaser RJ, Neviaser TJ, Neviaser JS: Concurrent rupture of the rotator cuff and anterior dislocation of the shoulder in the older patient. J Bone Joint Surg Am 1988;70:1308-1311
- 02. 104.
- A 60-year-old woman who has had pain and numbness in her thumb, index, and long fingers underwent an uneventful open carpal tunnel release. Postoperatively, she reports complete resolution of her symptoms but now notes progressive thenar atrophy that was not present prior to surgery. What is the most likely cause of this finding?
- 1- Incomplete release of the transverse carpal ligament
- 2- Cervical radiculopathy
- 3- Unrecognized Martin-Gruber anastomosis
- 4- Unrecognized transligamentous motor branch of the median nerve
- 5- Median artery pseudoaneurysm
- Question 02.104
- Answer = 4
- Reference(s)
- Lanz U: Anatomical variations of the median nerve in the carpal tunnel. J Hand Surg Am1977;2:44-53.
- Bennett JB Crouch CC: Compression syndrome of the recurrent motor branch of the median nerve. J Hand Surg Am 1982;7:407-409.
- 02.105.
- The sartorius muscle can be identified along the medial aspect of the knee joint by its
- 1- position deep to the medial collateral ligament.
- 2- tendinous character.
- 3- insertion on the posteromedial aspect of the tibia.
- 4- central location in the pes anserinus expansion.
- 5- relationship to the saphenous nerve on its posterior border.
- Question 02.105
- Answer = 5
- Reference(s)
- Hoppenfeld S, deBoer P: Surgical Exposures in Orthopaedics: The Anatomical Approach, ed 2. Philadelphia, PA, JB Lippincott, 1994, pp 389-441
- 02. 106.
- A patient injured in a motor vehicle accident has a dislocation of the left shoulder and a contralateral clavicle fracture. He has a Glasgow Coma Scale score of 9. Attempts to obtain a lateral cervical spine radiograph visualizing C7-T 1 are unsuccessful. Which of the following studies should be obtained to "clear" the cervical spine?
- 1- MRI
- 2- CT with sagittal reconstruction
- 3- CT myelography
- 4- Flexion-extension lateral radiograph
- 5- Pillar views
- Question 02.106
- Answer = 2
- Reference(s)
- Beme JD, Velmahos GC, El-Tawil Q, et al: Value of complete cervical helical computed tomographicscanning in identifying cervical spine injury in the unevaluable blunt trauma patient with multiple injuries: A prospective study. J Trauma 1999;47:896-902; Discussion 902-903.
- Tan E, Schweitzer ME, Vaccaro L, Spetell AC: Is computed tomography of nonvisualized C7-Tl cost-effective? J Spinal Disord 1999;12:472-476
- 02. 107.
- What is the preferred treatment of a localized tibial diaphyseal adamantinoma?
- 1- Wide intercalary tibial resection
- 2- Wide resection and chemotherapy
- 3- Radiation therapy and wide resection
- 4- Through-knee amputation
- 5- Below-knee amputation 4 cm proximal to the tumor
- Question 02.107
- Answer = 1
- Reference(s)
- Hazelbag HM, Taminiau AH Fleuren GJ Hogendoorn PC: Adamantinoma of the long bones: A clinicopathological study of thirty-two patients with emphasis on histologic subtype, precursor lesion, and biological behavior. J Bone Joint Surg Am 1994;76:1482-1499.
- Qureshi AA, Shott S, Mallin BA, Gitelis S: Current trends in the management of adamantinoma of long bones: An international study. J Bone Joint Surg Am 2000;82:1122-1131
- 02. 108.
- What is the best indication for intramedullary nailing of the humerus?
- 1- Impending pathologic fracture from metastatic disease
- 2- Open fracture
- 3- Closed fracture with an associated femur fracture
- 4- Closed fracture with radial nerve palsy
- 5- Nonunion of a midshaft fracture
- Question 02.108
- Answer = 1
- Reference(s)
- Koval KJ ed Orthopaedic Knowledge Update 7. Rosemont, IL, American Academy of OrthopaedicSurgeons, 2002, pp 263-271.
- Chapman JR, Hanley MB, Agel J, Benca PJ: Randomized prospective study of humeral shaft fracture fixation: Intramedullary nails versus plates. J Orthop Trauma 2000;14:162-166
- 02. 109.
- Playground injuries in children can best be minimized by installing which of the
- following ground surfaces?
- I- Sand
- 2- Bark
- 3- Rubber
- 4- Asphalt
- >- Concrete
- Question 02.109
- Answer = 3
- Reference(s)
- Mott A, Rolfe K, James R, et al: Safety of surfaces and equipment for children in playgrounds. Lancet1997;349:1874-1876.
- Lillis KA, Jaffe DM: Playground injuries in children. Pediatr Emerg Care 1997;13:149-153
- 02.110.
- Figure 20 shows the radiograph of a patient who has medial foot pain. What is the host course of action in management of the phalanx lesion?
- 1- MRI
- 2- Fourth metatarsophalangeal joint amputation
- 3- Needle biopsy
- 4- Observation
- 5- Open biopsy
- Figures 20
- Question 02.110
- Answer = 4
- Reference(s)
- Stess RM., Tang RE: Enchondroma of the proximal phalanx. J Foot Ankle Surg 1995;34:79-81.
- 02. 111.
- A 32-year-old professional baseball player notes an episode of sharp dorsal u1nar wrist pain after swinging at a bad pitch. Examination reveals that extension and u1nar deviation of the wrist elicit a painful snap. What is the most likely diagnosis?
- 1- Triangular fibrocartilage complex tear
- 2- Distal radioulnar joint capsule tear
- 3- Extensor carpi u1naris subluxation
- 4- Lunotriquetral instability
- 5- Pisotriquetral instability
- Question 02.111
- Answer = 3
- Reference(s)
- Eckhardt WA, Palmer AK: Recurrent dislocation of extensor carpi u1naris tendon. J Hand Surg Am 1981;6:629-631
- 02. 112.
- A 24-year-old female runner has had forefoot pain that is worsened by prolonged weight bearing for the past 3 weeks. Examination reveals tenderness about the third metatarsal neck. A radiograph is shown in Figure 21. What is the most likely cause of her symptoms?
- 1- Stress fracture
- 2- Osteonecrosis of the metatarsal head
- 3- Morton's neuroma
- 4- Infection
- 5- Extensor tendinitis
- Figures 21
- Question 02.112
- Answer =1
- Reference(s)
- McBryde AM Jr: Stress fractures in runners. Clin Sports Med 1985;4:737-752.
- McBryde AM: Stress fractures of the foot and ankle, in Drez DJ, DeLee JC eds Orthopaedic SportsMedicine. Philadelphia, PA, WB Saunders, 1994, pp 1970-1981.
- 02. 113.
- An 18-month-old child sustains acute blunt trauma to the knee. Standard AP and lateral radiographs fail to show the child's patella. What is the most probable explanation for this inability to view the patella?
- 1- Osteochondral dysplasia
- 2- Normal growth and development
- 3- Osteomyelitis of the patella
- 4- Dislocation of the patella
- 5- Osteonecrosis of the patella
- Question 02.113
- Answer = 2
- Reference(s)
- Ogden JA: Diagnostic imaging, in Ogden JA ed Skeletal Injury in the Child, ed 2. New York, NY, Springer-Verlag, 2000, pp 115-146.
- Ogden JA: Knee, in Ogden JA ed Skeletal Injury in the Child, ed 2. New York, NY, Springer-Verlag, 2000, pp 929-990.
- Herring JA: Growth and development, in Herring JA ed Tachdjian's Pediatric Orthopaedics, ed 3.Philadelphia, PA, WB Saunders, 2002, pp 3-22.
- Herring JA: Lower extremity injuries, in Herring JA ed Tachdjian's Pediatric Orthopaedics, ed 3. Philadelphia, PA, WB Saunders, 2002, pp 2251-2438.
- 02. 114.
- During the exposure of the middle window of the ilioinguinal approach, the surgeon encounters a vascular bundle crossing the superior pubic ramus. The artery is most likely an anastamosis between which of the following arteries'?
- 1- Superficial epigastric and common femoral
- 2- Superficial femoral and profunda femoris
- 3- Internal iliac and superior gluteal
- 4- External iliac and internal iliac
- 5- External iliac and obturator
- Question 02.114
- Answer = 5
- Reference(s)
- Tornetta P III, Hochwald N, Levine R: Corona mortis: Incidence and location. Clin Orthop 1996;329:97-101.
- Gilroy AM, Hermey DC, DiBenedetto LM, Marks SC Jr, Page DW, Lei QF: Variability of the obturator vessels. Clin Anat 1997;10:328-332.
- 02.115.
- What is the primary effect of vitamin D?
- 1- Strongly stimulates intestinal absorption of calcium and phosphate
- 2- Stimulates parathyroid hormone release
- 3- Stimulates calcium release from bone
- 4- Promotes urinary excretion of phosphate
- 5- Inhibits osteoclastic bone resorption
- Question 02.115
- Answer = 1
- Reference(s)
- Buckwalter JA, Einhorn TA, Simon SR eds Orthopaedic Basic Science: Biology and Biomechanics of the Musculoskeletal System, ed 2. Rosemont, IL, American Academy of Orthopaedic Surgeons, 2000, pp 319-369
- 02. 116.
- A 30-year-old professional athlete injured the lateral aspect of his right foot 1 week ago. Examination reveals tenderness only at the proximal aspect of the fifth metatarsal. A radiograph is shown in Figure 22. Based on these findings, what is the best course of action?
- 1- Ankle-stirrup
- 2- Short leg cast
- 3- Postoperative hard shoe
- 4- Fracture brace
- 5- Internal fixation
- Figures 22
- Question 02.116
- Answer = 5
- Reference(s)
- Kavanaugh JH, Brower TD, Mann RV: The Jones fracture revisited. J Bone Joint Surg Am1978;60:776-782.
- Mindrebo N, Shelbourne KD Van Meter CD, Rettig AC: Outpatient percutaneous screw fixation of the acute Jones fracture. Am J Sports Med 1993;21:720-723l
- 02. 117.
- A 12-year-old girl underwent pinning of a slipped capital femoral epiphysis of the right hip 3 years ago. At the same time, the left hip, which was normal, was pinned prophylactically as shown in Figure 23a. She was lost to follow-up for 3 years but now reports a limp, pain with weight bearing, and mild loss of motion in the left hip. The current radiograph is shown in Figure 23b. Management should now consist of
- 1-application of skin traction to the left lower extremity.
- 2-subtotal capsulectomy of the hip.
- 3-removal of the hardware.
- 4- advancement of the existing screw into the femoral head.
- 5- revision of the screw fixation.
- Figures 23a
- Figures 23b
- Question 02.117
- Answer = 5
- Reference(s)
- Emery RJ Todd RC, Dunn DM: Prophylactic pinning in slipped upper femoral epiphysis: Prevention of complications. J Bone Joint Surg Br 1990;72:217-219.
- Roy RR, Crawford AH Idiopathic chondrolysis of the hip: Management by subtotal capsulectomy and aggressive rehabilitation. J Pediatr Orthop 1988;8:203-207.
- 02. 118.
- Figure 24 shows the sagittal T_-weighted MRI scan of an 1 1-year-old patient who was a nonrestrained passenger in a motor vehicle accident. Examination reveals no neurologic deficit. What is the most likely mechanism of injury?
- 1- Extension
- 2- Rotation
- 3- Flexion-distraction
- 4- Extension-distraction
- 5- Distraction
- Question 02.118
- Answer = 3
- Reference(s)
- Ferguson RL, Allen BL Jr: A mechanistic classification of thoracolumbar spine fractures. Clin Orthop 1984;189:77-88.
- 02. 119.
- History reveals that a 3-year-old boy did not walk independently until age 20 months, and his gait always has been abnormal. In addition, he has no bowel or bladder control. An AP radiograph of the lumbar spine and pelvis is shown in Figure 25. Observation of the child's gait will most likely reveal
- I- a gluteus mediusl urch.
- 2- a short leg pattern.
- 3- an antalgic limp.
- 4- toe walking.
- 5- hyperextension of the hips.
- Figures 25
- Question 02.119
- Answer = 1
- Reference(s)
- Herring JA: Disorders of the spinal cord, in Herring JA ed Tachdjian's Pediatric Orthopaedics, ed 3. Philadelphia, PA, WB Saunders, 2002, pp 1249-1320.
- Phillips WA: Sacral agenesis, in Weinstein SL ed The Pediatric Spine Principles and Practice, ed 2. Philadelphia, PA, Lippincott Williams & Wilkins, 2001, pp 193-201.
- Van Buskirk CS, Ritterbusch JF: Natural history of distal spinal agenesis. J Pediatr Orthop Br 1997;6:146-152.
- 02. 120.
- A 6-year-old girl has had intermittent shoulder pain. Her mother reports the recent development of swelling over the scapula. A bone scan shows this to be an isolated bone lesion. Plain radiographs, an MRI scan, and a biopsy specimen are shown in Figures 26a through 26d. What is the most likely diagnosis?
- I- Osteosarcoma
- 2- Ewing's sarcoma
- 3- Chondrosarcoma
- 4- Rhabdomyosarcoma
- 5- Langerhan's cell histiocytosis
- Figures 26a
- B
- C
- B
- D
- Question 02.120
- Answer = 2
- Reference(s)
- McCarthy EF Frassica FJ: Primary bone tumors, in McCarthy EF Frassica FJ eds Pathology of Bone and Joint Disorders with Clinical and Radiographic Correlation. Philadelphia, PA, WB Saunders, 1998, pp 258-260
- 02. 121.
- Figures 27a and 27b show the clinical photograph and radiograph of a 22-year-old man
- who was involved in an altercation in a bar 2 hours ago. Examination reveals that he has full active extension of all fingers and can make a fist with minimal pain. Management should consist of
- 1- IV antibiotics, splinting, and observation.
- 2- IV antibiotics and immediate debridement in
- the operating room
- 3- local wound care and delayed fixation of the
- metacarpal head fracture.
- 4- irrigation of the wound in the emergency department,
- oral antibiotics, and immediate active range-
- of-motion exercises.
- 5- insertion of an indwelling joint catheter with serial
- infusion of antibiotic fluids.
- 27 A
- B
- Question 02.121
- Answer = 2
- Reference(s)
- Mennen U, Howells CJ: Human fight-bite injuries of the hand: A study of 100 cases within 18 months. J Hand Surg Br 1991;16:431-435.
- Zubowicz VN, Gravier M: Management of early human bites of the hand: A prospective randomized study. Plast Reconstr Surg 1991;88:111-114
- 02. 122.
- The sudden development of hypotension and hypoxia is noted in a 72-year-old woman who is undergoing cemented total hip arthroplasty. What is the most likely cause of these changes?
- 1- Blood loss
- 2- Cement pressurization of the femoral component
- 3- Elevated serum levels of methacrylate monomer
- 4- Myocardial infarction
- 5- Preexisting deep venous thrombosis
- Question 02.122
- Answer = 2
- Reference(s)
- Ries MD, Lynch F, Rauscher LA, Richman J, Mick C, Gomez M: Pulmonary function during and after total hip replacement: Findings in patients who have insertion of a femoral component with and without cement. J Bone Joint Surg Am 1993;75:581-587.
- Pitto RP, Hamer H, Fabiani R, Radespiel-Troeger M, Koessler M: Prophylaxis against fat and bone-marrow embolism during total hip arthroplasty reduces the incidence of postoperative deep-vein thrombosis: A controlled, randomized clinical trial. J Bone Joint Surg Am 2002;84:39-48.
- 02. 123.
- Nonsurgical management is most appropriate for which of the following transverse patellar fractures?
- 1- Nondisplaced fracture in an active 25-year-old laborer who can perform a straight leg raise
- 2- Minimally displaced fracture in a 40-year-old patient who cannot perform a straight
- leg raise
- 3- Displaced fracture in an 80-year-old retired farmer
- 4- Displaced fracture in a patient on renal dialysis
- 5- Displaced fracture in a patient with diabetes mellitus
- Question 02.123
- Answer = 1
- Reference(s)
- Kellam JF, Fischer TJ, Tornetta P III, Bosse MJ Harris MB eds Orthopaedic Knowledge Update: Trauma 2. Rosemont, IL, American Academy of Orthopaedic Surgeons, 2000, pp 157-166.
- Pritchett JW Nonoperative treatment of widely displaced patella fractures. Am J Knee Surg 1997;10:145-148
- 02. 124.
- An 18-year-old college athlete sustained injuries in a skiing accident 3 months ago. Examination and radiographs reveal a healing lateral tibial plateau fracture with 1 mm of joint incongruity, a reconstructed anterior cruciate ligament tear, good knee stability, normal axial alignment, a resolving hip contusion, good ankle and hip stability, no ankle tenderness, and a normal radiographic appearance of the ankle and hip. What is the overall prognosis for these injuries?
- I - Greater incidence of posttraumatic knee arthritis but not hip arthritis
- 2- Greater incidence of posttraumatic hip arthritis but not knee arthritis
- 3- Greater incidence of both posttraumatic hip and knee arthritis
- 4- Greater incidence of posttraumatic arthritis of the hip, knee, and ankle
- 5- Equivalent incidence of posurautnatic hip or knee arthritis
- Question 02.124
- Answer = Item Deleted
- Reference(s)
- 02. 125.
- A 3-year-old boy refuses to bear weight on his right lower extremity. His parents report that he has had a cold for 3 days with intermittent chills. He has an oral temperature of 100.6°F (37.7°C). Examination reveals that the right hip is irritable on internal and external rotation. Laboratory studies show an erythrocyte sedimentation rate of 51 mm/h (normal up to 20 mm/h) and a serum WBC of 9,500/mm3 (normal 3,50010,500/mm3). The next most appropriate step in management should consist of
- 1- hospital admission and observation for 24 hours.
- 2- an outpatient bone scan.
- 3- MRI of the hips.
- -i- blood cultures.
- 5- aspiration of the hip.
- Question 02.125
- Answer = 5
- Reference(s)
- Kocher MS, Zurakowski D, Kasser JR: Differentiating between septic arthritis and transient synovitis of the hip in children: An evidence-based clinical prediction algorithm. J Bone Joint Surg Am 1999;81:1662-1670.
- Kunnamo I, Kallio P, Pelkonen P, Hovi T: Clinical signs and laboratory tests in the differential diagnosis of arthritis in children. Am J Dis Child 1987;141:34-40
- 02. 126.
- A 4-year-old girl is referred for an asymptomatic abnormality of the distal fcmur. An ohlique radiograph is shown in Figure 28. What is the next step in evaluation?
- I- Comparison radiographs of the contralateral knee
- 2- Total body bone scan
- 3- MRI
- 4- Aspiration for culture and Gram stain
- 5- Incisional biopsy
- Figures 28
- Question 02.126
- Answer = Item Deleted
- Reference(s)
- 02. 127.
- Type I skeletal muscle fibers are characterized by
- l - fast speed of contraction.
- 2- low strength of contraction.
- 3- low aerobic capacity.
- 4- high anaerobic capacity.
- 5- large motor unit size.
- Question 02.127
- Answer = 2
- Reference(s)
- Buckwalter JA, Einhorn TA, Simon SR eds Orthopaedic Basic Science: Biology and Biomechanics of the Musculoskeletal System, ed 2. Rosemont, IL, American Academy of Orthopaedic Surgeons, 2000, pp 683-716.
- Gauthier GF Skeletal muscle fiber types, in Engle AG Banker BQ eds Myology: Basic and Clinical. New York, NY, McGraw-Hill, 1994, pp 255-283.
- 02. 128.
- A patient undergoes anterior drawer stress examination of both ankles under anesthesia. A lateral radiograph of the symptomatic ankle is shown in Figure 29a, and the asymptomatic ankle is shown in Figure 29b. The difference seen in these two studies is the result of
- 1- normal side-to-side differences.
- 2- injury to the anterior inferior tibiofibular ligament.
- 3- injury to the anterior talofibular ligament.
- 4- disruption of the posterior capsule of the ankle.
- 5- rupture of the Achilles tendon.
- 29 A
- B
- Question 02.128
- Answer = 3
- Reference(s)
- Clanton TO: Athletic injuries to the soft tissues of the foot and ankle, in Coughlin MJ Mann RA eds Surgery of the Foot and Ankle, ed 7. St Louis, MO, Mosby Publishers, 1999, pp 1090-1209.
- Kannus P, Renstrom P: Treatment for acute tears of the lateral ligaments of the ankle: Operation, cast, or early controlled mobilization. J Bone Joint Surg Am 1991;73:305-312.
- Marder RA: Current methods for the evaluation of ankle ligament injuries. Instr Course Lect 1995;44:349-357.
- 02. 129.
- The arterial supply to the lateral arm flap is supplied by which of the following arteries''
- l- Posterior radial collateral
- 2- Circumflex scapular
- 3- Thoracodorsal
- 4- Recurrent radial
- 5- Posterior circurnflex humeral
- Question 02.129
- Answer = 1
- Reference(s)
- Katsaros J, Schusterman M, Beppu M, Banis JC Jr, Acland RD: The lateral upper arm flap: Anatomy and clinical applications. Ann Plast Surg 1984;12:489-500.
- Masquelet A, Gilbert A: An Atlas of Flaps in Limb Reconstruction. Philadelphia, PA, JB Lippincott, 1995, pp 55-59
- 02. 130.
- A worker in a chemical plant is involved in a hydrofluoric acid spill and has a burn to the dorsum of his hand despite copious water dilution. Management should consist of
- 1- emergent escharotomies and broad-spectrum IV antibiotics.
- 2- emergent neutralization wash with concentrated sodium hydroxide solution.
- 3- phenol injection to the affected area.
- 4- polyethylene glycol injection into the affected area.
- 5- calcium gluconate injection into the affected area.
- Question 02.130
- Answer = 5
- Reference(s)
- Jelenko C III: Chemicals that "burn". J Trauma 1974;14:65-72.
- Salisbury RE, Dingeldein GP: The burned hand and upper extremity, in Green DP, Hotchkiss RN III,
- Pederson WC eds Operative Hand Surgery, ed 3. Philadelphia, PA, Churchill Livingstone, 1993, pp 2007-2031.
- 02.131.
- A 10-year-old girl sustained a physeal fracture of the distal medial tibial physis 6 months ago. Figures 30a and 30b show current radiographs of a physeal bar. Cross-section CT scans show that the bar surface area is less than 50% of the physis. The next most appropriate step in management should consist of
- 1- observation.
- 2- physeal bar resection with interposition.
- 3- osteotomy of the distal tibia.
- 4- epiphysiodesis of the distal tibia.
- 5- epiphysiodesis of the distal tibia and fibula.
- Figures 29A
- B
- 30 A
- B
- Question 02.131
- Answer = 2
- Reference(s)
- Carlson WO, Wenger DR: A mapping method to prepare for surgical excision of a partial physeal arrest. J Pediatr Orthop 1984;4:232-238.
- Price C, Phillips J, Devito D: Management of fractures, in Morrissy RT Weinstein SL eds Lovell and Winter's Pediatric Orthopaedics, ed 5. Philadelphia, PA, Lippincott Williams & Wilkins, 2001, pp 1319-1422.
- 02. 132.
- Specific to the hip joint, the use of osteoarticular allografts is not preferred after tumor resections of the proximal femur because of which of the following factors?
- 1- Increased infection rate
- 2- Fixation of the allograft to the remaining femur
- 3- Insufficient supply of grafts
- 4- Poor abductor function
- 5- Inability to match femoral head size exactly
- Question 02.132
- Answer = 5
- Reference(s)
- Mankin HJ Gebhardt MC, Jennings LC, Springfield DS, Tomford WW: Long-term results of allograft replacement in the management of bone tumors. Clin Orthop 1996;324:86-97.
- 02. 133.
- Which of the following factors is associated with a higher incidence of low back pain?
- 1- Limb-length discrepancy
- 2- Increased lumbosacral angle
- 3- Spondylolisthesis
- 4- Transitional vertebra
- 5- Smoking cigarettes
- Question 02.133
- Answer = 5
- Reference(s)
- Rowe ML: Low back pain in industry: A position paper. J Occup Med 1969;11:161-169.
- Frymoyer JW Pope MH Clements JH, Wilder DG, MacPherson B, Ashikaga T: Risk factors in low back pain: An epidemiological survey. J Bone Joint Surg Am 1983;65:213-218
- 02. 134.
- A 73-year-old woman has had gradually worsening right groin pain for the past several years; however, she has never sought medical attention for her symptoms. A plain radiograph, bone scan, and CT scan are shown in Figures 31a through 31c. Initial management should consist of
- I- acetaminophen.
- 2- bisphosphonates.
- 3- total hip arthroplasty.
- 4- incisional biopsy.
- 5- radiation therapy.
- 31 A
- C
- B
- Question 02.134
- Answer = 2
- Reference(s)
- Gutteridge DH, Retallack RW, Ward LC, Stuckey BG et al: Clinical, biochemical, hematologic, and radiographic responses in Paget's disease following intravenous pamidronate disodium: A 2-year study. Bone 1996;19:387-394.
- Rongstad KM, Wheeler DL, Enneking W.F.: A comparison of the amount of vascularity in pagetic and normal human bone. Clin Orthop 1994;306:247-249.
- 02. 135.
- Insertion of an iliosacral screw with the screw tip exiting the sacral ala anteriorly most likely results from failure to adequately visualize the screw tract on what radiographic view of the pelvis?
- 1- AP
- 2- Inlet
- 3- Outlet
- 4- Obturator oblique
- 5- Iliac oblique
- Question 02.135
- Answer = 2
- Reference(s)
- Kellam JF, Fischer TJ, Tornetta P III, Bosse MJ Harris MB eds Orthopaedic Knowledge Update: Trauma 2. Rosemont, IL, American Academy of Orthopaedic Surgeons, Rosemont, IL, 2000, pp 267-275.
- Matta JM Tornetta P III: Internal fixation of unstable pelvic ring injuries. Clin Orthop 1996;329:129-140
- 02. 136.
- Under the principle of patient autonomy, which of the following is not a physician's responsibility?
- 1- Respect a patient's autonomy.
- 2- Support a patient's right to make informed decisions about his or her treatment.
- 3- Be certain that a patient's decisions conform with principles of ethical practice.
- 4- Allow a patient unconstrained rights to decide on his or her own treatment.
- 5- Withhold care if a patient believes it to be inappropriate.
- Question 02.136
- Answer = Item Deleted
- Reference(s)
- 02. 137.
- The incidence of acute hematogenous osteomyelitis resulting from Haemophilus influenzae has
- 1- increased markedly.
- 2- decreased markedly.
- 3- remained the same.
- 4- remained the same as the incidence of Staphylococcus aureus.
- 5- remained the same as other gram-positive organisms.
- Question 02.137
- Answer = 2
- Reference(s)
- Bowerman SG, Greene NE, Mencio GA: Decline of bone and joint infections attributable to haemophilia influenza type b. Clin Orthop 1997;341:128-133.
- Morrissy RT.: Bone and joint sepsis, in Morrissy RT.: Weinstein SL eds Lovell and Winter's Pediatric Orthopaedics, ed 5. Philadelphia, PA, Lippincott Williams & Wilkins, 2001, pp 459-507.
- 02. 138.
- Six months after undergoing a distal soft-tissue realignment procedure to correct a hallux valgus deformity, the patient notes progressive deformity and shoe wear problems. A standing radiograph is shown in Figure 32. What is the most likely cause of this problem?
- 1- Undercorrection of the intermetatarsal angle
- 2- Inadequate repair of the medial capsule of the MTP joint
- 3- Excessive release of the lateral capsular soft tissues
- 4- Contracture of the flexor hallucis longus
- 5- Contracture of the medial metatarsosesamoid ligament
- 32
- Question 02.138
- Answer = 3
- Reference(s)
- Mann RA, Coughlin MJ Adult hallux valgus, in Coughlin MJ Mann RA eds Surgery of the Foot and Ankle, ed 6. St Louis, MO, Mosby Publishers, 1993, pp 284-294.
- Lutter LD, Mizel MS, Pfeffer GB eds Orthopaedic Knowledge Update: Foot and Ankle. Rosemont, IL, American Academy of Orthopaedic Surgeons, 1994, pp 141-154
- 02. 139.
- Figures 33a and 33b show the radiographs of a 17-month-old girl who has a hip
- abnormality. Examination reveals marked limitation of abduction of the right hip. What
- is the next most appropriate step in management?
- 1- Hip spica cast
- 2- Closed reduction, adductor myotomy, and hip spica cast
- 3- Open reduction
- 4- Open reduction and femoral shortening
- 5- Observation
- 33A
- 33B
- Question 02.139
- Answer = 2
- Reference(s)
- Gabuzda GM, Renshaw TS: Reduction of congenital dislocation of the hip. J Bone Joint Surg Am 1992;74:624-631.
- Weinstein SL: Developmental hip dysplasia, in Morrissy RT.: Weinstein SL eds Lovell and Winter's Pediatric Orthopaedics, ed 5. Philadelphia, PA, Lippincott Williams & Wilkins, 2001, pp 906-956
- 02. 140.
- Low-grade malignant cartilage tumors of bone are best distinguished from benign cartilage tumors by
- 1- immunohistochemical studies.
- 2- evaluating pain pattern and staging studies.
- 3- uptake on a bone scan.
- 4- the histologic appearance.
- 5- the location of the lesion.
- Question 02.140
- Answer = 2
- Reference(s)
- Marco RA, Gitelis S, Brebach GT., Healy JH: Cartilage tumors: Evaluation and treatment. J Am Acad Orthop Surg 2000;8:292-304
- 02.141.
- During revision of a failed total hip femoral component, the new implant should bypass the most distal cortical defect by a minimum of how many cortical diameters?
- I- 0
- 2- 1
- 3- 2
- 4- 3
- 5- 4
- Question 02.141
- Answer = 3
- Reference(s)
- Maurer SG, Baitner AC, Di Cesare PE: Reconstruction of the failed femoral component and proximal bone loss in revision hip surgery. J Am Acad Orthop Surg 2000;8:354-363.
- Haddad FS, Masri BA, Garbuz DS, Duncan CP: Femoral bone loss in total hip arthroplasty:
- Classification and preoperative planning. Instr Course Lect 2000;49:83-96.
- 02.142.
- A 32-year-old man has a C6 fracture. Examination reveals a C7 sensory level and absence of motor strength in the C7 muscles and below. His posterior column functions and bulbocavernosus reflex are intact. What is the most likely diagnosis?
- 1- Spinal shock
- 2- Posterior cord syndrome
- 3- Anterior cord syndrome
- 4- Central cord syndrome
- 5- Complete spinal cord injury
- Question 02.142
- Answer = 3
- Reference(s)
- McGuire RA: Physical examination in spinal trauma, in Levine AM, Eismont FJ, Garfin SR, Zigler JE eds Spinal trauma. Philadelphia, PA, WB Saunders, 1998, pp 16-27.
- Wiesel SW, Lauerman WC: Adult spinal disorders: Spine trauma, in Wiesel SW, Delahay JN eds Principles of Orthopaedic Medicine and Surgery. Philadelphia, PA, WB Saunders, 2001, pp 439-464
- 02. 143.
- A 25-year-old man has had medial knee pain for the past 5 years that has been worsening over the last 6 months with a progressive varus deformity of the leg. History reveals that he underwent a partial medial meniscectomy at age 16 years. Examination reveals no knee effusion, active range of motion from 3° to 125°, a positive Lachman test, increased lateral joint line opening of 6 mm, and a standing mechanical axis of 11° of varus. What is the next most appropriate step in treatment?
- 1- Meniscus allograft
- 2- Posterolateral reconstruction
- 3- Anterior cruciate ligament reconstruction
- 4- High tibial osteotomy
- 5- Supracondylar femoral osteotomy
- Question 02.143
- Answer = 4
- Reference(s)
- Noyes FR, Barber-Westin SD, Hewett TE: High tibial osteotomy and ligament reconstruction for varus angulated anterior cruciate ligament-deficient knees. Am J Sports Med 2000;28:282-296.
- Noyes FR, Roberts CS: High tibial osteotomy in knees with associated chronic ligament deficiencies, in Jackson DW (ed): Master Techniques in Orthopaedic Surgery: Reconstructive Knee Surgery. New York, NY, Raven Press, 1995, pp 185-210
- 02. 144.
- A 14-year-old boy with distal limb arthrogryposis reports left forefoot pain. He ambulates with mild circumduction because of moderately severe and rigid equinus of the left foot that has recurred following talectomy. There is a tender callus located plantar to the fifth metatarsal head. The hindfoot is in mild varus. A radiograph of the foot in maximum dorsiflexion is shown in Figure 34. Treatment should consist of
- 1- shaving of the plantar callus.
- 2- excision of the fifth metatarsal head.
- 3- transfer of the anterior tibial tendon to the lateral cuneiform.
- 4- Dwyer calcaneal osteotomy with open wedge osteotorny of the medial cuneiform.
- 5- supramalleolar osteotomy.
- Figure 34
- Question 02.144
- Answer = 5
- Reference(s)
- Napiontek M, Nazar J: Tibial osteotomy as a salvage procedure in the treatment of congenital talipes equinovarus. J Pediatr Orthop 1994;14:763-767.
- Handelsmann JE, et al: Correction of fixed equinovarus deformity by supramalleolar osteotomy in older children. Presented at POSNA Newport, RI, May 6-9, 1992. J Pediatr Orthop 1993;13:115.
- 02. 145.
- A 20-year-old man who underwent arthroscopic Bankart repair 1 year ago for recurrent anterior inferior dislocations now reports two episodes of subluxation. He denies any history of additional trauma. A CT arthrogram is shown in Figure 35. What is the most likely cause of the persistent instability?
- 1- Capsular redundancy
- 2- Labral detachment
- 3- Subscapularis tear
- 4- Glenoid deficiency
- 5- Hill-Sachs lesion
- Figure 35
- Question 02.145
- Answer = 4
- Reference(s)
- Green MR, Christensen KP Arthroscopic Bankart procedure: Two- to five-year followup with clinical correlation to severity of glenoid labral lesion. Am J Sports Med 1995;23:276-281.
- Walch G, Boileau P, Levigne C, Mandrino A, Neyret P, Donell S: Arthroscopic stabilization or recurrent anterior shoulder dislocation: Results of 59 cases. Arthroscopy 1995;11:173-179
- 02. 146.
- A 33-year-old man sustains the closed fractures shown in Figures 36a and 36b. The
- neurologic and vascular status of the extremity are normal. What is the most appropriate
- initial management?
- 1- Open reduction and internal fixation of the tibia and talus
- 2- Open reduction and internal fixation of the tibia with ankle arthrodesis
- 3- Open reduction of the tibia and talus and application of a hybrid external fixator
- 4- Closed reduction and a long leg cast
- 5- Spanning external fixation of the ankle joint
- 36 A
- B
- Question 02.146
- Answer = 5
- Reference(s)
- Sirkin M, Sanders R, DiPasquale T, Herscovici D Jr: A staged protocol for soft tissue management in the treatment of complex pilon fractures. J Orthop Trauma 1999;13:78-84.
- Patterson MJ, Cole JD: Two-staged delayed open reduction and internal fixation of severe pilon fractures. J Orthop Trauma 1999;13:85-91.
- 02. 147.
- Which of the following terms is defined as the probability of finding an effect when, in fact, there is not one (type I statistical error)?
- 1- Statistical power
- 2- Correlation coefficient
- 3- Standard deviation
- 4- Significance level
- 5- Regression analysis
- Question 02.147
- Answer = 4
- Reference(s)
- Buckwalter JA, Einhorn TA, Simon SR eds Orthopaedic Basic Science: Biology and Biomechanics of the Musculoskeletal System, ed 2. Rosemont, IL, American Academy of Orthopaedic Surgeons, 2000, pp 2-17
- 02. 148.
- With dorsiflexion of the ankle, the fibula will move in which of the following directions?
- 1- Internal rotation and lateral translation
- 2- Internal rotation and medial translation
- 3- External rotation and medial translation
- 4- External rotation and distal translation
- 5- External rotation and proximal translation
- Question 02.148
- Answer = 5
- Reference(s)
- Scranton PE Jr, McMaster JG Kelly E: Dynamic fibular function: A new concept. Clin Orthop 1976;118:76-81.
- Michelson JD, Checcone M, Kuhn T, Varner K: intra-articular load distribution in the human ankle joint during motion. Foot Ankle Int 2001;22:226-233
- 02. 149.
- An active 80-year-old woman with progressively worsening left hip pain is unable to walk after falling. A radiograph is shown in Figure 37. Treatment should consist of
- 1- open reduction and internal fixation with a screw and plate.
- 2- open reduction and internal fixation with an intramedullary hip screw.
- 3- open reduction and internal fixation with delayed conversion to total hip
- arthroplasty.
- 4- primary total hip arthroplasty.
- 5- hemi-arthroplasty.
- Figure 37
- Question 02.149
- Answer = 4
- Reference(s)
- Chan KC Gill GS: Cemented hemiarthroplasties for elderly patients with intertrochanteric fractures. Clin Orthop 2000;371:206-215
- 02. 150. (deleted)
- Figures 38a and 38b show the radiographs of a 65-year-old woman with progressive pain over the first metatarsophalangeal joint with a 40 degree arc of motion. She also reports difficulty with shoe wear. Surgical treatment should consist of
- 1-interpositional arthroplasty with an autogenous tendon.
- 2-cheilectomy.
- 3-arthrodesis.
- 4-a Silastic implant.
- 5- Keller procedure.
- B
- Figure 38 A
- Question 02.150
- Answer = Item Deleted
- Reference(s)
- 02. 151.
- A 32-year-old man reports gradually worsening right arm pain and an enlarging mass over the medial aspect of his distal arm. A radiograph, T1- and T2-weighted coronal MRI scans, and a biopsy specimen are shown in Figures 39a through 39d. What is the most appropriate management?
- 1- Wide excision, radiation therapy, and chemotherapy
- 2- Wide excision and radiation therapy
- 3- Wide excision and chemotherapy
- 4- Wide excision only
- 5- Radiation therapy only
- A
- B
- C
- C
- D
- Question 02.151
- Answer = 3
- Reference(s)
- Okada K, Unni KK Swee RG Sim FH High grade surface osteosarcoma: A clinicopathologic study of 46 cases. Cancer 1999;85:1044-1054.
- Okada K, Kubota H, Ebina T, Kobayashi T, Abe E, Sato K: High-grade surface osteosarcoma of the humerus. Skeletal Radiol 1995;24:531-534.
- Schajowicz F, McGuire MH Santini Araujo E, Muscolo DL, Gitelis S: Osteosarcomas arising on the surfaces of long bones. J Bone Joint Surg Am 1988;70:555-564.
- Wold LE, Unni KK Beabout JW Pritchard DJ: High-grade surface osteosarcomas. Am J Surg Pathol 1984;8:181-186
- 02. 152.
- Figure 40 shows an axial CT scan at the L2-L3 level. The tip of the arrow lies in what
- structure?
- I- Pedicle of L2
- 2- Pedicle of L3
- 3- Inferior articular facet of L2
- 4- Superior articular facet of L2
- 5- Superior articular facet of L3
- Figure 40
- Question 02.152
- Answer = 5
- Reference(s)
- Parke WW: Applied anatomy of the spine, in Herkowitz HN Garfin SR, Balderston RA, Eismont FJ,
- Bell GR, Wiesel SW eds The Spine, ed 4. Philadelphia, PA, WB Saunders, 1999, pp 29-73
- 02. 153.
- An intoxicated 27-year-old man who sustains a closed head injury in a motor vehicle
- accident is comatose upon arrival at the hospital. Radiographs of the cervical spine
- reveal a bilateral C4-C5 facet dislocation. Closed reduction with tongs is performed in
- the emergency room. Examination now reveals slight malalignment (approximately 10°k subluxation). The patient is still intubated and sedated, and the neurosurgical consultant feels that the patient may not be capable of giving informed consent for several weeks. His family members all refuse to become involved in his care and decision making. Both the orthopaedic and neurosurgical consultants agree that surgical stabilization and fusion, within the next few days, is the optimal course of treatment. What is the next most appropriate step in management?
- 1- Defer surgery until the patient can give informed consent.
- 2- Perform surgery without informed consent.
- 3- Ask a judge or representative of the court to give consent.
- 4- Ask the hospital Ethics Committee to give consent.
- 5- Declare a "medical emergency" and perform surgery immediately.
- Question 02.153
- Answer = 3
- Reference(s)
- Kahler DM, Davis W: Inability to obtain formal informed consent in the face of a standard surgical indication. J Bone Joint Surg Am 2001;83:142-143
- 02. 154.
- Which of the following clinical findings are considered musculoskeletal features of Ehlers-Danlos syndrome?
- I- Congenital hip dislocation, clubfoot, scoliosis
- 2- Congenital hip dislocation, joint hypermobility, syndactyly of the hands and feet
- 3- Frequent fractures, wormian bones, syndactyly of the hands and feet
- 4- Joint hypermobility, subtalar coalition, duplication of the digits
- 5- Muscular hypotonia, slow motor development, terminal arnelia
- Question 02.154
- Answer = 1
- Reference(s)
- Giunta C, Supert-Furga A, Spranger S, Cole WG Steinmann B: Ehlers-Danlos syndrome type VII: Clinical features and molecular defects. J Bone Joint Surg Am 1999;81:225-238.
- McMaster MJ: Spinal deformity in Ehlers-Danlos syndrome: Five patients treated by spinal fusion. J Bone Joint Surg Br 1994;76:773-777
- 02. 155.
- A 19-year-old man is combative, confused, and requires intubation after a motor vehicle accident. An AP radiograph of the pelvis reveals 3 cm of symphyseal separation and bilateral anterior widening of the sacroiliac joints. Abdominal ultrasound reveals no obvious pathology. After administration of IV crystalloid, the patient is normotensive. What is the most appropriate next step in management?
- 1- External fixation of the pelvis
- 2- Open reduction and internal fixation of the pubic symphysis
- 3- CT of the head
- 4- CT of the abdomen
- 5- Exploratory laparotomy
- Question 02.155
- Answer = 3
- Reference(s)
- Kellam JF Fischer TJ, Tornetta P III, Bosse MJ, Harris MB eds Orthopaedic Knowledge Update: Trauma 2. Rosemont, IL, American Academy of Orthopaedic Surgeons, 2000, pp 229-237.
- Beaty JH (ed): Orthopaedic Knowledge Update 6. Rosemont, IL, American Academy of Orthopaedic Surgeons, 1999, pp 123-130.
- 02. 156.
- At what point during total knee arthroplasty is the vascular supply to the patella most at risk`?
- 1- Lateral release
- 2- Fat pad excision
- 3- Lateral meniscectomy
- 4- Peripheral patella peg hole reaming
- 5- Midvastus approach
- Question 02.156
- Answer = 1
- Reference(s)
- Dennis DA: Periprosthetic fractures following total knee arthroplasty. Instr Course Lect 2001;50:379-389.
- McMahon MS, Scuderi GR, Glashow JL Scharf SC, Meltzer LP, Scott WN Scintigraphic determination of patellar viability after excision of infrapatellar fat pad and/or lateral retinacular release in total knee arthroplasty. Clin Orthop 1990;260:10-16.
- 02. 157.
- A 9-month-old infant has a hypoplastic thumb. Examination reveals a narrowed thumbindex web space, hypoplasia of the intrinsic thenar muscles, and thumb carpometacarpal (CMC) and metacarpophalangeal (MP) joint instability. The thumb is radially abducted. Radiographs of the thumb show the full complement of bones, but they are hypoplastic relative to the contralateral normal thumb. Reconstruction should consist of
- 1- ring sublimis opponensplasty, long sublimis flexorplasty, and extensor indicis proprius transfer for
- extension.
- 2- CMC and MP fusion in palmar abduction.
- 3- abductor digiti minimi opponensplasty, extensor indicis transfer for thumb extension, and release of the
- anomolous abductor pollicis brevis.
- 4- thumb ablation and index pollicization.
- 5- vascularized second toe metatarsophalangeal joint transfer to the CMC joint and abductor digiti minimi
- opponensplasty.
- Question 02.157
- Answer = 4
- Reference(s)
- Manske PR, McCarroll HR Jr: Reconstruction of the congenitally deficient thumb. Hand Clin 1992;8:177-196.
- Egloff DV Verdan C: Pollicization of the index finger for reconstruction of the congenitally hypoplastic or absent thumb. J Hand Surg Am 1983;8:839-848
- 02. 158.
- Following, surgical treatment of an intertrochanteric femur fracture with a Fixed angle dynamic hip screw, what factor has the greatest effect on the rate of implant cutout?
- 1- Length of the side plate
- 2- Length of the barrel
- 3- Angle of the device
- 4- Use of a compression screw
- 5- Tip-apex distance
- Question 02.158
- Answer = 5
- Reference(s)
- Kellam JF Fischer TJ, Tornetta P III, Bosse MJ, Harris MB eds Orthopaedic Knowledge Update:Trauma 2. Rosemont, IL, American Academy of Orthopaedic Surgeons, 2000, pp 125-131.
- Baumgaertner MR, Curtin SL, Lindskog DM, Keggi JM The value of the tip-apex distance in predicting failure of fixation of peritrochanteric fractures of the hip. J Bone Joint Surg Am 1995;77:1058-1064.
- 02. 159.
- A 69-year-old man reports the acute onset of nonradicular low back pain, and nonsurgical management results in pain resolution. A bone scan and CT scan are shown in Figures 41a and 41b. What is the most likely diagnosis?
- 1- Enostosis
- 2-Osteoid osteoma
- 3-Osteoblastoma
- 4-Osteosarcoma
- 5- Metastatic prostate carcinoma
- Figures 41a
- 41b
- Question 02.159
- Answer = 1
- Reference(s)
- Flemming DJ, Murphey MD, Carmichael BB, Bernard SA Primary tumors of the spine. SeminMusculoskelet Radiol 2000;4:299-320.
- Murphey MD, Andrews CL, Flemming DJ, Temple HT, Smith WS, Smirniotopoulos JG From the archives of the AIFIP: Primary tumors of the spine. Radiologic, pathologic correlation. Radiographics 1996;16:1131-1158.
- Greenspan A: Bone island (enostosis): Current concept. A review. Skeletal Radiol 1995;24:111-115
- 02.160.
- A 13-year-old girl has had activity-related pain and intermittent swelling at the right second metatarsophalangeal joint for the past year. Nonsurgical management has failed to provide relief. Radiographs are shown in Figures 42a and 42b. Treatment should consist of
- 1- resection of the accessory navicular and advancement of the posterior tibialis tendon
- (Kidner procedure).
- 2- excision of the medial (tibial) sesamoid.
- 3- resection of a coalition.
- 4- bunionectomy.
- 5- joint debridement.
- Figures 42a
- 42b
- Question 02.160
- Answer = 5
- Reference(s)
- Mosca VS: The foot, in Morrissy RT., Weinstein SL eds Lovell and Winter's Pediatric Orthopaedics, ed 5. Philadelphia, PA, Lippincott Williams & Wilkins, 2001, pp 1151-1216.
- Sproul J, Klaaren H, Mannarino F: Surgical treatment of Freiberg's infraction in athletes. Am J Sports Med 1993;21:381-384.
- Herring JA: Disorders of the foot, in Herring JA (ed): Tachdjlan's Pediatric Orthopaedics, ed 3. Philadelphia, PA, WB Saunders, 2002, pp 891-1038
- 02. 161.
- A professional baseball pitcher undergoes u1nar collateral reconstruction of the elbow. What structure of the u1nar collateral ligament is being repaired?
- 1- Anterior bundle
- 2- Posterior bundle
- 3- Transverse bundle
- 4- Medial bundle
- 5- Lateral bundle
- Question 02.161
- Answer = 1
- Reference(s)
- Azar FM, Andrews JR, Wilk KE Groh D: Operative treatment of u1nar collateral ligament injuries of the elbow in athletes. Am J Sports Med 2000;28:16-23.
- Conway JE, Jobe FW Glousman RE, Pink M: Medial instability of the elbow in throwing athletes: Treatment by repair or reconstruction of the ulnar collateral ligament. J Bone Joint Surg Am 1992;74:67-83.
- 02. 162.
- Which of the following is considered a key characteristic of osteoblasts?
- 1- Produce acid phosphatases
- 2- Produce osteocalcin when stimulated by 24,25-dihydroxy vitaminD
- 3- Responsive to parathyroid hormone
- 4- Completely surrounded by mineralized bone matrix
- 5- Resorb mineralized bone
- Question 02.162
- Answer = 3
- Reference(s)
- Buckwalter JA, Einhorn TA, Simon SR eds Orthopaedic Basic Science: Biology and Biomechanics of the Musculoskeletal System, ed 2. Rosemont, IL, American Academy of Orthopaedic Surgeons, 2000, pp 319-369.
- Ducy P, Schinke T, Karsenty G: The osteoblast: A sophisticated fibroblast under central surveillance. Science 2000;289:1501-1504
- 02. 163.
- A 38-year-old man sustains the injury shown in Figures 43a through 43c. According to
- the Letournel and Judet fracture classification system, the fracture of the acetabulum
- would be classified as what pattern?
- l- Anterior column plus posterior hemitransverse
- 2- Associated both column
- 3- Posterior column plus posterior wall
- 4- T-shaped
- 5- Transverse plus posterior wall
- Figures 43a
- 43c
- 43b
- Question 02.163
- Answer = 4
- Reference(s)
- Letournel E, Judet R eds Fractures of the Acetabulum, ed 2. Berlin, Germany, Springer Verlag, 1993, pp 63-66.
- Judet R, Judet J, Letournel E Fractures of the acetabulum: Classification and surgical approaches for open reduction. Bone Joint Surg Am 1964;46:1615.
- 02. 164.
- What type of prosthetic knee is considered most appropriate for a 65-year-old man with a knee disarticulation?
- 1- Constant friction
- 2- Variable friction
- 3- Weight activated
- 4- Polycentric
- 5- Fluid control
- Question 02.164
- Answer = 4
- Reference(s)
- Koval KJ (ed): Orthopaedic Knowledge Update 7. Rosemont, IL, American Academy of Orthopaedic Surgeons, 2002, pp 139-148.
- Pinzur MS, Bowker JH: Knee disarticulation. Clin Orthop 1999;361:23-28
- 02. 165.
- Patients with the most severe neurologic and physical manifestations of fragile X syndrome will most likely have which of the following features?
- 1- Male gender
- 2- Scoliosis
- 3- Joint laxity
- 4- Flat feet
- 5- Pectus excavatum
- Question 02.165
- Answer = 1
- Reference(s)
- Davids JR, Hagerman RJ Eilert RE: Orthopaedic aspects of fragile-X syndrome. J Bone Joint Surg Am 1990;72:889-896.
- Warren ST, Nelson DL: Advances in molecular analysis of fragile X syndrome. JAMA 1994;271:536-542
- 02. 166.
- A 62-year-old woman has a painful swelling in her left hip region. Examination reveals a palpable mass. Staging work-up shows this to be an isolated lesion. A plain radiograph, MRI scans, and a biopsy specimen are shown in Figures 44a through 44d. Management should consist of
- 1- wide excision, radiation therapy, and chemotherapy.
- 2- wide excision and radiation therapy.
- 3- wide excision and chemotherapy.
- 4- wide excision only.
- 5- prophylactic stabilization and radiation therapy.
- 44a
- 44b
- 44c
- 44d
- Question 02.166
- Answer = 4
- Reference(s)
- Campanacci M, Enneking W.F.: Central chrondrosarcomas, in Campanacci M, Enneking W.F.: (eds.): Bone and Soft Tissue Tumors. New York, NY, Springer-Verlag, 1999, pp 283-318.
- McCarthy EF Frassica FJ Primary bone tumors, in McCarthy EF Frassica FJ (eds.): Pathology of Bone and Joint Disorders with Clinical and Radiographic Correlation. Philadelphia, PA, WB Saunders, 1998, pp 235-244
- 02. 167.
- A 15-year-old boy has a both-bone fracture of the forearm. Radiographs after closed reduction are shown in Figure 45. Management should now consist of
- 1- a long arm cast.
- 2- a short arm cast.
- 3- a functional brace.
- 4- open reduction and internal fixation.
- 5- repeat closed reduction.
- Figure 45
- Question 02.167
- Answer = 4
- Reference(s)
- Verstreken L, Delronge G, Lamoureux J: Shaft forearm fractures in children: Intramedullary nailing with immediate motion. A preliminary report. J Pediatr Orthop 1988;8:450-453.
- Price C, Phillips J, Devito D: Management of fractures, in Morrissy RT., Weinstein SL (eds.): Lovell and Winter's Pediatric Orthopaedics, ed 5. Philadelphia, PA, Lippincott Williams & Wilkins, 2001, pp 1319-1422
- 02. 168.
- A 24-year-old man reports pain in the thumb at the metacarpophalangeal joint following a fall while skiing. Examination reveals significant swelling, ecchymosis, and a palpable mass on the u1nar side of the joint. Valgus stress testing of the joint reveals a 45-degree laxity in extension. Radiographs show no fractures. What is the next most appropriate step in management?
- 1- Ultrasound of the joint
- 2- MRI
- 3- Surgical repair of the u1nar collateral ligament
- 4- Primary fusion of the thumb metacarpophalangeal joint
- 5- Thumb spica cast for 6 weeks
- Question 02.168
- Answer = 3
- Reference(s)
- Heyman P, Gelberman RH Duncan K, Hipp JA: Injuries of the u1nar collateral ligament of the thumb metacarpophalangeal joint: Biornechanical and prospective clinical studies on the usefulness of valgus stress testing. Clin Orthop 1993;292:165-171
- 02. 169.
- A 40-year-old man who sustained multiple injuries in a motor vehicle accident is in class III hemorrhagic shock. Which of the following findings would most likely be present in this patient?
- 1- Normal mental status
- 2- Normal systolic blood pressure
- 3- Urine output of 15 mL/h
- 4- Respiratory rate of 20 breaths/min
- 5- Pulse rate of less than 100 beats/min
- Question 02.169
- Answer = 3
- Reference(s)
- Kellam JF Fischer TJ, Tornetta P III, Bosse MJ, Harris MB (eds.): Orthopaedic Knowledge Update: Trauma 2. Rosemont, IL, American Academy of Orthopaedic Surgeons, 2000, pp 229-237
- 02. 170.
- A 17-year-old snowboarder has had chronic lateral ankle pain after taking a hard fall 4 weeks ago. What is the most common cause of the continued pain?
- 1- Osteochondritis dissecans of the talus
- 2- Peroneal subluxation
- 3- An occult fracture of the fifth metatarsal
- 4- Fracture of lateral process of the tales
- 5- Stress fracture of the fibula
- Question 02.170
- Answer = 4
- Reference(s)
- Boon AJ, Smith J, Zobitz ME, Amrami KM: Snowboarder's talus fracture: Mechanism of injury. Am J Sports Med 2001;29:333-338.
- Bladin C, Giddings P, Robinson M: Australian snowboard injury data base study: A four-year prospective study. Am J Sports Med 1993;21:701-704.
- 02. 171.
- Figure 46a shows the AP radiograph of a 32-year-old man who was injured in a
- motorcycle accident. A closed reduction is performed promptly. The postreduction
- radiographs and CT scan are shown in Figures 46b and 46c. Definitive treatment should
- consist of
- 1- nonsurgical management.
- 2- arthroscopic excision of the fragment.
- 3- excision of the fragment through an anterior approach.
- 4- open reduction and internal fixation of the fracture.
- 5- percutaneous screw fixation of the fracture.
- 46b
- 46a
- 46c
- Question 02.171
- Answer = 4
- Reference(s)
- Swiontkowski MF Thorpe M, Seiler JG Hansen ST: Operative management of displaced femoral head fractures: Case-matched comparison of anterior versus posterior approaches for Pipkin I and Pipkin II fractures. J Orthop Trauma 1992;6:437-442.
- Stannard JP, Harris HW Volgas DA, Alonso JE: Functional outcome of patients with femoral head fractures associated with hip dislocations. Clin Orthop 2000;377:44-56
- 02. 172.
- What is the most likely cause of atraumatic entrapment of the u1nar nerve at Guyon's canal?
- 1- Ulnar artery thrombosis
- 2- Ganglion cyst
- 3- Anomalous muscle
- 4- Schwannoma
- 5- Osteophyte
- Question 02.172
- Answer = 2
- Reference(s)
- Richmond DA: Carpal ganglion with u1nar nerve compression. J Bone Joint Surg Br 1963;45:513-515.
- McDowell CL, Henceroth WD: Compression of the u1nar nerve in the hand by a ganglion: Report of a case. J Bone Joint Surg Am 1977;59:980.
- 02. 173.
- The wear pattern commonly observed on the polyethylene posts of cruciate-substituting tibia] components is generally the result of
- 1- the sterilization method of the polyethylene.
- 2- component rotational impingement.
- 3- use of a metal reinforced post.
- 4- proximal distal pistoning at heel strike.
- 5- patellar impaction during flexion.
- Question 02.173
- Answer = 2
- Reference(s)
- Mikulak SA Mahoney OM, dela Rosa MA, Schmalzried TP Loosening and osteolysis with the press-fit condylar posterior-cruciate-substituting total knee replacement. J Bone Joint Surg Am 2001;83:398-403.
- 02. 174.
- A 49-year-old woman has had severe pain in her right buttock and thigh and paresthesias into her right foot for the past 10 weeks. Lumbar epidural steroids have provided only temporary relief, physical therapy exacerbates her pain, and she feels that she is getting worse. Examination reveals mild weakness in her right extensor hallucis longus and a positive straight leg raising sign. Axial MRI scans are shown in Figure 47. Management should now consist of
- l- observation and reevaluation in 6 weeks.
- 2- acupuncture.
- 3- repeat epidural steroids under fluoroscopic guidance.
- 4- L4-5 diskectomy and interbody fusion with cage fixation.
- 5- L4-5 hemilaminotomy and diskectomy.
- Fig 47
- Question 02.174
- Answer = 5
- Reference(s)
- Wisneski RJ Garfin SR, Rothman RH Lutz GE: Lumbar disc disease, in Herkowitz HN Garfin SR,
- BalderstonRA, Eismont FJ, Bell GR., Wiesel SW (eds.): The Spine, ed 4. Philadelphia, PA, WBSaunders, 1999, pp 613-679.
- Wiesel SW, Lauerman WC: Adult spinal disorders: Spine trauma, in Wiesel SW, Delahay JN (eds.): Principles of Orthopaedic Medicine and Surgery. Philadelphia, PA, WB Saunders, 2001, pp 439-511
- 02.175.
- A 16-year-old girl was treated as an infant for torticollis. She has not been seen for follow-up since age 2 years. She is now completely asymptomatic but has returned for evaluation because her family recalls being told that she might need cervical fusion when she becomes a teenager. Figures 48a and 48b show cervical flexion-extension lateral radiographs. What is the most likely diagnosis'?
- 1- Normal cervical spine
- 2- Occipitoatlantal instability
- 3- Atlantoaxial instability
- 4- Klippel-Feil syndrome but no instability
- 5- Subaxial instability
- Figures 48a
- Figures 48b
- Question 02.175
- Answer = 4
- Reference(s)
- Loder RT: The cervical spine, in Morrissy RT, Weinstein SL (ed.): Lovell and Winter's Pediatric Orthopaedics, ed 5. Philadelphia, PA, Lippincott Williams & Wilkins, 2001, pp 799-840.
- Herring JA: Disorders of the neck, in Herring JA (ed): Tachdjian's Pediatric Orthopaedics, ed 3. Philadelphia, PA, WB Saunders, 2002, pp 171-212.
- Dietz F: Congenital abnormalities of the cervical spine, in Weinstein SL (ed): The Pediatric Spine: Principles and Practice, ed 2. Philadelphia, PA, Lippincott Williams & Wilkins, 2001, pp 239-249.
- Martinez-Lozano AG: Appendix D: Classifications, in Weinstein SL (ed): The Pediatric Spine: Principles and Practice, ed 2. Philadelphia, PA, Lippincott Williams & Wilkins, 2001, pp 1026-1027.
- Rouvreau P, Glorion C, Langlais J, Noury H, Pouliquen JC: Assessment and neurologic involvement of patients with cervical spine congenital synostosis as in Klippel-Feil syndrome: Study of 19 cases.
- 02. 176.
- A 17-year-old boy sustained an ipsilateral fracture of the femoral neck and shaft in a motor vehicle accident 4 years ago. He now reports a 6-month history of groin pain and limited active range of motion of the hip. A radiograph of the femur reveals the femoral shaft is well healed. A radiograph and a bone scan delayed image are shown in Figures 49a and 49b. What is the next most appropriate step in management?
- 1- Nonsteroidal anti-inflammatory drugs
- 2- IV antibiotics
- 3- Implant removal and vascularized bone graft
- 4- Implant removal and core decompression
- 5- Erthyrocyte sedimentation rate,
- C reactive protein, and hip aspiration
- Fig 49a
- Fig 49b
- Question 02.176
- Answer = 5
- Reference(s)
- Green SA Larson MJ, Moore TJ: Chronic sepsis following intramedullary nailing of femoral fractures. J Trauma 1987;27:52-57.
- Jenny JY Jenny G, Kempf I: Infection after reamed intramedullary nailing of lower limb fractures: A review of 1,464 cases over 15 years. Acta Orthop Scand 1994;65:94-96.
- Patzakis MJ, Wilkins J, Wiss DA: Infection following intramedullary nailing of long bones: Diagnosis and management. Clin Orthop 1986;212:182-191
- 02. 177.
- The sensitivity of color Doppler ultrasound to detect deep venous thrombosis in the lower extremity is
- 1- highest in the calf.
- 2- highest in the thigh.
- 3- highest in the pelvic veins.
- 4- comparable to venography.
- 5- operator independent.
- Question 02.177
- Answer = 2
- Reference(s)
- Salvati EA, Pellegrini VD Jr, Sharrock NE, et al: Recent advances in venous thromboembolic prophylaxis during and after total hip replacement. J Bone Joint Surg Am 2000;82:252-270.
- 02. 178.
- An 8-year-old boy with spastic herniparesis is referred for evaluation of a cavus foot. Examination reveals that the child walks on the lateral border of the affected foot. The hindfoot is inverted, and the forefoot is in equinus The involved extremity is 2 cm shorter than the contralateral side. What is the most likely cause of his foot deformity'?
- 1- Spasticity of the tibialis posterior
- 2- Spasticity of the extensor digitorum longus
- 3- Spasticity of the peroneus brevis and longus
- 4- Atrophy of the interossei and lumbricals
- 5- Weakness of the gastrocnernius and soleus
- Question 02.178
- Answer = 1
- Reference(s)
- Eilert RE: Cavus foot in cerebral palsy. Foot Ankle 1984;4:185-187.
- Shapiro F, Sprecht L: The diagnosis and orthopaedic treatment of childhood spinal muscular atrophy, peripheral neuropathy, Friedrelch ataxia, and arthrogyposis. J Bone Joint Surg Am 1993;75:1699-1714.
- Green NE, Griffin PP, Shiavi R: Split posterior tibial-tendon transfer in spastic cerebral palsy. J Bone Joint Surg Am 1983;65:748-754.
- 02. 179.
- A 34-year-old man sustains a type III acromioclavicular separation of his dominant shoulder. Initial management should consist of
- 1- reconstruction of the coracoclavicular ligament.
- 2- brief immobilization, followed by a gradual return to activity.
- 3- excision of the distal clavicle.
- 4- immobilization for 4 to 6 weeks, followed by aggressive rehabilitation.
- 5- a steroid injection.
- Question 02.179
- Answer = 2
- Reference(s)
- Clarke HD, McCann PD: Acromioclavicular joint injuries. Orthop Clin North Am 2000;31:177-187.
- Phillips AM, Smart C, Groom AF: Acrornioclavicular dislocation: Conservative or surgical therapy. Clin Orthop 1998;353:10-17
- 02. 180.
- A 13-year-old boy has had increasingly more pain in the right midfoot for the past year. He reports that the symptoms began insidiously, are activity related, and are relieved by rest. Examination reveals restriction of hindfoot inversion and eversion. A radiograph is shown in Figure 50. What is the most likely diagnosis?
- 1- Tarsal coalition
- 2- Calcaneal stress fracture
- 3- Juvenile arthritis
- 4- Occult ganglion
- 5- Fracture of the os trigonum
- Figure 50
- Question 02.180
- Answer = 1
- Reference(s)
- Lateur LM, Van Hoe LR, Van Ghillewe KV, Gryspeerdt SS, Baert AL, Dereymaeker GE: Subtalar coalition: Diagnosis with the C sign on lateral radiographs of the ankle. Radiology 1994;193:847-851.
- Sakellariou A, Sallomi D, Janzen DL, Munk PL, Claridge RJ Kiri VA: Talocalcaneal coalition: Diagnosis with the C-sign on lateral radiographs of the ankle. J Bone Joint Surg Br 2000;82:574-578.
- Mosca VS: The foot, in Morrissy RT, Weinstein SL (ed.): Lovell and Winter's Pediatric Orthopaedics, ed 5. Philadelphia, PA, Lippincott Williams & Wilkins, 2001, pp 1151-1216.
- 02. 181.
- A 42-year-old woman on dialysis has had an enlarging, firm, painless, soft-tissue mass of the left medial thigh for the past several years that has now begun to limit her ambulation A current frog-lateral radiograph and the same view obtained 4 years ago are shown in Figures 51a and 51b. Axial CT scans and a biopsy specimen are shown in Figures 51c through 51e. What is the most likely diagnosis?
- 1- Tumoral calcinosis
- 2- Secondary chondrosarcoma
- 3- Osteochondroma
- 4- Myositis ossificans
- 5- Extraskeletal osteosarcoma
- Figures 51a
- Figures 51b
- Figures 51c
- Figures 51d
- Figures 51e
- Question 02.181
- Answer = 1
- Reference(s)
- Cofan F, Garcia S, Combalia A, Campistol JM Oppenheimer F, Ramon R: Uremic tumoral calcinosis in patients receiving long-term hemodialysis therapy. J Rheumatol 1999;26:379-385.
- McCarthy EF Frassica FJ: Tumor-like lesions, in McCarthy EF Frassica FJ (ed.): Pathology of Bone and Joint Disorders with Clinical and Radiographic Correlation. Philadelphia, PA, WB Saunders, 1998, pp 291-293.
- 02. 182.
- Figure 52 shows the radiograph of a 2-year-old girl with a concentrically reduced hip. History reveals that she underwent Pavlik harness treatment for developmental dysplasia of the right hip during infancy. Despite the use of abduction bracing, she has persistent acetabular dysplasia. Management should now consist of
- I- continued abduction bracing.
- 2- varus osteotorny of the proximal femur.
- 3- Chiari osteotomy.
- 4- innominate osteotomy.
- 5- innominate osteotomy and varus
- osteotomy of the proximal femur.
- Figure 52
- Question 02.182
- Answer = 1
- Reference(s)
- Lindstrom JR, Ponseti IV, Wenger DR: Acetabular development after reduction in congenital dislocation of the hip. J Bone Joint Surg Am 1979;61:112-118.
- Weinstein SL: Developmental hip dysplasia, in Morrissy RT, Weinstein SL (ed.): Lovell and Winter's Pediatric Orthopaedics, ed 5. Philadelphia, PA, Lippincott Williams & Wilkins, 2001, pp 905-956
- 02. 183.
- A 50-year-old Asian woman comes to the emergency department reporting pain and swelling in her am after falling. She speaks some English but not fluently, and she does not have a primary care physician. Examination reveals a nondisplaced distal radius fracture which is placed in a cast. In addition to an orthopaedic follow-up appointment. what is the next most appropriate step in management?
- 1-Using an interpreter, refer her to a primary care physician, and explore the patient's
- preference for pain medication or other herbal treatment used at home.
- 2-Using an interpreter, refer her to a primary care physician and prescribe pain
- medication.
- 3-Using an interpreter, take a good history, explain your concerns, recommend a bone
- mass density test, and set up follow-up with a primary care physician.
- 4-Consult with the patient's family about the appropriateness of treatment
- recommendations.
- 5-Draw diagrams of the injury.
- Question 02.183
- Answer = 3
- Reference(s)
- National Institute of Health Consensus Development Conference Statement, March 27-29, 2000. Lau EM, Cooper C: The epidemiology of osteoporosis: The oriental perspective in a world context. Clin Orthop 1996;323:65-74.
- Kaiser Permanente National Diversity Council: A Provider's Handbook on Culturally Competent Care: Asian and Pacific Island Populations. Kaiser Permanente, 1999, p 16.
- Kaufert JM Putsch RW Communicating through interpreters in health care: Ethical dilemmas arising from differences in race, class, culture, language, and power. J Clin Ethics 1997;8:71-87.
- Downing B, Berg C: If you don't speak English. Cura 1991;21:9-14.
- 02. 184.
- A 60-year-old man reports atraumatic left shoulder pain that has been gradually. worsening over the past 6 months and is unrelated to activities. In the last month, he also notes numbness and tingling in his left ring and small fingers. He smokes two packs per day and has no other known medical problems. Examination reveals no motor weakness, a negative impingement sign, a negative Tinel sign at the cubital tunnel, and a negative elbow flexion test through 60 seconds. The next step in management should consist of
- 1- MRI of the shoulder.
- 2- MRI of the forearm.
- 3- electromyography.
- 4- cervical myelography.
- 5- a chest radiograph.
- Question 02.184
- Answer = 5
- Reference(s)
- Ziporyn T: Upper body pain: Possible tipoff to Pancoast tumor. JAMA 1981;246:1759, 1763.
- Spengler DM, Kirsh MM, Kaufer H: Orthopaedic aspects and early diagnosis of superior sulcustumor of lung (Pancoast). J Bone Joint Surg Am 1973;55:1645-1650.
- Bisbinas I, Langkamer VG: Pitfalls and delay in the diagnosis of Pancoast tumor presenting in orthopaedic units. Ann R Coll Surg Engl 1999;81:291-295.
- 02. 185.
- In the anterior (Henry) approach to the proximal radius, the dissection proceeds in the internervous plane between the brachioradialis and what other muscle:'
- 1- Supinator
- 2- Pronator teres
- 3- Flexor digitorum superficialis
- 4- Flexor pollicis longus
- 5- Flexor carpi u1naris
- Question 02.185
- Answer = 2
- Reference(s)
- Hoppenfeld S, Boer P (Eds): Surgical Exposures in Orthopaedics: The Anatomical Approach, ed 2. Philadelphia, PA, JB Lippincott, 1994, pp 117-131.
- Spinner M: Injuries to the Major Branches of Peripheral Nerves of the Forearm, ed 2. Philadelphia, PA, WB Saunders, 1978, pp 66-77.
- 02. 186.
- A 30-year-old female athlete who is 35 weeks pregnant notes partial giving way of her
- right knee with activities of daily living. She has continued to exercise regularly. Which
- of the following hormonal conditions is most likely related to her knee symptoms?
- I - Normal levels of progesterone and estradiol
- 2- Elevated level of progesterone
- 3- Elevated estradiol
- 4- Decreased progesterone
- 5- Decreased estradiol
- Question 02.186
- Answer = 3
- Reference(s)
- Charlton WP, Coslett-Charlton LM, Ciccotti MG: Correlation of estradiol in pregnancy and anterior cruciate ligament laxity. Clin Orthop 2001;387:165-170.
- Wojtys EM, Huston LJ, Lindenfeld TN, Hewett TE Greenfield ML: Association between the menstrual cycle and anterior cruciate ligament injuries in female athletes. Am J Sports Med 1998;26:614-619.
- 02. 187.
- A 12-year-old girl has idiopathic right genu valgum. A radiograph is shown in Figure 53. On the right, the mechanical axis passes through the lateral compartment of the knee. The skeletal age is 12 years. Management should consist of
- 1- physeal bar resection with interposition, bilaterally.
- 2- proximal medial tibial epiphyseal stapling.
- 3- proximal medial tibial epiphysiodesis.
- 4- observation.
- 5- a hip-knee-ankle-foot orthosis.
- Figure 53
- Question 02.187
- Answer = 2
- Reference(s)
- Stevens PM, Maguire M, Dales MD, Robins AJ: Physeal stapling for idiopathic genu valgum. JPediatr Orthop 1999;19:645-649.
- Schoenecker P: The lower extremity, in Morrissy RT., Weinstein SL eds Lovell and Winter's Pediatric Orthopaedics, ed 5. Philadelphia, PA, Lippincott Williams & Wilkins, 2001, pp 1059-1104.
- 02. 188.
- A 70-year-old woman has had worsening shoulder pain and limited motion for the past 2 years. Corticosteroid injections and physical therapy have failed to provide relief. Figures 54a through 54c show the plain radiographs and MRI scan. What is the next most appropriate step in treatment?
- I- Total shoulder arthroplasty
- 2- Humeral head replacement
- 3- Glenolmmeral arthrodesis
- 4- Rotator cuff repair
- 5- Arthroscopic acromioplasty
- Figures 54a
- Figures 54b
- Figures 54c
- Question 02.188
- Answer = 1
- Reference(s)
- Gartsman GM, Roddey TS, Hammerman SM.: Shoulder arthroplasty with or without resurfacing ofthe glenoid in patients who have osteoarthritis. J Bone Joint Surg Am 2000;82:26-34.
- Neer CS II, Watson KC Stanton FJ: Recent experience in total shoulder replacement. J Bone Joint Surg Am 1982;64:319-337.
- 02. 189.
- A 23-year-old man sustains a nondisplaced clavicle fracture and an ipsilateral scapular neck fracture with 1 cm of displacement. Definitive treatment should consist of
- 1- nonsurgical management.
- 2- open reduction and internal fixation of the clavicle.
- 3- open reduction and internal fixation of the scapula.
- 4- open reduction and internal fixation of the scapula and clavicle.
- 5- resection of the acrormoclavicular joint.
- Question 02.189
- Answer = 1
- Reference(s)
- Koval KJ (ed): Orthopaedic Knowledge Update 7. Rosemont, IL, American Academy of OrthopaedicSurgeons, 2002, pp 264-267.
- Edwards SG, Whittle AP, Wood GW II: Nonoperative treatment of ipsilateral fractures of the scapula and clavicle. J Bone Joint Surg Am 2000;82:774-780.
- 02. 190.
- In which of the following bones is the majority of length contributed by the proximal physic?
- 1- Humerus and tibia
- 2- Thumb metacarpal and radius
- 3- Femur and tibia
- 4- Femur and first metatarsal
- 5- Second metatarsal and humerus
- Question 02.190
- Answer = 1
- Reference(s)
- Ogden JA: Diagnostic imaging, in Ogden JA (ed): Skeletal Injury in the Child, ed 2. New York, NY, Springer-Verlag, 2000, pp 115-146.
- Herring JA: Growth and development, in Herring JA (ed): Tachdjian's Pediatric Orthopaedics, ed 3. Philadelphia, PA, WB Saunders, 2002, pp 3-22.
- Dimeglio A: Growth in pediatric orthopaedics, in Morrissy RT., Weinstein SL eds Lovell and
- Winter's Pediatric Orthopaedics, ed 5. Philadelphia, PA, Lippincott Williams & Wilkins, 2001, pp 33-62.
- 02. 191.
- What is the function of professional associations and licensing bodies?
- 1- Exist only to represent the interests of their members
- 2- Act to protect their members during disciplinary procedures
- 3- Advocate to government on behalf of members
- 4- Use collegiality as a means of establishing common goals and encouraging
- compliance with them
- 5- Review malpractice suits
- Question 02.191
- Answer = 4
- Reference(s)
- Cruess RL, Cruess SL, Johnston SE: Professionalism and medicine's social contract. J Bone Joint Surg Am 2000;82:1189-1194. Medical Professionalism in the New Millenium: A Physician Charter. Ann Intern Med 2002;136:243-246.
- 02.192.
- What structure is not directly involved in the regulation of calcium and phosphate metabolism?
- 1- Skin
- 2- Thyroid gland
- 3- Adrenal gland
- 4- Kidney
- 5- Liver
- Question 02.192
- Answer = 3
- Reference(s)
- Buckwalter JA, Einhorn TA, Simon SR eds Orthopaedic Basic Science: Biology and Biornechanicsof the Musculoskeletal System, ed 2. Rosemont, IL, American Academy of Orthopaedic Surgeons, 2000, pp 319-369.
- 02. 193.
- Which of the following nerves is primarily affected in the initial stages of Charcot-MarieTooth disease?
- 1- Posterior tibia]
- 2- Peroneal
- 3- Surat
- 4- Ulnar
- 5- Obturator
- Question 02.193
- Answer = 2
- Reference(s)
- Holmes JR, Hansen ST Jr: Foot and ankle manifestations of Charcot-Marie-Tooth disease. Foot Ankle1993;14:476-486.
- Alexander IA, Johnson KA: Assessment and management of pes cavus in Charcot-Marie-Tooth disease. Clin Orthop 1989;246:273-281.
- 02. 194.
- A 5-year-old boy injured his right elbow after falling during a camping trip 8 hours ago. Examination reveals that the elbow is swollen, and there is a 1-cm laceration over the medial side with minimal seepage of serosanguinous fluid. The neurovascular examination is intact. Radiographs show a completely displaced supracondylar fracture of the right humerus. Surgery should be scheduled immediately because of the
- I- presence of open fracture.
- 2- need for traction if reduction is delayed.
- 3- decreased ability to obtain a reduction.
- 4- increased incidence of nerve palsy after delayed reduction.
- 5- increased incidence of arterial injury after delayed reduction.
- Question 02.194
- Answer = 1
- Reference(s)
- Mehlman CT, Strub WM, Roy DR, Wall EJ, Crawford AH The effect of surgical timing on theperioperative complications of treatment of supracondylar humeral fractures in children. J BoneJoint Surg Am200l;83:323-327.
- Iyengar SR, Hoffinger SA Townsend DR: Early versus delayed reduction and pinning of typeIII displaced supracondylar fractures of the humerus in children: A comparative study. J Orthop Trauma 1999;13:51-55.
- 02. 195.
- A 25-year-old man sustains a closed midshaft femur fracture in a motorcycle accident. After insertion of an antegrade, statically locked femoral nail, intraoperative radiographs reveal an ipsilateral, nondisplaced femoral neck fracture. What is the most appropriate management?
- 1-Nonsurgical management of the femoral neck fracture
- 2-Stabilization of the femoral neck with lag screws anterior to the existing femoral nail
- 3-Removal of the nail and replacement with a long intramedullary hip screw
- 4-Removal of the nail and replacement with a reconstruction nail
- 5-Removal of the nail, stabilization of the femoral neck with lag screws, and retrograde
- nailing of the femoral shaft fracture
- Question 02.195
- Answer = 2
- Reference(s)
- Kellam JF Fischer TJ, Tornetta P III, Bosse MJ, Harris MB eds Orthopaedic Knowledge Update: Trauma 2. Rosemont, IL, American Academy of Orthopaedic Surgeons, 2000, pp 133-146.
- 02. 196.
- What is the most important factor when determining if surgery should be performed for a painful impending pathologic femur fracture in a patient with widely metastatic cancer?
- I- Projected length of survival
- 2- Difficulty of the planned procedure
- 3- Prior local radiation therapy
- 4- Age of the patient
- 5- Number of bony lesions
- Question 02.196
- Answer = 1
- Reference(s)
- Cancer Facts and Figures. Atlanta: American Cancer Society, 1999, pp 1-36. Frytak S, McLeod RA, Gunderson LL, Pritchard DJ, Unni KK Lung cancer, in Sim FH (ed): Diagnosisand Management of Metastatic Bone Disease: A Multidisciplinary Approach. New York, NY, Raven Press, 1988, pp 305-317.
- 02. 197.
- Figure 55 shows the serendipity view of a 17-year-old boy who was injured while playing football. Examination reveals that he has pain in the medial region of the right clavicle. There is no airway obstruction or circulatory embarrassment of the upper or lower extremities. Based on the radiographic findings, what is the most likely diagnosis?
- I- Clavicle shaft fracture
- 2- Distal clavicle fracture
- 3- Sternoclavicular posterior dislocation
- 4- Proximal clavicular physeal separation with
- posterior displacement
- 5- Congenital pseudarthrosis of the clavicle
- Question 02.197
- Answer = 4
- Reference(s)
- Denham R, Dingley A: Epiphyseal separation of the medial end of the clavicle. J Bone Joint Surg Am1967;49:1179.
- Price C, Phillips J, Devito D: Management of fractures, in Morrissy RT., Weinstein SL eds Lovell and Winter's Pediatric Orthopaedics, ed 5. Philadelphia, PA, Lippincott Williams & Wilkins, 2001, pp 1319-1422
- 02. 198.
- A 16-year-old female athlete reports recurrent lateral instability of the ankle that has failed to respond to physical therapy and time. History reveals that the initial injury was a medial subtalar dislocation. Stress radiographs are shown in Figures 56a through 56c. What is the most likely diagnosis?
- 1- Functional lateral ankle instability
- 2- Mechanical lateral ankle instability
- 3- Subtalar instability
- 4- Transverse tarsal instability
- 5- Syndesmotic instability
- Figures 56a
- Figures 56b
- Figures 56c
- Question 02.198
- Answer = 4
- Reference(s)
- Hooper G, McMaster MJ: Recurrent bilateral mid-tarsal subluxations: A case report. J Bone JointSurg Am 1979;61:617-619.
- Verhaar JA: Recurrent medial swivel dislocation of the foot. J Bone Joint Surg Br 1990;72:154-155.
- 02. 199.
- A young healthy patient sustains a severe crush injury resulting in a comminuted midshaft tibia fracture and an 8- x 10-cm medial soft-tissue defect with exposed bone. There is no associated vascular injury. Which of the following procedures will most predictably result in successful coverage
- 1- Split-thickness skin graft
- 2- Fasciocutaneousflap
- 3- Soleus flap
- 4- Gastrocnemius flap
- 5- Free-tissue transfer
- Question 02.199
- Answer = 5
- Reference(s)
- Pollak AN, McCarthy ML, Burgess AR: Short-term wound complications after application of flaps for coverage of traumatic soft-tissue defects about the tibia: The Lower Extremity Assessment Project (LEAP) Study Group. J Bone Joint Surg Am 2000;82:1681-1691.
- Browner BD, Jupiter JB Levine AM, Trafton PG eds Soft Tissue Coverage, ed 2. Philadelphia, PA, WB Saunders, 1998, pp 419-448.
- 02. 200.
- Current indications for a Bernese (Ganz) joint-preserving periacetabular osteotomy include
- 1- advanced osteoarthritis.
- 2- an open triradiate cartilage.
- 3- high subluxation of the femoral head.
- 4- symptomatic acetabular dysplasia in a young adult.
- 5- nonspherical congruity of the acetabulum and femoral head.
- Question 02.200
- Answer = 4
- Reference(s)
- Leunig M, Siebenrock KA, Ganz R: Rationale of periacetabular osteotomy and background work. Instr Course Lect 2001;50:229-238.
- 02. 201.
- A 1-week-old infant has prune belly syndrome. The spine shows no clinical deformity. The Barlow test and Ortolani maneuver are inconclusive. Both feet exhibit typical rigid clubfoot deformities. Further evaluation should include
- 1- flexion-extension radiographs of the cervical spine.
- 2- an AP radiograph of the pelvis.
- 3- bilateral hip ultrasonography.
- 4- MRI of the entire body.
- 5- simulated weight-bearing radiographs of both feet.
- Question 02.201
- Answer = 3
- Reference(s)
- Green NE, Lowery ER, Thomas R: Orthopaedic aspects of prune belly syndrome. J Pediatr Orthop 1993;13:496-501.
- Brinker MR, Palutsis RS, Sarwark JF The orthopaedic manifestations of prune-belly (Eagle-Barrett) syndrome. J Bone Joint Surg Am 1995;77:251-257.
- 02. 202.
- A study has been designed to identify factors that lead to a rare orthopaedic surgical complication. A group of patients with the complication has been identified, and the next step is to compare them to a matched group of patients without the complication. What is the name of this type of study?
- 1- Prospective cohort
- 2- Randomized clinical trial
- 3- Case series
- 4- Case control
- 5- Retrospective clinical series
- Question 02.202
- Answer = 4
- Reference(s)
- Buckwalter JA, Einhorn TA, Simon SR (eds.): Orthopaedic Basic Science: Biology and Biomechanics of the Musculoskeletal System, ed 2. Rosemont, IL, American Academy of Orthopaedic Surgeons, 2000, pp 2-17.
- Madison M: Planning the design phase of a clinical study. J Orthop Trauma 1997;11:63-67.
- 02. 203.
- A 10-year-old girl has had an enlarging painless mass in her arm. Axial T1- and T2- MRI scans are shown in Figures 57a and 57b. A biopsy specimen is shown in Figure 57c. Immunohistochemistry is positive for vimentin, desmin, and myosin but negative for CD99 cytokeratin, smooth muscle actin, and neural markers. What is the most likely diagnosis?
- 1- Extraskeletal Ewing's sarcoma
- 2- Leiomyosarcoma
- 3- Rhabdomyosarcoma
- 4- Synovial sarcoma
- 5- Malignant peripheral nerve sheath tumor
- Figures 57a
- Figures 57b
- Figures 57c
- Question 02.203
- Answer = 3
- Reference(s)
- Devoe K, Weidner N: Immunohistochemistry of small round-cell tumors. Semin Diagn Pathol2000;17 :216-224.
- Enzinger FM, Weiss SW: Rhabdomyosarcoma, in Soft-Tissue Tumors, ed 3. St Louis, MO, CV Mosby, 1995, p 539.
- Hays DM: Rhabdomyosarcoma. Clin Orthop 1993;289:36-49.
- 02. 204.
- An elderly patient reports the recent onset of difficulty with ambulation and urinary urgency. Examination reveals a positive Hoffmann's sign, u1nar drift of the ring and small fingers associated with active horizontal elevation of the upper extremities, and an extensor response of the toes bilaterally to plantar stimulation. These findings are consistent with
- 1- lumbar spinal stenosis.
- 2- peripheral vascular disease.
- 3- rheumatoid arthritis.
- 4- multiple sclerosis.
- 5- cervical myelopathy.
- Question 02.204
- Answer = 5
- Reference(s)
- Sherk HH Degenerative disorders, in Sherk HH (ed): The Cervical Spine, ed 2. Philadelphia, PA, JB Lippincott, 1989, pp 599-692.
- 02. 205.
- Gaucher's disease results in the abnormal accumulation of what material?
- 1- Homogentisic acid
- 2- Glucocerebrosides
- 3- Uric acid
- 4- Phosphoethanolamine
- 5- Calcium pyrophosphate
- Question 02.205
- Answer = 2
- Reference(s)
- Beutler E: Gaucher's disease. N Engl J Med 1991;325:1354-1360.
- 02. 206.
- A 10-year-old boy sustained a penetrating injury through his shoe to the plantar surface of his right foot while sliding into third base 1 week ago. He reports that the foot has become increasingly painful. Examination reveals that the foot is swollen, and the puncture wound is erythematous and has mild drainage that shows gram-negative rods and gram-positive cocci on Gram stain. Radiographs show a radiolucent area in the distal metaphysis of the third metatarsal. Management should consist of
- 1- splinting of the foot and oral antibiotics.
- 2- MRI and oral antibiotics.
- 3- hospital admission and IV antibiotics.
- 4- a bone scan and IV antibiotics.
- 5- open debridement and IV antibiotics.
- Question 02.206
- Answer = 5
- Reference(s)
- Niall DM, Murphy PG, Fogarty EE, Dowling FE, Moore DP: Puncture wound related pseudomonasinfections of the foot in children. Ir J Med Sci 1997;166:98-101.
- Laughlin TJ, Armstrong DG, Caporusso J, Lavery LA: Soft tissue and bone infections from puncturewounds in children. West J Med 1997;166:126-128.
- Haverstock BD, Grossman JP: Puncture wounds of the foot: Evaluation and treatment. Clin Podiatr Med Surg 1999;16:583-596.
- 02. 207.
- A 19-year-old man sustains an isolated, displaced transcervical femoral neck fracture in a motor vehicle accident. An urgent closed reduction attempt in the operating room results in residual varus angulation and 1 cm of displacement. Appropriate management should now consist of
- 1- in situ fixation.
- 2- bipolar herniarthroplasty.
- 3- total hip arthroplasty.
- 4- open reduction and internal fixation.
- 5- in situ fixation and bone grafting
- Question 02.207
- Answer = 4
- Reference(s)
- Swiontkowski MF Winquist RA, Hansen ST Jr: Fractures of the femoral neck in patients betweenthe ages of twelve and forty-nine years. J Bone Joint Surg Am 1984;66:837-846.
- Kellam JF Fischer TJ, Tornetta P III, Bosse MJ, Harris MB (eds.): Orthopaedic Knowledge Update: Trauma 2. Rosemont, IL, American Academy of Orthopaedic Surgeons, 2000, pp 115-124.
- 02. 208.
- A 17-year-old long-distance runner has pain in the posteromedial distal tibia that is present when he begins his run, decreases in intensity during the run, but returns toward the end of the run and persists. He has recently increased his training. Examination reveals tenderness along the posteromedial distal tibia that extends toward the knee. What is the most likely diagnosis?
- 1- Achilles tendinititis
- 2- Tibial stress fracture
- 3- Exertional compartment syndrome
- 4- Medial tibial stress syndrome
- 5- Popliteal artery entrapment syndrome
- Question 02.208
- Answer = 4
- Reference(s)
- Touliopolous S, Hershman EB Lower leg pain: Diagnosis and treatment of compartment syndromesand other pain syndromes of the leg. Sports Med 1999;27:193-204.
- Bennett JE, Reinking MF Pluemer B, Pentel A, Seaton M, Killian C: Factors contributing to thedevelopment of medial tibial stress syndrome in high school runners. J Orthop Sports Phys Ther 2001;31:504-510.
- 02. 209.
- Which of the following structures is primarily responsible for the tensile strength and elasticity of a peripheral nerve?
- 1- Axon
- 2- Perineurium
- 3- Endoneurium
- 4- Internal epineurium
- 5- External epineuriurn
- Question 02.209
- Answer = 2
- Reference(s)
- Haftek J: Stretch injury of peripheral nerve: Acute effects of stretching on rabbit nerve. J Bone JointSurg Br 1970;52:354-365.
- Lundborg G: Nerve Injury and Repair. Edinburgh, Churchill Livingstone, 1988, p 33.
- 02. 210. Most patients with osteomalacia will have which of the following findings?
- I- Hypocalcemia, hypophosphatemia, and a normal hemoglobin level
- 2- Hypocalcemia, euphosphatemia, and a normal hemoglobin level
- 3- Hypocalcemia, hypophosphatemia, and anemia
- 4- Hypocalcemia, hyperphosphatemia, and anemia
- 5- Hypercalcemia, hyperphosphatemia, and a normal hemoglobin level
- Question 02.210
- Answer = 1
- Reference(s)
- Delahay JN, Evans BG Metabolic bone disease, in Wiesel SW, Delahay JN (eds.): Principles of Orthopaedic Medicine and Surgery. Philadelphia, PA, WB Saunders, 2001, pp 198-229.
- 02. 211.
- A 15-month-old female infant who was born with an unstable hip is currently asymptomatic. History reveals that treatment consisted of abduction splinting for 6 weeks, followed by ultrasound that showed stabilization of the hip. All subsequent examinations have shown symmetric hip abduction. A current radiograph is shown in Figure 58. What is the most likely diagnosis?
- 1- Legg-Calve-Perthes disease
- 2- Osteonecrosis
- 3- Septic arthritis
- 4- Juvenile arthritis
- 5- Epiphyseal dysplasia
- Figure 58
- Question 02.211
- Answer = 2
- Reference(s)
- Weinstein SL: Developmental hip dysplasia and dislocation, in Morrissy RT., Weinstein SL (eds.):
- Lovell and Winter's Pediatric Orthopaedics, ed 5. Philadelphia, PA, Lippincott Williams & Wilkins, 2001, pp 905-956.
- Herring JA: Developmental dysplasia of the hip, in Herring JA (ed): Tachdjian's Pediatric Orthopaedics, ed 3. Philadelphia, PA, WB Saunders, 2002, pp 513-564.
- Salter RB Kostuik J, Dallas S: Avascular necrosis of the femoral head as a complication of treatment for congenital dislocation of the hip in young children: A clinical and experimental investigation. Can J Surg 1969;12:44-61.
- 02. 212.
- A 64-year-old man has right knee pain that limits his activities of daily living. Nonsurgical management has failed to provide relief. Radiographs are shown in Figures 59a and 59b. Treatment should consist of
- 1- fully constrained total knee arthroplasty.
- 2- posterior cruciate-substituting total knee arthroplasty.
- 3- posterior cruciiate-retaining total knee arthroplasty with a conforming tibial insert.
- 4- anteroposterior translating mobile-bearing total knee arthroplasty.
- 5- unicompartmental arthroplasty.
- Figures 59a
- Figures 59b
- Question 02.212
- Answer = 2
- Reference(s)
- Paletta GA Jr, Laskin RS: Total knee arthroplasty after a previous patellectomy. J Bone Joint SurgAm 1995;77:1708-1712.
- 02. 213.
- A 32-year-old patient has a painful thigh mass that changes in size with activity. Figure 60a shows a T1-weighted MRI scan, and Figure 60b shows a fat suppression T2-weighted MRI scan. Radiographs are normal. What is the most likely diagnosis?
- 1- Ganglion
- 2- Angiosarcoma
- 3- Synovial sarcoma
- 4- Lipoma
- 5- Hemangioma
- Figure 60b
- Figure 60a
- Question 02.213
- Answer = 5
- Reference(s)
- Suh JS, Hwang G, Hahn SB Soft tissue hemangiomas: MR manifestations in 23 patients. Skeletal Radiol 1994;23:621-625.
- 02. 214.
- An 8-year-old girl has had left arm pain for the past 6 weeks. A plain radiograph, axial T,- and T2-weighted MRI scans, and a biopsy specimen are shown in Figures 61a through 61d. Based on these findings, the next most appropriate step in management should consist of
- 1- observation with serial radiographs.
- 2- IV antibiotics.
- 3- curettage and bone grafting.
- 4- wide excision only.
- 5- wide excision with chemotherapy.
- Figures 61a
- Figures 61b
- Figures 61c
- Figures 61d
- Question 02.214
- Answer = 2
- Reference(s)
- McCarthy EF Frassica FJ: Infections of bone and joints, in McCarthy EF Frassica FJ eds Pathology of Bone and Joint Disorders with Clinical and Radiographic Correlation. Philadelphia, PA, WB Saunders, 1998, pp 153-164.
- 02. 215.
- Figure 62 shows a lateral dissection of the left elbow. The black pin labels the superficial branch of the radial nerve. What muscle is labeled by the red pin?
- 1- Extensor carpi radialis brevis
- 2- Supinator
- 3- Pronator teres
- 4- Flexor digitorum superficialis
- 5- Brachialis (lacertus fibrosis)
- Figure 62
- Question 02.215
- Answer = 2
- Reference(s)
- Hoppenfeld S, deBoer P: Surgical Exposures in Orthopaedics: The Anatomical Approach, ed 2. Philadelphia, PA, JB Lippincott, 1994, pp 23-117.
- 02. 216.
- A 13-year-old boy has a unilateral slipped capital femoral epiphysis that has been symptomatic for the past 3 months. The affected hip would be expected to show which of the following morphologic changes in the acetabulum?
- 1- No abnormalities
- 2- Decreased volume
- 3- Protrusio acetabulum
- 4- Increased anterior version
- 5- Increased vertical inclination
- Question 02.216
- Answer = 1
- Reference(s)
- Kordelle J, Richolt JA, Millis M, Jolesz FA, Kikinis R: Development of the acetabulum in patients withslipped capital femoral epiphysis: A three-dimensional analysis based on computed tomography. JPediatr Orthop 2001;21:174-178.
- Goodman DA, Feighan JE, Smith AD, Latimer B, Buly RL Cooperman DR: Subclinical slipped capital femoral epiphysis: Relationship to osteoarthrosis of the hip. J Bone Joint Surg Am 1997;79:1489-1497.
- 02. 217.
- A 32-year-old man has a displaced radial head fracture with an associated posterolateral elbow dislocation. Examination reveals pain and swelling throughout the elbow, forearm, and wrist. In addition to closed reduction of the elbow dislocation, management should include
- 1- a long arm cast.
- 2- a hinged elbow brace.
- 3- an external fixator with articulated elbow hinge.
- 4- radial head resection.
- 5- open reduction and internal fixation of the radial head.
- Question 02.217
- Answer = 5
- Reference(s)
- Popovic N, Gillet P, Rodriguez A, Lemaire R: Fracture of the radial head with associated elbowdislocation: Results of treatment using a floating radial head prosthesis. J Orthop Trauma 2000;14:171-177.
- Cohen MS, Hastings H II Acute elbow dislocation: Evaluation and management. J Am Acad Orthop Surg 1998;6:15-23.
- Frankie MA, Koval KJ, Sanders FW (et al): Partial head fractures associated with elbow dislocations treated by immediate stabilization and early motion. J Shoulder Elbow 1999;8:308-311.
- Koval KJ (ed): Orthopaedic Knowledge Update 7. Rosemont, IL, American Academy of Orthopaedic Surgeons, 2002, pp 308-311.
- 02. 218.
- What factor is associated with catastrophic failure of ceramic femoral heads?
- 1- Horizontal cup placement
- 2- Large femoral heads (greater than 32 mm)
- 3- Skirted femoral heads
- 4- Lubrication properties inferior to those of metal heads
- 5- Patient weight
- Question 02.218
- Answer = 3
- Reference(s)
- Skinner HB Ceramic bearing surfaces. Clin Orthop 1999;369:83-91.
- Jazrawi LM, Kummer FJ, Di Cesare PE: Alternative bearing surfaces for total joint arthroplasty. J Am Acad Orthop Surg 1998;6:198-203.
- 02. 219.
- The two most common manifestations of reactive arthritis (Reiter syndrome) are polyarthritis and urethritis (or cervicitis). What is the next most common manifestation?
- 1- Chronic bronchitis
- 2- Aortitis
- 3- Balanitis
- 4- Diarrhea
- 5- Eye disease
- Question 02.219
- Answer = 5
- Reference(s)
- Clauw DJ, Petzke F: Rheumatic disorders, in Wiesel SW, Delahay JN eds Principals of OrthopaedicMedicine and Surgery. Philadelphia, PA, WB Saunders, 2001, pp 231-263.
- Booth RE, Simpson JM Herkowitz HN Arthritis of the spine, in Herkowitz HN Garfin SR, Balderston
- RA, Eismont FJ, Bell GR., Wiesel SW eds The Spine, ed 4. Philadelphia, PA, WB Saunders, 1999, pp 429-453.
- 02. 220.
- A 24-year-old man sustains a displaced transcervical femoral neck fracture and is treated with multiple cannulated screws. Five months after the injury, the patient reports continued hip pain and is unable to bear weight. Sequential radiographs and a CT scan reveal 10° of varus angulation. Management should now consist of
- 1- continued observation with weight bearing.
- 2- valgus intertrochanteric osteotomy
- 3- bipolar hemiarthroplasty.
- 4- electrical stimulation.
- 5- cancellous autologous bone grafting to the femoral neck.
- Question 02.220
- Answer = 2
- Reference(s)
- Marti RK Schuller HM, Raaymakers EL: Intertrochanteric osteotomy for non-union of the femoralneck. J Bone Joint Surg Br 1989;71:782-787.
- Kellam JF Fischer TJ, Tornetta P III, Bosse MJ, Harris MB eds Orthopaedic Knowledge Update: Trauma 2. Rosemont, IL, American Academy of Orthopaedic Surgeons, 2000, pp 115-124.
- 02. 221.
- Which of the following structures is the primary restraint to inferior translation of the glenohumeral joint when the arm is adducted?
- 1- Superior glenobumeral ligament
- 2- Inferior glenohumeral ligament
- 3- Middle glenohumeral ligament
- 4- Supraspinatus tendon
- 5- Long head of the biceps tendon
- Question 02.221
- Answer = 1
- Reference(s)
- Harryman DT II Sidles JA, Harris SL, Matsen FA III: The role of the rotator interval capsule inpassive motion and stability of the shoulder. J Bone Joint Surg Am 1992;74:53-66.
- Warner JJ, Deng XH Warren RF Torzilli PA: Static capsuloligamentous restraints to superior- inferior translation of the glenohumeral joint. Am J Sports Med 1992;20:675-685.
- 02. 222.
- A 14-year-old girl sustained an injury to the ankle region in a fall while snowboarding. An axial CT scan is shown in Figure 63. Which of the following ligaments is attached to the fracture fragment?
- 1- Deltoid
- 2- Anterior tibiofibular
- 3- Anterior talofibular
- 4- Posterior tibiofibular
- 5- Posterior talofibular
- Figure 63
- Question 02.222
- Answer = 2
- Reference(s)
- Crawford AH Fractures and dislocations of the foot and ankle, in Green DP, Swiontkowski MFeds Skeletal Trauma in Children. Philadelphia, PA, WB Saunders, 1994, pp 463-466.
- Nunley JA: Fractures and fracture-dislocations of the ankle, in Coughlin MJ, Mann RA eds Surgery of the Foot and Ankle, ed 7. St Louis, MO, Mosby Publishers, 1999, p 1407.
- 02. 223.
- An 11-year-old girl reports the insidious onset of medial midfoot pain that has been slowly worsening for the past 2 months. She describes the pain as a nagging ache that is exacerbated by weight-bearing activity. Laboratory studies show a peripheral WBC of 7,500/mm3 (normal 3,500 to 10,500/mm3) and an erythrocyte sedimentation rate of 34 mm/h (normal 0 to 20 mm/h) Radiographs are shown in Figures 64a and 64b. What is the most likely diagnosis?
- 1- Juvenile arthritis
- 2- Charcot arthropathy
- 3- Subacute osteomyelitis
- 4- Osteosarcoma
- 5- Unicameral cyst
- Figures 64a
- Figures 64a
- Question 02.223
- Answer = 3
- Reference(s)
- Ezra E, Wientroub S: Primary subacute haematogenous osteomyelitis of the tarsal bones in children. J Bone Joint Surg Br 1997;79:983-986.
- Herring JA: Bone and joint infections, in Herring JA (ed): Tachdjian's Pediatric Orthopaedics, ed 3. Philadelphia, PA, WB Saunders, 2002, pp 1841-1878.
- Morrissy RT: Bone and joint sepsis, in Morrissy RT, Weinstein SL (eds.): Lovell and Winter's Pediatric Orthopaedics, ed 5. Philadelphia, PA, Lippincott Williams & Wilkins, 2001, pp 459-507.
- Hamdy RC, Lawton L, Carey T, Wiley J, Marton D: Subacute hematogenous osteomyelitis: Are biopsyand surgery always indicated? J Pediatr Orthop 1996;16:220-223.
- Reinehr T, Burk G, Michel E, Andler W: Chronic osteomyelitis in childhood: Is surgery always indicated? Infection 2000;28:282-286.
- 02. 224.
- An 8-year-old girl has had pain radiating down her right lower extremity. Examination reveals a mass in her right buttock. A plain radiograph, axial proton-density MRI scan, and a biopsy specimen are shown in Figures 65a through 65c. Cytogenetics studies reveal an 11 to 22 translocation. What is the most likely diagnosis`?
- 1- Extraskeletal Ewing's sarcoma
- 2- Leiomyosarcoma
- 3- Rhabdomyosarcoma
- 4- Synovial sarcoma
- 5- Metastatic neuroblastoma
- Figures 65a
- Figures 65b
- Figures 65c
- Question 02.224
- Answer = 1
- Reference(s)
- Devoe K, Weidner N: Immunohistochernistry of small round-cell tumors. Semin Diagn Pathol2000;17:216-224.
- Ahmad R, Mayol BR, Davis M, Rougraff BT: Extraskeletal Ewing's sarcoma. Cancer 1999;85:725-731.
- 02. 225.
- In addition to the sinus tarsi artery, the arterial supply to the talar body comes from which of the following structures?
- 1- Lateral plantar artery and deltoid artery
- 2- Artery of the tarsal canal and recurrent plantar artery
- 3- Artery of the tarsal canal and lateral plantar artery
- 4- Artery of the tarsal canal and deltoid artery
- 5- Recurrent plantar artery and deltoid artery
- 66 * American Academy of Orthopaedic Surgeons
- Question 02.225
- Answer = 4
- Reference(s)
- Kellam JF, Fischer TJ, Tornetta P III, Bosse MJ, Harris MB (eds.): Orthopaedic Knowledge Update: Trauma 2. Rosemont, IL, American Academy of Orthopaedic Surgeons, 2000, pp 203-225.
- 02. 226.
- A 6-year-old girl sustains a displaced fracture of the radial neck. Closed reduction under anesthesia results in a 20-degree residual angulation. The next step in management should consist of
- 1- a long arm splint.
- 2- open reduction and crossed Kirschner wire fixation.
- 3- open reduction and transcapitellar Kirschner wire fixation.
- 4- open reduction and Herbert screw fixation.
- 5- radial head excision.
- Question 02.226
- Answer = 1
- Reference(s)
- Vocke AK, Van Laer L: Displaced fractures of the radial neck in children: Long-term results andprognosis of conservative treatment. J Pediatr Orthop Br 1998;7:217-222.
- Sessa S, Lascombes P, Prevot J, Gagneux E: Fractures of the radial head and associated elbow injuries in children. J Pediatr Orthop Br 1996;5:200-209.
- 02. 227.
- A 42-year-old factory worker who has worked as a finish grinder on metal castings for the past 11 years reports persistent numbness of his fingers. Examination and diagnostic work-up will most likely reveal which of the following findings?
- 1- Cold intolerance
- 2- Diminished ulnar artery flow noted on Allen's test
- 3- Abnormal motor and sensory latencies
- 4- Increased vibration and temperature thresholds
- 5- Hyperhidrosis (increased palmar sweating)
- Question 02.227
- Answer = 4
- Reference(s)
- Stromberg T, Dahlin LB, Lundborg G: Hand problems in 100 vibration-exposed symptomatic maleworkers. J Hand Surg Br 1996;21:315-319.
- Rosen I, Stromberg T, Lundborg G: Neurophysiological investigation of hands damaged byvibration: Comparison with idiopathic carpal tunnel syndrome. Scand J Plast Reconstr Surg Hand Surg 1993;27:209-216.
- 02. 228.
- What muscle is considered most important in thumb opposition?
- 1- Abductor pollicis brevis
- 2- Abductor pollicis longus
- 3- Flexor pollicis longus
- 4- Opponens pollicis
- 5- Superficial head of the flexor pollicis brevis
- Question 02.228
- Answer = 1
- Reference(s)
- Buckwalter JA, Einhorn TA, Simon SR (eds.): Orthopaedic Basic Science: Biology and Biomechanics of the Musculoskeletal System, ed 2. Rosemont, IL, American Academy of Orthopaedic Surgeons, 2000, pp 731-827.
- 02. 229.
- Preconditioning an anterior cruciate ligament graft can reduce the amount of stress relaxation by as much as
- 1- 10%.
- 2- 20%.
- 3- 50%.
- 4- 75%.
- 5- 90%.
- Question 02.229
- Answer = 3
- Reference(s)
- Buckwalter JA, Einhorn TA, Simon SR (eds.): Orthopaedic Basic Science: Biology and Biomechanicsof the Musculoskeletal System, ed 2. Rosemont, IL, American Academy of Orthopaedic Surgeons,2000, pp 5 81-616.
- Graf BK Vanderby R Jr, Ulm MJ, Rogalski RP, Thielke RJ Effect of preconditioning on the viscoelastic response of primate patellar tendon. Arthroscopy 1994;10:90-96.
- 02. 230.
- Achondroplasia is caused by a deletion or alteration in the gene encoding which of the following proteins?
- 1- Type II collagen
- 2- Type IX collagen
- 3- Cartilage oligomeric matrix protein
- 4- Parathyroid hormone receptor
- 5- Fibroblast growth factor receptor
- Question 02.230
- Answer = 5
- Reference(s)
- Buckwalter JA, Einhorn TA, Simon SR (eds.): Orthopaedic Basic Science: Biology and Biomechanicsof the Musculoskeletal System, ed 2. Rosemont, IL, American Academy of Orthopaedic Surgeons, 2000, pp 19-76.
- Baitner AC, Maurer SG, Gruen MB, Di Cesare PE: The genetic basis of the osteochondrodysplasias. J Pediatr Orthop 2000;20:594-605.
- 02. 231.
- An 8-year-old boy with L3 myelomeningocele has a painless, progressive left thoracolumbar scoliosis that measures 50°. Sagittal alignment of the spine is normal. Following successful release of the tethered spinal cord, the scoliosis deformity will most likely show
- 1- partial resolution.
- 2- gradual but complete resolution.
- 3- immediate and complete resolution.
- 4- continued progression.
- 5- no additional progression.
- Question 02.231
- Answer = 4
- Reference(s)
- Pierz K, Banta J, Thomson J, Gahm N, Hartford J: The effect of tethered cord release on scoliosis in myelomeningocele. J Pediatr Orthop 2000;20:362-365.
- Hendrick EB Hoffman HJ Humphreys RP: The tethered spinal cord. Clin Neurosurg 1983;30:457-463.
- 02. 232.
- A 75-year-old man undergoes an uneventful cemented total knee arthroplasty. The total tourniquet time was 145 minutes. In the recovery room, examination reveals that the foot is cold and no pulses are palpable. Removal of the postoperative dressing results in no change. There is no swelling in the thigh, calf, or popliteal fossa. What is the most likely cause of this problem?
- I- Vascular spasm because of the excess tourniquet time
- 2- Overlengthening of the knee
- 3- Acute deep venous thrombosis
- 4- Arterial occlusion from an embolized atherosclerotic plaque
- 5- Compartment syndrome of the deep posterior compartment
- Question 02.232
- Answer = 4
- Reference(s)
- Ayers DC, Dennis DA, Johanson NA, Pellegrim VD: Complications of total knee arthroplasty. J Bone Joint Surg Am 1977;79:278-311.
- 02. 233.
- The "shaken baby syndrome" describes the association between multiple fractures in the long bones of abused infants and
- 1- intestinal perforation.
- 2- cardiac contusion.
- 3- splenic rupture.
- 4- renal avulsion.
- 5- chronic subdural hematoma
- Question 02.233
- Answer = 5
- Reference(s)
- Caffey J: Multiple fractures in the long bones of infants suffering from chronic subdural hematoma.Am J Radiol 1946;56:163-173.
- Akbarnia BA, Campbell RM: The role of the orthopaedic surgeon in child abuse, in Morrissy RT,
- Weinstein SL (eds.): Lovell and Winter's Pediatric Orthopaedics, ed 5. Philadelphia, PA, Lippincott Williams & Wilkins, 2001, pp 1423-1445.
- Herring JA: General principles of managing orthopaedic injuries, in Herring JA (ed): Tachdjian's Pediatric Orthopaedics, ed 3. Philadelphia, PA, WB Saunders, 2002, pp 2059-2086.
- 02. 234.
- A 9-year-old boy has activity-related pain at the medial joint line of his knee. He denies locking or effusion. A radiograph is shown in Figure 66. Management should consist of
- 1- arthroscopic partial meniscectomy.
- 2- excisional biopsy.
- 3- osteochondral graft.
- 4- activity restrictions.
- 5- periosteal transplantation.
- Figure 66
- Question 02.234
- Answer = 4
- Reference(s)
- Ogden JA: Knee, in Ogden JA (ed): Skeletal Injury in the Child, ed 2. New York, NY, Springer- Verlag, 2000, pp 929-990. Herring JA: Disorders of the knee, in Herring JA (ed): Tachdjian's Pediatric Orthopaedics, ed 3. Philadelphia, PA, WB Saunders, 2002, pp 789-838.
- Sales de Gauzy J, Mansat C, Darodes PH, Cahuzac JP: Natural course of osteochondritis dissecans in children. J Pediatr Orthop Br 1999;8:26-28.
- Hefti F, Beguiristain J, Krauspe R, et al: Osteochondritis dissecans: A multicenter study of the European Pediatric Orthopedic Society. J Pediatr Orthop Br 1999;8:231-245.
- 02. 235.
- Achondroplasia, hypochondroplasia, and thanatophoric dysplasia are three phenotypes of dwarfism caused by autosomal-dominant mutations resulting in abnormal levels of
- 1- type I collagen.
- 2- type 11 collagen.
- 3- fibroblast growth factor receptor 3.
- 4- fibrillin 1.
- 5- cartilage oligometric matrix protein.
- Question 02.235
- Answer = 3
- Reference(s)
- Baitner AC, Maurer SG, Gruen MB, Di Cesare PE: The genetic basis of the osteochondrodysplasias.J Pediatr Orthop 2000;20:594-605.
- Dietz FR, Mathews KD Update on the genetic bases of disorders with orthopaeadic manifestations. J Bone Joint Surg Am 1996;78:1583-1598.
- 02. 236.
- An 18-year-old man underwent an open shoulder capsular shift for recurrent instability. Postoperatively, the patient had new onset weakness of forearm supination and elbow flexion. Nerve conduction velocity studies performed 3 weeks later show a normal evoked response distally. An electromyograrn shows fibrillations and positive sharp waves. What is the most likely diagnosis?
- 1- Neurapraxia of the axillary nerve
- 2- Neurapraxia of the musculocutaneous nerve
- 3- Axonotmesis of the axillary nerve
- 4- Axonotmesis of the musculocutaneous nerve
- 5- Neurotmesis of the musculocutaneous nerve
- Question 02.236
- Answer = 4
- Reference(s)
- Richards RR, Hudson AR, Bertoia JT Urbaniak JR, Waddell JP: Injury to the brachial plexusuring Putti-Platt and Bristow procedures: A report of eight cases. Am J Sports Med1987;15:374-380.
- Roberts CS, Seligson D, Gleis G: Diagnosis and treatment of complications, in Browner BD, Jupiter
- JB, Trafton PG, Levine AM (eds.): Skeletal Trauma, ed 3. Philadelphia, PA, WB Saunders, 2002 (in press).
- Buckwalter JA, Einhorn TA, Simon SR (eds.): Orthopaedic Basic Science: Biology and Biomechanics of the Musculoskeletal System, ed 2. Rosemont, IL, American Academy of Orthopaedic Surgeons, 2000, pp 618-682.
- 02. 237.
- A 35-year-old woman reports a history of aching pain in her forearm that gradually resolved over the past week, but now she is unable to actively flex the interphalangeal joint of her thumb. Examination reveals absent flexor pollicis longus (FPL) function and hyperextension of the distal interphalangeal joint of the index finger with attempts at pinch. The best course of action should be
- 1- repair of the FPL tendon.
- 2- observation.
- 3- obtaining a fasting blood glucose level.
- 4- exploration of the median nerve.
- 5- MRI of the forearm.
- Question 02.237
- Answer = 2
- Reference(s)
- Miller-Breslow A, Terrono A, Millender LH Nonoperative treatment of anterior interosseous nerveparalysis. J Hand Surg Am 1990;15:493-496.
- Goulding PJ, Schady W: Favourable outcome in non-traumatic anterior interosseous nerve lesions. J Neurol 1993;240:83-86.
- 02. 238.
- A 15-year-old boy with spastic diplegic cerebral palsy has a bunion that is painful during shoe wear and ambulation. The hallux valgus deformity is not passively correctable, and the great toe is overlapped by the second toe. There is no pain with passive motion of the metatarsophalangeal joint of the great toe. An AP radiograph of the forefoot is shown in Figure 67. Treatment should consist of
- 1-Silastic arthroplasty of the first metatarsophalangeal joint.
- 2-chevron osteotomy of the first metatarsal neck.
- 3-adductor hallucis tenotomy with osteotomy of the base of the first metatarsal.
- 4-excision of the prominent bunion with medial capsulorrhaphy.
- 5- arthrodesis of the first metatarsophalangeal joint.
- Figure 67
- Question 02.238
- Answer = 5
- Reference(s)
- Davids JR, Mason TA, Danko A, Banks D, Blackhurst D: Surgical management of hallux valgusdeformity in children with cerebral palsy. J Pediatr Orthop 2001;21:89-94.
- Jenter M, Lipton GE, Miller F: Operative treatment for hallux valgus in children with cerebral palsy. Foot Ankle Int 1998;19:830-835.
- 02. 239.
- A 36-year-old man who is a C6 quadriplegic is scheduled to undergo surgery for a broken hip. Because he is completely insensate, he is given no anesthesia for the surgery. During the procedure, monitoring reveals hypertension (200/120 mm Hg), facial flushing, and he reports a headache. What is the most likely cause of these findings?
- 1- Hypoxia
- 2- Fluid overload
- 3- Autonomic dysreflexia
- 4- Psychogenic factors
- 5- Orthostatic positioning
- Question 02.239
- Answer = 3
- Reference(s)
- Frymoyer JW Ducker TB, Hadler NM, Kostuik JP, Weinstein IN, Whitecloud TS III (eds.): The Adult Spine: Principles and Practice. Philadelphia, PA, Lippincott-Raven, 1997, pp 713-714.
- 02. 240.
- A 27-year-old man with a traumatic transtibial amputation reports buckling of his knee in the stance phase. What is the most likely cause of his symptoms?
- 1- Ineffective suspension
- 2- Prosthetic foot too soft
- 3- Prosthetic foot placed too far forward
- 4- Prosthetic foot placed too far back
- 5- Excessive prosthetic length
- Question 02.240
- Answer = 4
- Reference(s)
- Transtibial amputation, in Bowker J, Michael J (eds.): Atlas of Limb Prosthetics. St Louis, MO, Mosby-Year Book, 1992, pp 429-479.
- Beaty JH (ed): Orthopaedic Knowledge Update 6. Rosemont, IL, American Academy of Orthopaedic Surgeons, 1999, pp 139-149.
- 02. 241.
- A 13-year-old girl with idiopathic adolescent scoliosis underwent uneventful posterior spinal instrumentation with titanium rods from T4 to L4. No intraoperative complications or abnormalities were noted during somatosensory-evoked potential monitoring. Motor function in the upper and lower extremities was normal on awakening, but within 24 hours of surgery, rapid-onset paraplegia developed. Blood pressure is normal. What is the next most appropriate step in management?
- I- Consultation with psychiatry to rule out conversion reaction
- 2- Needle aspiration of the suspected wound hematoma
- 3- MRI of the thoracolumbar spine
- 4- Electromyography with nerve conduction velocity studies
- 5- Immediate removal of spinal instrumentation and administration of corticosteroids
- answer
- back
- Question 02.241
- Answer = 5
- back to this question
- next question
- Reference(s)
- Wenger DR, Mubarak SJ Leach J: Managing complications of posterior spinal instrumentation and fusion. Clin Orthop 1992;284:24-33.
- Winter RB Lonstein JE: Juvenile and adolescent scoliosis, in Rothman RH Simeone FA eds The Spine. Philadelphia, PA, WB Saunders, 1975, pp 325-372.
- 02. 242.
- Malreduction of radial shaft fractures in adults affects outcome predominately by causing
- I - restricted flexion and extension of the elbow.
- 2- restricted flexion and extension of the wrist.
- 3- restricted forearm rotation.
- 4- distal radioulnar joint dysfunction.
- 5- cosmetic deformity
- answer
- back
- Question 02.242
- Answer = 3
- back to this question
- next question
- Reference(s)
- Kellam JF, Fischer TJ, Tornetta P III, Bosse MJ, Harris MB eds Orthopaedic Knowledge Update: Trauma 2. Rosemont, IL, American Academy of Orthopaedic Surgeons, 2000, pp 53-63.
- Schemitsch EH, Richards RR: The effect of malunion on functional outcome after plate fixation of fractures of both bones of the forearm in adults. J Bone Joint Surg Am 1992;74:1068-1078.
- 02. 243.
- The tibial insert shown in Figure 68 was removed after 10 years in vivo. Failure was
- most likely the result of
- l- knee instability.
- 2- infection.
- 3- malalignment.
- 4- third body abrasive wear.
- 5- the sterilization method.
- answer
- back
- Question 02.243
- Answer = 5
- back to this question
- next question
- Reference(s)
- Collier JP, Mayor MB, McNamara JL Surprenant VA, Jensen RE: Analysis of the failure of 122 polyethylene inserts from uncemented tibial knee components. Clin Orthop 1991;273:232-242.
- 02. 244.
- A 38-year-old man with acquired immunodeficiency syndrome has had gradually
- progressive left groin pain for the past 2 years. A plain radiograph, a biopsy specimen,
- and results of an acid-fast stain are shown in Figures 69a through 69c. Management
- should consist of
- 1- observation.
- 2- antimicrobial chemotherapy.
- 3- synovectomy with curettage and
- bone grafting of the cysts.
- 4- total hip arthroplasty.
- 5- internal hemipelvectomy.
- answer
- back
- Figures 69a
- Figures 69b
- Figures 69c
- Question 02.244
- Answer = 2
- back to this question
- next question
- Reference(s)
- Sequeira W, Co H, Block JA: Osteoarticular tuberculosis: Current diagnosis and treatment. Am J Ther 2000;7:393-398.
- Silber JS, Whitfield SB Anbari K, Vergillio J, Gannon F, Fitzgerald RH Jr: Insidious destruction of the hip by Mycobacterium tuberculosis and why early diagnosis is critical. J Arthroplasty 2000;15 :392-397.
- Caparros AB, Sousa M, Ribera Zabalbeascoa J, Uceda Carrascosa P, Moya Corral F: Total hip arthroplasty for tuberculosis coxitis. Int Orthop 1999;23:348-350.
- 02. 245.
- The 2000 Declaration of Helsinki, as adopted by the World Health Organization, states
- that research protocols defining the efficacy or safety of a new drug or treatment protocol should use human control groups who receive
- 1- no treatment.
- 2- placebo treatment.
- 3- the best current treatment.
- 4- competitive new treatments.
- 5- historical treatment protocols.
- answer
- back
- Question 02.245
- Answer = 3
- back to this question
- next question
- Reference(s)
- Burstein AH Swiontkowski MF Update on human rights issues in clinical research. J Bone Joint Surg Am 2001;83:161-163.
- 02. 246.
- An 18-month-old infant has bilateral metaphyseal corner fractures of the distal femoral metaphyses of unknown origin. Following a skeletal survey, the next step in management should consist of
- 1- notification of child protective services.
- 2- bilateral long leg casts and discharge.
- 3- bilateral percutaneous pinning, long leg casts, and discharge.
- 4- Bryant's traction.
- 5- modified Bryant's traction.
- answer
- back
- Question 02.246
- Answer = 1
- back to this question
- next question
- Reference(s)
- Loder RT., Bookout C: Fracture patterns in battered children. J Orthop Trauma 1991;5:428-433.
- Akbarnia B, Campbell RM.: The role of the orthopaedic surgeon in child abuse, in Morrissy RT.,
- Weinstein SL eds Lovell and Winter's Pediatric Orthopaedics, ed 5. Philadelphia, PA, Lippincott
- Williams & Wilkins, 2001, pp 1423-1445 .
- 02. 247.
- A 16-year-old female sw mmer has had right shoulder pain for the past 6 months.
- Ultrasound treatment and rest have failed to provide relief. Examination reveals bilateral 2+ glenohumeral translation, hyperextensible elbows, and a positive impingement sign. Management should now consist of
- 1- corticosteroid injection.
- 2- scapular muscle and rotator cuff strengthening.
- 3- arthroscopic capsular shift.
- 4- arthroscopic thermal capsulorrhaphy
- 5- arthroscopic acromioplasty.
- answer
- back
- Question 02.247
- Answer = 2
- back to this question
- next question
- Reference(s)
- Burkhead WZ Jr, Rockwood CA Jr: Treatment of instability of the shoulder with an exercise program.J Bone Joint Surg Am 1992;74:890-896.
- Schenk TJ, Brems JJ: Multidirectional instability of the shoulder: Pathophysiology, diagnosis, and management. J Am Acad Orthop Surg 1998;6:65-72 .
- 02. 248.
- Following tibial flizarov lengthening in animals. histologic examination of the anterior tibial artery typically shows a significant increase in the
- 1- number of vasa vasorum.
- 2- thickness of the intimal wall.
- 3- diameter of the collagen fibers in the vessel wall.
- 4- diameter of the elastic fibers in the vessel wall.
- 5- inflammatory response to tearing of the intima.
- answer
- back
- Question 02.248
- Answer = 1
- back to this question
- next question
- Reference(s)
- Fink B, Singer J, Braunstein S, Schwinger G, Schmielau G, Ruther W: Behavior of blood vessels during lower-leg lengthening using the Ilizarov method. J Pediatr Orthop 1999;19:748-753.
- Ippolito E, Peretti G, Bellocci M, et al: Histology and ultrastructure of arteries, veins, and peripheral nerves during limb lengthening. Clin Orthop 1994;308:54-62.
- 02. 249.
- A patient who underwent an endoscopic plantar fascia release 3 months ago now reports pain in the medial arch with weight bearing and notes progressive flattening of the foot. What is the most likely explanation for this problem?
- 1- Recurrence of the calcaneal heel spur
- 2- Surgical injury to the medial plantar nerve
- 3- Excessive release of the plantar fascia
- 4- Injury to the posterior tibial tendon
- 5- Injury to the spring (calcaneonavicular) ligament
- answer
- back
- Question 02.249
- Answer = 3
- back to this question
- next question
- Reference(s)
- Palumbo RC, Kodrois SA Baxter DE: Endoscopic plantar fasciotomy: Indications, techniques, andcomplications. Sports Med Arthroscopy Rev 1994;2:317-322.
- Daly PJ, Kitaoka HB Chao EY Plantar fasciotomy for intractable plantar fasciltis: Clinical results and biomechanical evaluation. Foot Ankle 1992;13:188-195.
- Kitaoka HB Luo ZP An KN.: Mechanical behavior of the foot and ankle after plantar fascia release in the unstable foot. Foot Ankle Int 1997;18:8-15.
- 02. 250.
- Figure 70 shows the radiograph of an 80-year-old woman who reports the recent
- development of incapacitating left knee pain that renders her unable to walk. She has
- been advised to undergo left total knee arthroplasty. The patient is at increased risk for
- which of the following surgical or postoperative complications?
- 1- Tibial component loosening
- 2- Knee dislocation
- 3- Increased polyethylene wear
- 4- Peroneal nerve palsy
- 5- Infection
- answer
- back
- Figure 70
- Question 02.250
- Answer = 4
- back to this question
- next question
- Reference(s)
- Johanson NA: Neurovascular complications following total knee replacement: Causes, treatment, and prevention. Instr Course Lect 1997;46:181-184.
- Easley ME, Insall JN, Scuderi GR., Bullek DD: Primary constrained condylar knee arthroplasty for the arthritic valgus knee. Clin Orthop 2000;380:58-64.
- 02. 251.
- When a cylindrical bone is subjected to a torsional load, the maximum tensile load is
- generated in a plane that is
- 1- 30° to the long axis.
- 2- 45° to the long axis.
- 3- 60° to the long axis.
- 4- parallel (0°) to the long axis.
- 5- perpendicular (90°) to the long axis.
- answer
- back
- Question 02.251
- Answer = 2
- back to this question
- next question
- Reference(s)
- Buckwalter JA, Einhorn TA, Simon SR (eds.): Orthopaedic Basic Science: Biology and Biomechanicsof the Musculoskeletal System, ed 2. Rosemont, IL, American Academy of Orthopaedic Surgeons, 2000, pp 134-180.
- 02. 252.
- An 8-year-old girl with arthrogryposis multiplex congenita has a unilateral left knee
- flexion contracture despite undergoing posterior knee capsulotomy and hamstring
- releases. The arc of active knee motion measures from 40° to 80° of flexion. If a distal
- femoral extension osteotomy is performed, what would be the most likely outcome in 4
- years?
- 1- Improved knee arc of motion
- 2- Knee ankylosis
- 3- Patella alta
- 4- Recurrence of the knee flexion deformity
- 5- Anterior subluxation of the tiobiofemoral joint
- answer
- back
- Question 02.252
- Answer = 4
- back to this question
- next question
- Reference(s)
- DelBello DA, Watts HG: Distal femoral extension osteotomy for knee flexion contracture in patients with arthrogryposis. J Pediatr Orthop 1996;16:122-126.
- Sodergard J, Ryoppy S: The knee in arthrogryposis multiplex congenita. J Pediatr Orthop 1990;10:177-182.
- 02.253.
- The distinguishing morphologic characteristic of the myofibroblast, in contrast to the
- fibroblast and smooth muscle cell, is the presence of
- 1- prominent intracellular bundles of microfilaments.
- 2- poorly developed secretory organelles.
- 3- a well-developed enveloping basal lamina.
- 4- a rounded, smooth nucleus.
- 5- extracellular appendages.
- answer
- back
- Question 02.253
- Answer = 1
- back to this question
- next question
- Reference(s)
- Tomasek JJ, Vaughan MB, Haaksma CJ: Cellular structure and biology of Dupuytren's disease. Hand Clin 1999;15:21-34.
- 02. 254.
- Which of the following is considered a fundamental advantage when using the extended slide trochanteric osteotomy for revision total hip arthroplasty?
- 1-Only a small wafer of bone is removed from the greater trochanter.
- 2-The sciatic nerve is more readily exposed and protected.
- 3-The gluteus medius muscle is detached, allowing distal advancement of the greater trochanter.
- 4-The greater trochanter may be reflected proximally, facilitating exposure of the acetabulum.
- 5-The vastus lateralis muscle remains attached.
- answer
- back
- Question 02.254
- Answer = 5
- back to this question
- next question
- Reference(s)
- Chen WM, McAuley JP, Engh CA Jr, Hopper RH Jr, Engh CA: Extended slide trochanteric osteotomy for revision total hip arthroplasty. J Bone Joint Surg Am 2000;82:1215-1219.
- 02. 255.
- A 6-year-old boy sustains an uncomplicated supracondylar fracture of the distal humerus. The fracture heals in 20° of varus malalignment. What is the most frequent problem associated with this outcome?
- 1- Early ulnar-humeral arthritis
- 2- Significant loss of elbow function
- 3- Cosmetic appearance
- 4- Tardy ulnar nerve palsy
- 5- Humeral length discrepancy in excess of 2 cm
- answer
- back
- Question 02.255
- Answer = 3
- back to this question
- next question
- Reference(s)
- Wilkins KE, Beaty JH, Chambers HG, Toniolo RM: Fractures and dislocations of the elbow region, in
- Rockwood CA Jr, Wilkins KE, Beaty JH (eds.): Fractures in Children, ed 4. Philadelphia, PA,Lippincott-Raven, 1996, pp 653-904.
- Herring JA: Upper extremity injuries, in Herring JA (ed): Tachdjian's Pediatric Orthopaedics, ed 3. Philadelphia, PA, WB Saunders, 2002, pp 2115-2250 .
- 02. 256.
- A patient is to receive hydrocodone/APAP postoperatively, 1 to 2 tablets orally every 3 to 4 hours as needed. After 2 weeks, studies show that the patient's laboratory values are abnormal. What would be the expected abnormality secondary to taking this medication?
- 1- An elevated erythrocyte sedimentation rate
- 2- An elevated liver enzymes profile (LFTs)
- 3- An elevated renal profile (BUN creatinine)
- 4- Anemia
- 5- Leukocytosis
- answer
- back
- Question 02.256
- Answer = 2
- back to this question
- next question
- Reference(s)
- Frymoyer J, Ducker TB, Hadler NM, Kostuik JP, Weinstein JN, Whitecloud TS III (eds.): The Adult Spine: Principles and Practice. Philadelphia, PA, Lippincott-Raven, 1997, pp 278-279.
- 02. 257.
- What is the most likely cause of the lesion shown in Figure 71
- 1- Delamination and fatigue wear
- 2- Adhesive and abrasive wear
- 3- Backside wear
- 4- Creep
- 5- Dissociation of the liner locking mechanism
- answer
- back
- Figure 71
- Question 02.257
- Answer = 2
- back to this question
- next question
- Reference(s)
- McKellop HA, Campbell P, Park SH, et al: The origin of submicron polyethylene wear debris in total hip arthroplasty. Clin Orthop 1995;311:3-20.
- 02. 258.
- Figure 72 shows the planar technetium Tc 99m bone scan of a 14-year-old boy who has
- had localized back pain for the past 6 weeks. Pain is reported with hyperextension of thelumbar spine. The neurologic examination and radiographs of the lumbar spine are
- normal. What is the most likely diagnosis?
- 1- Herniated nucleus pulposus at L4-L5
- 2- Herniated nucleus pulposus at L5-S1
- 3- Spondylolysis at L5
- 4- Hemangioma
- 5- Tuberous sclerosis
- answer
- back
- Figure 72
- Question 02.258
- Answer = 3
- back to this question
- next question
- Reference(s)
- Anderson K, Sarwark JF, Conway JJ, Logue ES, Schafer MF Quantitative assessment with SPELTimaging of stress injuries of the pars interarticularis and response to bracing. J Pediatr Orthop2000;20:28-33.
- Lonstein J: Spondylolysis and spondylolisthesis, in Morrissy RT., Weinstein SL (eds.): Lovell and Winter's Pediatric Orthopaedics, ed 5. Philadelphia, PA, Lippincott Williams & Wilkins, 2001, pp 777-798.
- 02. 259.
- Which of the following findings is considered predictive of poor wound healing potential following amputation?
- I- Serum albumin level of 4.0 g/dL
- 2- Absolute lymphocyte count of 1,250/mm3
- 3- Hemoglobin level of 10.5 g/dL
- 4- Ischemic index of 0.7 at the surgical site
- 5- Transcutaneous po2 of 35 mm Hg of the surgical site
- answer
- back
- Question 02.259
- Answer = 2
- back to this question
- next question
- Reference(s)
- Beaty JH (ed): Orthopaedic Knowledge Update 6. Rosemont, IL, American Academy of Orthopaedic Surgeons, 1999, pp 139-146.
- Dickhaut SC, DeLee JC Page CP.: Nutritional status: Importance in predicting wound-healing after amputation. J Bone Joint Surg Am 1984;66:71-75.
- 02. 260.
- A 40-year-old woman has had pain and swelling in the metatarsophalangeal (MTP) joint of the second toe for the past 5 months. Dorsal-plantar stress of the MTP joint
- reproduces the pain. The remainder of the foot examination and radiographs is normal.
- Shoe modification, steroid injections, and anti-inflammatory drugs have failed to provide relief. The next most appropriate step in management should consist of
- 1- arthrodesis of the second MTP joint.
- 2- MTP synovectomy and reconstruction of the MTP joint capsule.
- 3- dorsiflexion osteotomy of the second metatarsal.
- 4- resection of the second metatarsal head (resection arthroplasty).
- 5- resection of the base of the proximal phalanx.
- answer
- back
- Question 02.260
- Answer = 2
- back to this question
- next question
- Reference(s)
- Coughlin MJ, Mann RA: Lessor toe deformities, in Coughlin MJ, Mann RA (eds.): Surgery of the Fool
- and Ankle, ed 7. St Louis, MO, Mosby Publishers, 1999, pp 320-391.
- Mizel MD, Miller RA, Scioli MW (eds.): Orthopaedic Knowledge Update: Foot and Ankle 2.
- Rosemont, IL, American Academy of Orthopaedic Surgeons, 1998, pp 163-166.
- Fortin PT, Myerson MS: Second metatarsophalangeal joint instability. Foot Ankle Int 1995;16:306-313.
- 02. 261.
- A 35-year-old man has pain and is unable to move his right arm after falling in a
- mountain bike accident. Passive motion reveals 60° of forward flexion and 10 degrees ofabduction. External rotation is not possible because of pain. An AP radiograph of the
- shoulder is interpreted as normal with no fractures. What is the next most appropriate
- step in management?
- I- MRI
- 2- CT
- 3- Axillary radiograph
- 4- Arthrography
- 5- Electromyography and nerve conduction velocity studies
- answer
- back
- Question 02.261
- Answer = 3
- back to this question
- next question
- Reference(s)
- Hawkins RJ Neer CS II, Pianta RM, Mendoza FX Locked posterior dislocation of the shoulder. JBone Joint Surg Am 1987;69:9-18.
- Oakes DA, McAllister DR: An atypical appearance of a posterior dislocation of the shoulder with a fracture of the proximal humerus. J Shoulder Elbow Surg 2001;10:182-185.
- 02. 262.
- What type of displaced unstable posterior pelvic ring injury will benefit most from open reduction and internal fixation?
- 1- Transforaminal sacral fracture
- 2- Sacral alar fracture
- 3- Iliac fracture
- 4- Sacroiliac fracture-dislocation
- 5- Sacroiliac dislocation
- answer
- back
- Question 02.262
- Answer = 5
- back to this question
- next question
- Reference(s)
- Starr AJ: Long-term functional prognosis of posterior injuries in high-energy pelvic disruption. J Orthop Trauma 1998;12:592-593.
- Koval KJ (ed): Orthopaedic Knowledge Update 7. Rosemont, IL, American Academy of Orthopaedic Surgeons, 2002, pp 395-403.
- 02. 263.
- An asymptomatic 12-year-old girl with developmental dysplasia of the hip underwent open reduction at age 2 years. The current radiograph shown in Figure 73 reveals
- 1- a normal hip.
- 2- slipped capital femoral epiphysis.
- 3- Legg- Calve-Perthes disease.
- 4- dysplasia secondary to partial lateral growth arrest.
- 5- early osteoarthritis.
- answer
- back
- Figure 73
- Question 02.263
- Answer = 4
- back to this question
- next question
- Reference(s)
- Kim HW Morcuende JA, Dolan LA, Weinstein SL: Acetabular development in developmentaldysplasia of the hip complicated by lateral growth disturbance of the capital femoral epiphysis. JBone Joint Surg Am 2000;82:1692-1700.
- Weinstein SL: Developmental hip dysplasia and dislocation, in Morrissy RT, Weinstein SL (eds.): Lovell and Winter's Pediatric Orthopaedics, ed 5. Philadelphia, PA, Lippincott Williams & Wilkins,2001, pp 905-956.
- Herring JA: Developmental dysplasia of the hip, in Herring JA (ed): Tachdjian's Pediatric Orthopaedics, ed 3. Philadelphia, PA, WB Saunders, 2002, pp 513-654.
- Keret D, MacEwen GD Growth disturbance of the proximal part of the femur after treatment for congenital dislocation of the hip. J Bone Joint Surg Am 1991;73:410-423.
- Campbell P, Tarlow SD: Lateral tethering of the proximal femoral physis complicating the treatment of congenital hip dysplasia. J Pediatr Orthop 1990;10:6-8.
- 02. 264.
- While performing a revision total knee arthroplasty with a trial component in place, the surgeon notes that the knee has full extension but is loose in flexion. What is the next step in managing this situation?
- 1- Resect more distal femur and use a thicker polyethylene insert.
- 2- Use a smaller femoral component.
- 3- Use a more constrained polyethylene insert.
- 4- Cut more posterior slope on the tibia.
- 5- Release the posterior capsule.
- answer
- back
- Question 02.264
- Answer = 1
- back to this question
- next question
- Reference(s)
- Callaghan JJ, Dennis DA, Paprosky WG Rosenberg AG (eds.): Orthopaedic Knowledge Update: Hipand Knee Reconstruction. Rosemont, IL, American Academy of Orthopaedic Surgeons, 1995,pp 323-327.
- Hoeffel DP, Rubash HE: Revision total knee arthroplasty: Current rationale and techniques for femoralcomponent revision. Clin Orthop 2000;380:116-132.
- Rand JA: Planning for revision total knee arthroplasty. Instr Course Lect 1999;48:161-166.
- 02. 265.
- A 21-year-old woman who was involved in a motorcycle accident sustains the isolated injury shown in Figures 74a and 74b. Examination reveals that the skin is intact, and there is no evidence of compartment syndrome. Neurovascular examination of the foot is normal. Management should consist of
- I- a long leg cast.
- 2- half-pin external fixation.
- 3- thin wire external fixation.
- 4- plate osteosynthesis.
- 5- intramedullary nailing.
- answer
- back
- Figures 74a
- Figures 74b
- Question 02.265
- Answer = 1
- back to this question
- next question
- Reference(s)
- Sarrmiento A, Latta LL: Functional fracture bracing. J Am Acad Orthop Surg 1999;7:66-75.
- Sarmiento A, Sharpe FE, Ebramzadeh E, Normand P, Shankwiler J: Factors influencing the outcome of closed tibial fractures treated with functional bracing. Clin Orthop 1995;315:8-24.
- 02. 266.
- Osteosarcoma is more common in patients with what other malignancy?
- 1- Ewing's sarcoma
- 2- Rhabdomyosarcoma
- 3- Wilms' tumor
- 4- Neuroblastoma
- 5- Retinoblastoma
- answer
- back
- Question 02.266
- Answer = 5
- back to this question
- next question
- Reference(s)
- Buckwalter JA, Einhorn TA, Simon SR (eds.): Orthopaedic Basic Science: Biology and Biomechanics of the Musculoskeletal System, ed 2. Rosemont, IL, American Academy of Orthopaedic Surgeons,2000, pp 427-441.
- 02. 267.
- A 35-year-old woman who sustained a distal radius fracture 3 months ago is now diagnosed with complex regional pain syndrome. In addition to appropriate medical management, therapy should include
- 1- dynamic splinting.
- 2- static progressive splinting.
- 3- aggressive passive range-of-motion exercises.
- 4- aggressive resistive strengthening exercises.
- 5- gentle pain-free active range-of-motion exercises.
- answer
- back
- Question 02.267
- Answer = 5
- back to this question
- next question
- Reference(s)
- Lankford LL: Reflex sympathetic dystrophy, in Hunter JM Schneider HL Mackin EJ, Callahan AD (eds.): Rehabilitation of the Hand: Surgery and Therapy, ed 3. St Louis, MO, CV Mosby, 1990,pp 763-784.
- Poplawski ZJ Wiley AM, Murray JF Post-traumatic dystrophy of the extremities. J Bone Joint SurgAm 1983;65:642-655.
- Schutzer SF, Gossling HR: The treatment of reflex sympathetic dystrophy syndrome. J Bone Joint Surg Am 1984;66:625-629.
- 02. 268.
- With upper extremity amputations, use of a myoelectric prosthesis is most successful for what level?
- 1- Midhumerus
- 2- Midforearm
- 3- Wrist disarticulation
- 4- Shoulder disarticulation
- 5- Elbow' disarticulation
- answer
- back
- Question 02.268
- Answer = 2
- back to this question
- next question
- Reference(s)
- Kritter AE: Myoelectric prostheses. J Bone Joint Surg Am 1985;67:654-657.
- Gaine WJ Smart C, Bransby-Zachary M: Upper limb traumatic amputees: Review of prosthetic use. J Hand Surg Br 1997;22:73-76.
- 02. 269.
- What is the classification of the bone defect shown in Figure 75?
- 1- Segmental
- 2- Combined segmental and cavitary
- 3- Mal alignment/malunion
- 4- Ectasia (cavitary)
- 5- Stenosis
- answer
- back
- Figure 75
- Question 02.269
- Answer = 4
- back to this question
- next question
- Reference(s)
- Maurer SG, Baitner AC, Di Cesare PE: Reconstruction of the failed femoral component and proximal femoral bone loss in revision hip surgery. J Am Acad Orthop Surg 2000;8:354-363.
- Haddad FS, Masri BA, Garbuz DS, Duncan CP.: Femoral bone loss in total hip arthroplasty: Classification and preoperative planning. Instr Course Lect 2000;49:83-96.
- 02. 270.
- An otherwise asymptomatic 16-year-old girl has the rash shown in Figure 76. History
- reveals that she was vacationing in the woods of the northeastern United States in the
- preceding week. Examination reveals no adenopathy. What organism is the most likely
- etiologic agent?
- 1- Bartonella henselae
- 2- Borrelia burgdorferi
- 3- Mycobacterium tuberculosis
- 4- Sporothrix schenckii
- 5- Zygomycetes rhizopus
- answer
- back
- Figure 76
- Question 02.270
- Answer = 2
- back to this question
- next question
- Reference(s)
- Steere AC: Lyme disease. New Engl J Med 2001;345:115-125.
- 02. 271.
- What qualitative term is used to describe the ability of a material to absorb energy before failure?
- 1- Toughness
- 2- Stress
- 3- Ultimate strength
- 4- Strain
- 5- Elastic modulus
- answer
- back
- Question 02.271
- Answer = 1
- back to this question
- next question
- Reference(s)
- Buckwalter JA, Einhorn TA, Simon SR (eds.): Orthopaedic Basic Science: Biology and Biomechanics of the Musculoskeletal System, ed 2. Rosemont, IL, American Academy of Orthopaedic Surgeons,2000, pp 181-215.
- Currey JD: The design of mineralized hard tissues for their mechanical functions. J Exp. Biol. 1999;202:3285-3294.
- 02. 272.
- The rationale for using a larger diameter femoral head in total hip arthroplasty is a lower
- 1- incidence of dislocation.
- 2- incidence of polyethylene component dissociation from the metal shell.
- 3- manufacturing cost.
- 4- rate of volumetric polyethylene wear.
- 5- rate of acetabular component loosening.
- answer
- back
- Question 02.272
- Answer = 1
- back to this question
- next question
- Reference(s)
- Bartz RL Nobel PC, Kadakia NR, Tullos HS: The effect of femoral component head size on posterior dislocation of the artificial hip joint. J Bone Joint Surg Am 2000;82:1300-1307.
- 02. 273.
- A 19-year-old man has hand pain after striking a wall with his clenched fist.
- Examination reveals swelling and tenderness at the dorsum of the hand. AP and lateral
- radiographs do not show any obvious pathology. Which of the following imaging studies should be ordered next?
- 1- CT
- 2- 30-degree pronated view
- 3- 45-degree supinated view
- 4- Carpal tunnel view
- 5- Clenched fist PA view
- answer
- back
- Question 02.273
- Answer = 2
- back to this question
- next question
- Reference(s)
- Bora FW Jr, Didizian NH: The treatment of injuries to the carpometacarpal joint of the little finger. J Bone Joint Surg Am 1974;56:1459-1463.
- Towles JG Dislocations and fracture-dislocations at the carpometacarpal joint of the fingers. Hand Clin 1988;4:103.
- 02. 274.
- A patient is scheduled to undergo a total hip arthroplasty for arthritis following a markedly displaced acetabular fracture that has healed. What is the advantage of having treated the fracture with open reduction and internal fixation compared to nonsurgical management?
- 1- Decreased need for bone grafting
- 2- Decreased instability
- 3- Less intraoperative blood loss
- 4- Less surgical time
- 5- Higher hip scores postoperatively
- answer
- back
- Question 02.274
- Answer = 1
- back to this question
- next question
- Reference(s)
- Bellabarba C, Berger RA, Bentley CD, et al: Cementless acetabular reconstruction after acetabular fracture. J Bone Joint Surg Am 2001;83:868-876.
- 02. 275.
- To optimize recovery, management of acute back pain should consist of bed rest for what period of time?
- 1- No more than 3 days, followed by a return to normal activities as soon as possible
- 2- Two to 7 days, followed by restricted activity for 2 to 3 weeks
- 3- At least 7 days with no additional restrictions
- 4- Two weeks, followed by a gradual return to normal activities
- 5- None under any circumstance
- answer
- back
- Question 02.275
- Answer = 1
- back to this question
- next question
- Reference(s)
- Deyo RA, Diehl AK, Rosenthal M: How many days of bed rest for acute low back pain? A randomized clinical trial. N Engl J Med 1986;315:1064-1070 .
for more mcqs in the bank
for more mcqs in the bank
ORTHOPEDIC MCQS ONLINE BANK OITE 20 |
ORTHOPEDIC MCQS ONLINE BANK OITE 97 |
ORTHOPEDIC MCQS BANK OITE 96 |
ORTHOPEDIC MCQS BANK OITE 99 |
ORTHOPEDIC MCQS BANK OITE98 |
ORTHOPEDIC MCQS ONLINE OB PATHOLOGY 1A |
ORTHOPEDIC MCQS ONLINE OB HAND 1A |
Links For. ORTHOPEDICS MCQS QUESTION BANK |
ortho mcqs |
ORTHOPEDIC MCQS ONLINE OB 20 RECONSTRUCTION 1D |
ORTHOPEDIC MCQS ADULT RECONSTRUCTIVE OB 20 1C |
ORTHOPEDIC MCQS ONLINE OB 20 2B RECONSTRUCTION |
ORTHOPEDIC MCQS ONLINE OB 20 RECONSTRUCTION 1A |
ORTHOPEDIC MCQS ONLINE OB 20 TRAUMA 2D |
ORTHOPEDIC MCQS ONLINE OB 20 TRAUMA 2B |
ORTHOPEDIC MCQS ONLINE 20 OB TRAUMA 2A |
ORTHOPEDIC MCQS ONLINE 20 OB TRAUMA 1D |
ORTHOPEDIC MCQS 20 OB TRAUMA 1C |
ORTHOPEDIC MCQS 20 OB TRAUMA 1B |
ORTHOPEDIC MCQS 20OB TRAUMA 1A |
ORTHOPEDIC MCQS ONLINE 010 PEDIATRIC |
ORTHOPEDIC MCQS 010 ONLINE |
ORTHOPEDIC MCQS 010 Adult Reconstructive Surgery... |
ORTHOPEDIC MCQS ONLINE 011 RECONSTRUCTION |
ORTHOPEDIC MCQS ONLINE 011 PATHOLOGY |
ORTHOPEDIC MCQS 011 ANATOMY IMAGING |
ORTHOPEDIC MCQS O11 UPPER EXTREMITY |
ORTHOPEDIC MCQS ONLINE 012 SPINE |
ORTHOPEDIC MCQS ONLINE 012 TRAUMA |
Orthopedic MCQS online 012 FOOT AND ANKLE |
ORTHOPEDIC MCQS ONLINE 013 PEDIATRIC |
ORTHOPEDIC MCQS ONLINE 013 SPORT |
ORTHOPEDIC MCQS ONLINE 013 BASIC |
ORTHOPEDIC MCQS ONLINE 014 ANATOMY IMAGING |
ORTHOPEDIC MCQS ONLINE 014 UPPER EXTREMITY |
ORTHOPEDIC MCQS ONLINE 014 PATHOLOGY |
ORTHOPEDIC MCQS ONLINE 015 TRAUMA |
ORTHOPEDIC MCQS ONLINE 015 FOOT AND ANKLE e |
ORTHOPEDIC MCQS ONLINE 015Spine |
ORTHOPEDIC MCQS ONLINE PEDIATRIC 016 |
ORTHOPEDIC MCQS ONLINE RECONSTRUCTION 016 |
ORTHOPEDIC MCQS ONLINE SPORT016 |
ORTHOPEDIC MCQS ONLINE HAND 017 |
ORTHOPEDIC MCQS ONLINE PATHOLOGY 017 |
Orthopedic MCQS online Shoulder and Elbow 017 |
Orthopedic MCQS online Anatomy 017 |
Orthopedic MCQS online Basic 018 |
Orthopedic MCQS online Spine 0018 |
Orthopedic MCQS Trauma 0018 |
Orthopedic MCQS RECON0019 |
Orthopedic Mcqs Sport 0019 |
Orthopedics Mcqs Hand0019 |
ORTHO MCQS RECON019 |
HAND AND WRIST MCQS 019 |
ORTHO MCQS Shoulder and Elbow 0192 |
ORTHO MCQS Shoulder and Elbow 019 |
ORTHO MCQS PEDS 10 |
ORTHO MCQS SPORT 10 |
ORTHO MCQS BANK 011 FREE 04 |
ORTHO MCQS BANK 011 FREE 03 |
ORTHO MCQS 011 FREE BANK 02 |
ORTHO MCQS 011 FREE BANK |
Orthopedic MCQS online Hip and knee ADULT... |
ORTHOPEDIC MCQS OB 20 TRAUMA1 |
ORTHOPEDIC MCQS OB 20 BASIC5 |
ORTHOPEDIC MCQS OB 20 BASIC7 |
ORTHOPEDIC MCQS OB 20 BASIC 6 |
ORTHOPEDIC MCQS OB 20 BASIC 44 |
ORTHOPEDIC MCQS OB 20 BASIC 4 |
ORTHOPEDIC MCQS OB 20 BASIC 3 |
ORTHOPEDIC MCQS OB 20 BASIC 2 |
ORTHOPEDIC MCQS OB 20 BASIC 1 |
ORTHOPEDIC MCQS OB 20 SHOULDER AND ELBOW4 |
ORTHOPEDIC MCQS OB 20 SHOULDER AND ELBOW3 |
ORTHOPEDIC MCQS OB 20 SHOULDER AND ELBOW 2 |
1ORTHOPEDIC MCQS OB 20 SHOULDER AND ELBOW |
ORTHOPEDIC MCQS BANK WITH ANSWER HIP 01 |
ORTHOPEDIC MCQS BANK WITH ANSWER PEDS 01 |
ORTHOPEDIC MCQS BANK WITH ANSWER SPORT 01 |
ORTHOPEDIC MCQS BANK WITH ANSWER ANATOMY 02 |
ORTHOPEDIC MCQS BANK WITH ANSWER PATHOLOGY 02 |
ORTHOPEDIC MCQS BANK WITH ANSWER SHOULDER 02 |
ORTHOPEDIC MCQS WITH ANSWER FOOT 03 |
ORTHOPEDIC MCQS WITH ANSWER SPINE 03 |
ORTHOPEDIC MCQS WITH ANSWER TRAUMA 03 |
ORTHOPEDIC MCQS WITH ANSWER HIP 04 |
ORTHOPEDIC MCQS WITH ANSWER PEDS 04 |
ORTHOPEDIC MCQS WITH ANSWER SPORT 04 |
ORTHOPEDIC MCQS WITH ANSWER ANATOMY 05 |
ORTHOPEDIC MCQS WITH ANSWER TUMOR/ONCOLOGY 05 |
ORTHOPEDIC MCQS WITH ANSWER UPPER LIMB 05 |
ORTHOPEDIC MCQS WITH ANSWERS ONLINE SPINE 06 |
ORTHOPEDIC MCQS WITH ANSWERS ONLINE FOOT AND ANKLE... |
ORTHOPEDIC MCQS WITH ANSWERS ONLINE TRAUMA 06 |
ORTHOPEDIC MCQS with Answers ONLINE BASIC 06 |
ORTHOPEDIC MCQS ONLINE PEDIATRICS 07 |
ORTHOPEDIC MCQS ONLINE HIP AND KNEE RECON 07 |
ONLINE ORTHOPEDIC MCQS SPORT07 |
ONLINE ORTHOPEDIC MCQS UPPER LIMB08 |
ONLINE ORTHOPEDIC MCQS ONCOLOGY/TUMOR08 |
ONLINE ORTHOPEDIC MCQS ANATOMY08 |
ONLINE ORTHOPEDIC MCQS FOOT0 9 |
ONLINE ORTHOPEDIC MCQS SPINE0 9 |
ONLINE ORTHOPEDIC MCQS TRAUMA 9 |
Orthopedic MCQS online sports Medicine |
Orthopedic MCQS online Shoulder and Elbow |
Orthopedic MCQS online Hip and knee |
online orthopedic mcqs |
Shoulder and elbow: Mcqs AND EMQS Answers |
Shoulder And Elbow: Questions Mcqs AND EMQS |
Hand and wrist: Answers MCQS EMQS |
Hand and wrist: MCQ AND EMQ Questions |
2021 SHOULDER AND ELBOW MCQS FREE |
Pediatric Orthopaedic MCQS Self-Assessment... |
Self-Assessment Examination 2020 Adult Spine MCQS |
Foot and Ankle free MCQS2020 Online |
UPDATED ORTHOPEDIC MCQS |
FREE Orthopedics MCQS 2022 1951.-2000. |
FREE Orthopedics MCQS 2022 1901.-1950. |
FREE Orthopedics MCQS 2022 1851-1900. |
FREE Orthopedics MCQS 2022 1751-1850.. |
FREE Orthopedics MCQS 2022 1751-1800.. |
Foot and Ankle FREE ORTHOPEDICS MCQS Question 11 |
FREE Orthopedics MCQS 2022 1701-1750. |
FREE Orthopedics MCQS 2022 1651-1700 |
FREE Orthopedics MCQS 2022 1601-1650. |
ORTHOPEDIC MCQS FREE 2023 |
FREE Orthopedics MCQS 2022 1551-1600 |
FREE Orthopedics MCQS 2022 1501-1550 |
FREE Orthopedics MCQS 2022 1451-1500 |
FREE Orthopedics MCQS 2022 1401-1450 |
FREE Orthopedics MCQS 2022 1351 -1400 |
ORTHOPEDICS HYPERGUIDE 2022 MCQ-1301-1350 |
ORTHOPEDICS HYPERGUIDE 2022 MCQ-1251-1300 |
ORTHOPEDICS HYPERGUIDE 2022 MCQ-1151-1200 |
ORTHOPEDICS HYPERGUIDE 2022 MCQ-1101 1150 |
ORTHOPEDICS HYPERGUIDE 2022 MCQ1051-1100 |
ORTHOPEDICS HYPERGUIDE 2022 MCQ1001-1051 |
ORTHOPEDICS HYPERGUIDE MCQ 951-1000 |
ORTHOPEDICS HYPERGUIDE MCQ 901-950 |
ORTHOPEDICS HYPERGUIDE MCQ 851-900 |
ORTHOPEDICS HYPERGUIDE MCQ 800-850 |
ORTHOPEDICS HYPERGUIDE MCQ 751-800 |
ORTHOPEDICS HYPERGUIDE MCQ 701-750 |
ORTHOPEDICS HYPERGUIDE MCQ 651-700 |
ORTHOPEDICS HYPERGUIDE MCQ 601-650 |
ORTHOPEDICS HYPERGUIDE MCQ 551-600 |
ORTHOPEDICS HYPERGUIDE MCQ 501-550 |
ORTHOPEDICS HYPERGUIDE MCQ 451-500 |
ORTHOPEDICS HYPERGUIDE MCQ 401-450 |
ORTHOPEDICS HYPER
ORTHOPEDIC MCQS ONLINE BANK OITE 22 FOR OITE 22 FIGURES CLICK OITE22FIG 02.1
back
for more mcqs in the bank for more mcqs in the bank GUIDE MCQ 351-400 |
ORTHOPEDICS HYPERGUIDE MCQ 301-350 |
ORTHOPEDICS HYPERGUIDE MCQ 251-300 |
ORTHOPEDICS HYPERGUIDE MCQ 201-250 |
ORTHOPEDICS HYPERGUIDE MCQ 151-200 |
ORTHOPEDICS HYPERGUIDE MCQ 101-150 |
FREE Orthopedics MCQS 2022 51-100 |
Orthopedics |